Chapter 55: Assessment of Nervous System Lewis: Medical-Surgical Nursing, 10th Edition, Lewis Chapter 62: Musculoskeletal Trauma and Orthopedic Surgery, Lewis 10th Chapter 58 Chronic Neurologic Problems Evolve NCLEX practice, Med-Surg: Lewis: Chapter...

Réussis tes devoirs et examens dès maintenant avec Quizwiz!

the following are characteristic of which type of hematoma? -bleeding into brain mass -1-14 days of injury -<LOC -contralateral hemiparesis -changes in respiration -permanent neuro deficits -poor prognosis -r/t HTN and trauma

subacute subdural

onset of encephalitis is:

sudden

symptoms of encephalitis include

sudden fever severe headache stiff neck vomiting and drowsiness any CNS abnormality increased ICP (severe) shock (severe)

What type of craniectomy includes >HOB 30-45 degrees >neutral alignment >avoid positioning on operative side

supratentorial

management for viral encephalitis is

symptomatic and supportive

a "penetrating" brain injury means:

tearing of the brain, skull fracture or gunshot wound

The extent of neuro deterioration from compensatory volume changes is determined by:

the extent of compression of the brain tissue

How does manitol decrease ICP? (2 modes)

1. plasma expansion (reduces hematocrit and blood viscosity; increases CBF and O2 delivery) 2. osmotic effect (fluid moves from tissues into blood vessels-- reduces ICP by decrease in total brain fluid content)

3 treatment principles for head injuries:

1. prevent secondary injury 2. timely dx 3. surgery if necessary

right temporal lobe

recognition of other sound memories (music, animal sounds etc.)

left temporal lobe

recognition of spoken language memories

cases of bacterial meningitis must be...

reported to CDC (mandatory! )

Halo Traction

Consists of a headpiece with four pins, two anterior and two posterior May be attached to a vest

Temporal lobe

contains auditory area for receiving and interpreting impulses from the ear

how to avoid hypoxia in treatment of ICP?

controlled hyperventilation -- keep CO2 of 35 in ICU

meningococcal conjugate vaccine is designed to offer protection agains:

four serogroups of neisseria meningitidis (A, C, V, and W), which account for approximately 70% of cases in the US

type of brain tumor largely dependent on family history

gliomas

for a nasal fluid sample with blood in it, this radiologic finding indicates the presence of CSF: and indicates which diagnosis?

halo's sign temporal arteritis (damage/inflammation to temporal arteries)

s/s of meningitis include (big 4:)

headache nausea vomiting fever

fractures evolve over...

hours

self perception-self concept

how you feel about yourself general emotional pattern

fluid treatment for increased ICP:

hypERtonic saline (3%NS) (moves water out of cells and into blood)

damage to temporal lobe manifestations

inability to recognize written or spoken language, inability to recognize music or environment noises

Changes in vital signs with intracranial pressure are due to:

increasing pressure on the pons, medulla, hypothalamus, and thalamus bradycardia may be due to pressure & stimulation of vagus nerve

complications and deaths in encephalitis are more common in:

infants and elderly

Complications of skull fractures may include

infections hematoma tissue damage

What type of craniectomy includes >straight alignment >position on side

infratentorial

When obtaining a health history and physical assessment for a patient with possible multiple sclerosis (MS), the nurse should _____________

inquire about any urinary tract problems. rational: Urinary tract problems with incontinence or retention are common symptoms of MS. Chest pain and skin rashes are not symptoms of MS. A decrease in libido is common with MS.

An inflammation of the meninges-- the membranes that cover the brain (dura mater, arachnoid mater, & pia mater) is called

meningits

brain tumors may be the result of

metastatic cancer of the lungs or breast

Interventions for ICP

monitor vitals keep HOB at 30 degrees monitor neuro status I & O

Nutrition guideline for ICP =

parenteral or enteral feedings within 3 days keep patient normovolemic watch I&O super carefully

i meningitis is ___________ in nature, CSF is cloudy and pressure is elevated, glucose is decreased, protein is elevated and WBC and RBCs are elevated

bacterial

when a patient with ICP is being given barbituates, how is it dosed?

based on bedside EEG tracing and the pt's ICP

encephalitis results in degeneration of the neurons of the :

cortex

excision of a portion of the skull

craniectomy

opening the skull surgically

craniotomy

main treatment for subdural and epidural hematomas

craniotomy craniectomy

Nursing interventions that INCREASE ICP ("DONT'S") =

-Neck flexion -head roatation -clumping activities -ETT suctioning (hyperventilate with 100% O2 prior to suction when it MUST be done) -coughing -straining -valsalva maneuver -loud noise/conversations

Broca's Area

-Non-fluent aphasia - Damage to frontal lobe -Short phases "walk dog" -"i will take the dog for a walk" -Understand speech of others and easily frustrated

nursing interventions for meningitis include:

-rest -IV fluids -hypothermia (control fever)

treatment and outcome for viral meningitis:

-symptomatic management -disease is self-limiting -full recovery expected

surgery options for ICP include:

-ventriculostomy -mass evacuation -hemicraniectomy

Difference in blood flow between white and gray matter of the brain:

-white matter has slower flow @ approx 25 mL/min per 100 g -gray matter has a faster flow @ approx 75 mL/min per 100 g

Complications of intracranial surgery post-op may include:

->BP and <pulse may indicate ICP >hypovolemia >blood clot suspected if dilated pupil on operative side >cerebral edema, ifarction, metabolic and hydrocephalus >salty taste in mouth may indicate CSF -infection

Emergency treatment for head injuries include:

-ABCs -stabilize cervical spine (assume neck injury) -IV access (admin. fluids cautiously) -score of 8, intubate (anticipate at any time) -control external bleeding -removal of pt's clothing (allows FULL assessment of body) -keep pt warm -ongoing monitoring

Wernicke's area

-Fluent aphasia -left temporal lobe -May speak in long sentences with no meaning -Difficulty understanding speech

Drug therapy for meningitis (bacterial) includes:

-IV antibiotics (ampicillin, penicillin, cephalosporin) -codeine for headache -dexamethasone -acetaminophen or aspirin for fever -phenytoin IV -mannitol IV for diuresis (decrease ICP)

4 Types of intracranial hematomas include: and are mostly differentiated by:

1. epidural 2. acute subdural 3. subacute subdural 4. chronic subdural TIME/progression/ extent of bleed

A patient with Parkinson's disease is admitted to the hospital for treatment of an acute infection. Which nursing interventions will be included in the plan of care:

-Use an elevated toilet seat -Cut patient's food into small pieces -Place an arm chair at the patient's bedside rational: Since the patient with Parkinson's has difficulty chewing, food should be cut into small pieces. An armchair should be used when the patient is seated so that the patient can use the arms to assist with getting up from the chair. An elevated toilet seat will facilitate getting on and off the toilet. High protein foods will decrease the effectiveness of L-dopa. Parkinson's is a steadily progressive disease without acute exacerbations.

drug therapy for encephalitis may include:

-acyclovir -vidarabine (*may reduce mortality but may not help CNS*) -prophylactic dilantin or other antiseizure

Post-op guidelines for intracranial surgeries include:

-avoid extreme head rotation (increases ICP) -frequent neuro chekcs (sluggish pupils for 24-48 hours) -decreased responsiveness on 2nd day post op due to edema -peiorbital edema and ecchymosis common -elevate HOB -cold compresses -NO NOSE BLOWING, COUGHING, OR SNEEZING

-ICP is maintained by: -How?

-compensatory mechanisms within the cranium -as volume of one component increases, volume of the others compensate to maintain ICP and prevent neuro changes from happening.

Examples of decreased motor function seen with increased ICP may include...

-contralateral hemiparesis -hemiplegia -possible localization or withdrawal from painful stimuli -decorticate posturing -decerebrate posturing

Drug therapy for ICP consists of:

-corticosteroids (inflammation) -osmotic diuretics -analgesics -sedation (sometimes -- uncontrolled ICP)

Examples of ocular changes with increased ICP may include (4): These findings are the result of compression of which nerve?

-dilation of the ipsilateral pupil -sluggish or no response to light -inability to move the eye upward -ptosis of the eyelid (drooping) *oculomotor nerve (cranial nerve III) if the brain shifts from midline & paralyzes the pupil sphincter

most common causes of VIRAL meningitis include (4):

-enterovirus -arbovirus -HIV & -HSV *most often spread through direct contact with respiratory secretions*

temperature maintenance guidelines for ICP include:

-fever well-controlled (acetaminophen) -cool baths, blankets, ice pack, or intravascular cooling devices -avoid shivering (sedatives may be needed)

Examples of "change in LOC" assessment findings for increased ICP include:

-flattening of affect -change in orientation -decreased level of attention -dramatic

4 classes of brain tumor based on location

-gliomas -meningiomas -pituitary gland -acoustic neuromas

prevention of meningitis methods include the following:

-haemophilus vaccine (HIB) -MENINGOCOCCAL CONJUCATE VACCINE -early tx of respiratory and ear infections -prophylactic ATB

VIRAL meningitis usually presents with these clinical s/s's:

-headache -moderate to high fever -photophobia & -stiff neck

symptoms of brain tumors may include

-headaches -N/V -visual symptoms (papilledema on retinal exam) -sensory disturbances -seizures -changes in mentation or personality -local: focal weakness

with encephalitis, the following (4) complications develop in cerebral hemispheres:

-hemorrhage -edema -necrosis -small lacunae

occupational risks for brain tumors include:

-high levels of radiation -formaldehyde exposure -vinyl chloride -chemicals

complications of corticosteroids include:

-hyperglycemia -increased incidence of infections -GI bleeding

What are examples of "changes in VSs" assessment findings for increased ICP?

-increasing systolic pressure -bradycardia -pulse full and bounding -irregular respirations (increased BP and respirations decreased pulse and temperature?? --double check online quiz)

diagnostics for encephalitis

-lumbar puncture (CSF pressure elevated but fluid clear) -blood test for west nile

Diagnostic studies for acute brain/head injuries include:

-lumbar puncture with blood in CSF -MRI -PET scan -Doppler -CT scan (best) -GCS!

the most common causes of bacterial meningitis are (4)

-meningococcus (contagious) -streptococcus -staph & -pneumococcus

why is the pt's temperature important with ICP?

-metabolic demands such as fever, agitation/shivering, pain, and seizures can increase ICP

treatment for concussions include:

-mild analgesics -return to ER if not resolved or postconcussion syndrome suspected

other assessment findings for meningitis may include:

-photophobia -indications of ICP -nuchal rigidity -positive kernig's sign -positive brudzinski's sign -seizure -decreased mentation -focal neurologic defects

Manitol is contraindicated for (3):

-pulmonary edema -renal disease -elevated serum osmolality

Nursing interventions that DECREASE ICP ("DO'S") =

-purposeful touch -good body & head alignment -change position slowly -HOB elevated at least 30 -family visits -favorite music

Nonsurgical interventions for brain tumors include:

-radiation/chemo -blood-brain-barrier disruption -recombinant DNA/monoclonal antibodies -stereotactic radiosurgery -dexamethasone & anticonvulsants -immunotherapy/ hyperthermia

Complications of contusions may include:

-rebleeding -seizures -potential for increased hemorrhage if on anticoagulants

Effect of increased PaCO2 has what effect on brain?

-relaxes smooth muscle -dilates cerebral vessels -decreases cerebrovascular resistance -increases cerebra blood flow (CBF) (and vice versa for decreased PaCO2)

Diagnostic studies-neurologic

1) CSF 2) cerebrial angiography (catheter into femoral or brachial artery) 3) CT 4) MRI 5) PET 6) myelography 7) EEG 8)electromyography and nerve conduction (eg. carpel tunnel test) 9) carotid duplex (ultrasound)

CSF assessment normal values

1) RBC's should not be present 2) pH 7.35 3) glucose 45-74 4) protein 15-45 5) albumin 10-30 6) CSF pressure 60-150 7) WBC 0-8

Six major indicators of increased ICP include:

1) change in LOC 2) change in vital signs 3) ocular signs 4) decreased motor function 5) headache 6) vomiting

Circle of Willis

1) communicating arteries that join the basilar and internal carotid arteries 2) regulates cerebral blood flow 3) acts as safety valve when pressure changes are present

pons

1) connect link between cerebellum and rest of the nervous system 2) cranial nerves V (Trigeminal: connection with pons (sensory), VI (Abducens-connection with pons (motor),VII (Facial; movement of facial muscles, secretions of lacrimal and salivary glands, taste ), VIII (Vestibulocochlear; sense of hearing and equilibrium )

parietal lobe

1) contains sensory cortex 2) interprets spatial information 3) interprets pain, temperature to skin 4) right-left orientation 5) mathematics

midbrain

1) cranial nerves III (contains sensory cortex)and IV ( Abducens-connection with pons (motor) lateral rectus of the eye) originate here 2) center for certain reflexes of the eye and ear, such as moving the eyes to read

nervous system assessment

1) health perception-health management 2) nutritional-metabolic 3) elimination 4) activity-exercise 5) sleep-rest 6) cognitive-perceptual 7) self perception-self control 8) role-relationship 9) sexuality-reproductive 10) coping-stress tolerance 11) value-belief

frontal lobe function

1) higher cognitive function 2) primary motor area, voluntary motor movement of skeletal muscle 3) memory retention 4) voluntary eye movement

cerebellum

1) maintains trunk stability and equilibrium, 2) coordinates voluntary muscles ensuring smooth orderly function 3) maintains muscle tone

occipital lobe

1) receives visual information, 2) stores visual memories 3) interprets impulses from the retina (such as retaining what you have read)

CFS

1) surrounds brain stem and spinal cord 2) 500 mL is produced daily

medulla oblongata

1) vital centers such as respiratory( muscles of respiration) and cardiac (regulate the rate and force of heart beat) 2) connected with cranial nerves XI (Accessory; neck and shoulder movement ) and XII (Hypoglossal; tongue movement )

subjective data: neurology assessment

1)determine mental functioning 2)assess medications 3)past or present neurologic problems 4)growth and development history

prefrontal area

1)reasoning 2) problem solving 3) emotional stability, concentration, motivation, 4) ability to formulate goals and plan, to initiate, maintain or terminate actions 5) ability to self-monitor 6) ability to use feedback

objective data: neurology assessment

1)vital signs 2)mental status 3) orientation x4 4) memory 5) mood and affect 6) intellectual performance 7) head, neck and back inspection 8) corneal, cough, gag reflex 9) cranial nerves III-VII assessment 10) motor function 11) sensory function

13 Steps outlining the progression of ICP:

1. insult to brain 2. tissue edema 3. increased ICP 4. compression of ventricles 5. compression of blood vessels 6. decreased cerebral blood flow 7. decreased O2 with death of brain cells 8. edema around necrotic tissue 9. increased ICP with compression of brainstem and respiratory center 10. accumulation of CO2 11. vasodilation 12. increased ICP resulting from increased blood volume 13. DEATH

1. In a patient with a disease that affects the myelin sheath of nerves, such as multiple sclerosis, the glial cells affected are the a. microglia. b. astrocytes. c. ependymal cells. d. oligodendrocytes.

1. d

10. The nurse is caring for a patient with peripheral neuropathy who is scheduled for EMG studies tomorrow morning. The nurse should a. ensure the patient has an empty bladder. b. instruct the patient about the risk of electric shock. c. ensure the patient has no metallic jewelry or metal fragments. d. instruct the patient that pain may be experienced during the study.

10. d

10. Which type of seizure is most likely to cause death for the patient? a. Subclinical seizures b. Myoclonic seizures c. Psychogenic seizures d. Tonic-clonic status epilepticus

10. d. Tonic-clonic status epilepticus is most dangerous because the continuous seizing can cause respiratory insufficiency, hypoxemia, cardiac dysrhythmia, hyperthermia, and systemic acidosis, which can all be fatal. Subclinical seizures may occur in a patient who is sedated, so there is no physical movement. Myoclonic seizures may occur in clusters and have a sudden, excessive jerk of the body that may hurl the person to the ground. Psychogenic seizures are psychiatric in origin and diagnosed with videoelectroencephalography (EEG) monitoring. They occur in patients with a history of emotional abuse or a specific traumatic episode.

11. A patient admitted to the hospital following a generalized tonic-clonic seizure asks the nurse what caused the seizure. What is the best response by the nurse? a. "So many factors can cause epilepsy that it is impossible to say what caused your seizure." b. "Epilepsy is an inherited disorder. Does anyone else in your family have a seizure disorder?" c. "In seizures, some type of trigger causes sudden, abnormal bursts of electrical brain activity." d. "Scar tissue in the brain alters the chemical balance, creating uncontrolled electrical discharges."

11. c. A seizure is a paroxysmal, uncontrolled discharge of neurons in the brain, which interrupts normal function, but the factor that causes the abnormal firing is not clear. Seizures may be precipitated by many factors and although scar tissue may make the brain neurons more likely to fire, it is not the usual cause of seizures. Epilepsy is established only by a pattern of spontaneous, recurring seizures.

meningococcal conjugate vaccine is safe for people aged :

12-55 yrs

12. A patient with a seizure disorder is being evaluated for surgical treatment of the seizures. The nurse recognizes that what is one of the requirements for surgical treatment? a. Identification of scar tissue that is able to be removed b. An adequate trial of drug therapy that had unsatisfactory results c. Development of toxic syndromes from long-term use of antiseizure drugs d. The presence of symptoms of cerebral degeneration from repeated seizures

12. b. Most patients with seizure disorders maintain seizure control with medications but if surgery is considered, three requirements must be met: the diagnosis of epilepsy must be confirmed, there must have been an adequate trial with drug therapy without satisfactory results, and the electroclinical syndrome must be defined. The focal point must be localized but the presence of scar tissue is not required.

13. The nurse teaches the patient taking antiseizure drugs that this method is most commonly used to measure compliance and monitor for toxicity. a. A daily seizure log b. Urine testing for drug levels c. Blood testing for drug levels d. Monthly electroencephalography (EEG)

13. c. Serum levels of antiseizure drugs are monitored regularly to maintain therapeutic levels of the drug, above which patients are likely to experience toxic effects and below which seizures are likely to occur. Many newer drugs do not require drug level monitoring because of large therapeutic ranges. A daily seizure log and urine testing for drug levels will not measure compliance or monitor for toxicity. EEGs have limited value in diagnosis of seizures and even less value in monitoring seizure control.

14. Priority Decision: When teaching a patient with a seizure disorder about the medication regimen, what is it most important for the nurse to emphasize? a. The patient should increase the dosage of the medication if stress is increased. b. Most over-the-counter and prescription drugs are safe to take with antiseizure drugs. c. Stopping the medication abruptly may increase the intensity and frequency of seizures. d. If gingival hypertrophy occurs, the drug should be stopped and the health care provider notified.

14. c. If antiseizure drugs are discontinued abruptly, seizures can be precipitated. Missed doses should be made up if the omission is remembered within 24 hours and patients should not adjust medications without professional guidance because this also can increase seizure frequency and may cause status epilepticus. Antiseizure drugs have numerous interactions with other drugs and the use of other medications should be evaluated by health professionals. If side effects occur, the physician should be notified and drug regimens evaluated.

Following a generalized tonic-clonic seizure, the patient is tired and sleepy. What care should the nurse provide? a. Suction the patient before allowing him to rest. b. Allow the patient to sleep as long as he feels sleepy. c. Stimulate the patient to increase his level of consciousness. d. Check the patient's level of consciousness every 15 minutes for an hour.

16. b. In the postictal phase of generalized tonic-clonic seizures, patients are usually very tired and may sleep for several hours and the nurse should allow the patient to sleep as long as necessary. Suctioning is performed only if needed and decreased level of consciousness is not a problem postictally unless a head injury has occurred during the seizure.

17. During the diagnosis and long-term management of a seizure disorder, what should the nurse recognize as one of the major needs of the patient? a. Managing the complicated drug regimen of seizure control b. Coping with the effects of negative social attitudes toward epilepsy c. Adjusting to the very restricted lifestyle required by a diagnosis of epilepsy d. Learning to minimize the effect of the condition in order to obtain employment

17. b. One of the most common complications of a seizure disorder is the effect it has on the patient's lifestyle. This is because of the social stigma attached to seizures, which causes patients to hide their diagnosis and to prefer not to be identified as having epilepsy. Medication regimens usually require only once- or twice-daily dosing and the major restrictions of lifestyle usually involve driving and high-risk environments. Job discrimination against the handicapped is prevented by federal and state laws and patients only need to identify their disease in case of medical emergencies.

How many mL of blood is needed for every 100 gm of brain tissue in order to maintain a normal metabolic state? Amount of blood in mL passing thru 100 g of brain tissue in 1 minute:

50 to 55 mL per 100 gm CBF -- cerebral blood flow

19. Which chronic neurologic disorder involves a deficiency of the neurotransmitters acetylcholine and γ aminobutyric acid (GABA) in the basal ganglia and extrapyramidal system? a. Myasthenia gravis b. Parkinson's disease c. Huntington's disease d. Amyotrophic lateral sclerosis (ALS)

19. c. Huntington's disease (HD) involves deficiency of acetylcholine and γ-aminobutyric acid (GABA) in the basal ganglia and extrapyramidal system that causes the opposite symptoms of parkinsonism. Myasthenia gravis involves autoimmune antibody destruction of cholinergic receptors at the neuromuscular junction. Amyotrophic lateral sclerosis (ALS) involves degeneration of motor neurons in the brainstem and spinal cord.

The patient has an order for phenytoin (Dilantin) 100 mg q8hr IV. Available is a phenytoin injection containing 50 mg/mL. How many milliliters of solution should the nurse draw up for the dose? _____mL

2 100 mg ÷ 50 mg/mL = 2 mL

signs of encephalitis appear within _____________ of infection

2 to 3 days

recovery time for encephalitis is usually _________________ (unless severe)

2-3 weeks

2. Drugs or diseases that impair the function of the extrapyramidal system may cause loss of a. sensations of pain and temperature. b. regulation of the autonomic nervous system. c. integration of somatic and special sensory inputs. d. automatic movements associated with skeletal muscle activity.

2. d

A patient who is having an acute exacerbation of multiple sclerosis has a prescription for methylprednisolone (Solu-Medrol) 160 mg IV. The label on the vial reads: methylprednisolone 125 mg in 2 mL. How many mL will the nurse administer?

2.56. With a concentration of 125 mg/2 mL, the nurse will need to administer 2.56 mL to obtain 160 mg of methylprednisolone.

20. A 38-year-old woman has newly diagnosed multiple sclerosis (MS) and asks the nurse what is going to happen to her. What is the best response by the nurse? a. "You will have either periods of attacks and remissions or progression of nerve damage over time." b. "You need to plan for a continuous loss of movement, sensory functions, and mental capabilities." c. "You will most likely have a steady course of chronic progressive nerve damage that will change your personality." d. "It is common for people with MS to have an acute attack of weakness and then not to have any other symptoms for years."

20. a. Most patients with multiple sclerosis (MS) have remissions and exacerbations of neurologic dysfunction or a relapsing-remitting initial course followed by progression with or without occasional relapses, minor remissions, and plateaus that progressively cause loss of motor, sensory, and cerebellar functions. Intellectual function generally remains intact but patients may experience anger, depression, or euphoria. A few people have chronic progressive deterioration and some may experience only occasional and mild symptoms for several years after onset.

21. During assessment of a patient admitted to the hospital with an acute exacerbation of MS, what should the nurse expect to find? a. Tremors, dysphasia, and ptosis b. Bowel and bladder incontinence and loss of memory c. Motor impairment, visual disturbances, and paresthesias d. Excessive involuntary movements, hearing loss, and ataxia

21. c. Specific neurologic dysfunction of MS is caused by destruction of myelin and replacement with glial scar tissue at specific areas in the nervous system. Motor, sensory, cerebellar, and emotional dysfunctions, including paresthesias as well as patchy blindness, blurred vision, pain radiating along the dermatome of the nerve, ataxia, and severe fatigue, are the most common manifestations of MS. Constipation and bladder dysfunctions, short-term memory loss, sexual dysfunction, anger, and depression or euphoria may also occur. Excessive involuntary movements and tremors are not seen in MS.

23. Mitoxantrone (Novantrone) is being considered as treatment for a patient with progressive-relapsing MS. The nurse explains that a disadvantage of this drug compared with other drugs used for MS is what? a. It must be given subcutaneously every day. b. It has a lifetime dose limit because of cardiac toxicity. c. It is an anticholinergic agent that causes urinary incontinence. d. It is an immunosuppressant agent that increases the risk for infection.

23. b. Mitoxantrone (Novantrone) cannot be used for more than 2 to 3 years because it is an antineoplastic drug that causes cardiac toxicity, leukemia, and infertility. It is a monoclonal antibody given IV monthly when patients have inadequate responses to other drugs. It increases the risk of progressive multifocal leukoencephalopathy.

6. The nurse is assessing the muscle strength of an older adult patient. The nurse knows the findings cannot be compared with those of a younger adult because a. nutritional status is better in young adults. b. muscle bulk and strength decrease in older adults. c. muscle strength should be the same for all adults. d. most young adults exercise more than older adults.

6. b,

What is a normal CPP measurement?

60 to 100 mmHg

Priority Decision: A patient with MS has a nursing diagnosis of self-care deficit related to muscle spasticity and neuromuscular deficits. In providing care for the patient, what is most important for the nurse to do? a. Teach the family members how to care adequately for the patient's needs. b. Encourage the patient to maintain social interactions to prevent social isolation. c. Promote the use of assistive devices so the patient can participate in self-care activities. d. Perform all activities of daily living (ADLs) for the patient to conserve the patient's energy.

24. c. The main goal in care of the patient with MS is to keep the patient active and maximally functional and promote self-care as much as possible to maintain independence. Assistive devices encourage independence while preserving the patient's energy. No care activity that the patient can do for himself or herself should be performed by others. Involvement of the family in the patient's care and maintenance of social interactions are also important but are not the priority in care.

25. A patient with newly diagnosed MS has been hospitalized for evaluation and initial treatment of the disease. Following discharge teaching, the nurse realizes that additional instruction is needed when the patient says what? a. "It is important for me to avoid exposure to people with upper respiratory infections." b. "When I begin to feel better, I should stop taking the prednisone to prevent side effects." c. "I plan to use vitamin supplements and a high-protein diet to help manage my condition." d. "I must plan with my family how we are going to manage my care if I become more incapacitated."

25. b. Corticosteroids used in treating acute exacerbations of MS should not be abruptly stopped by the patient because adrenal insufficiency may result and prescribed tapering doses should be followed. Infections may exacerbate symptoms and should be avoided and high-protein diets with vitamin supplements are advocated. Long-term planning for increasing disability is also important.

26. The classic triad of manifestations associated with Parkinson's disease is tremor, rigidity, and bradykinesia. What is a consequence related to rigidity? a. Shuffling gait b. Impaired handwriting c. Lack of postural stability d. Muscle soreness and pain

26. d. The degeneration of dopamine-producing neurons in the substantia nigra of midbrain and basal ganglia lead to this triad of signs. Muscle soreness, pain, and slowness of movement are patient function consequences related to rigidity. Shuffling gait, lack of postural stability, absent arm swing while walking, absent blinking, masked facial expression, and difficulty initiating movement are all related to bradykinesia. Impaired handwriting and hand activities are related to the tremor of Parkinson's disease (PD).

27. A patient with a tremor is being evaluated for Parkinson's disease. The nurse explains to the patient that Parkinson's disease can be confirmed by a. CT and MRI scans. b. relief of symptoms with administration of dopaminergic agents. c. the presence of tremors that increase during voluntary movement. d. cerebral angiogram that reveals the presence of cerebral atherosclerosis.

27. b. Although clinical manifestations are characteristic in PD, no laboratory or diagnostic tests are specific for the condition. A diagnosis is made when at least two of the three signs of the classic triad are present and it is confirmed with a positive response to antiparkinsonian medication. Research regarding the role of genetic testing and MRI to diagnose PD is ongoing. Essential tremors increase during voluntary movement whereas the tremors of PD are more prominent at rest.

28. Which observation of the patient made by the nurse is most indicative of Parkinson's disease? a. Large, embellished handwriting b. Weakness of one leg resulting in a limping walk c. Difficulty rising from a chair and beginning to walk d. Onset of muscle spasms occurring with voluntary movement

28. c. The bradykinesia of PD prevents automatic movements and activities such as beginning to walk, rising from a chair, or even swallowing saliva cannot be executed unless they are consciously willed. Handwriting is affected by the tremor and results in the writing trailing off at the end of words. Specific limb weakness and muscle spasms are not characteristic of PD.

29. A patient with Parkinson's disease is started on levodopa. What should the nurse explain about this drug? a. It stimulates dopamine receptors in the basal ganglia. b. It promotes the release of dopamine from brain neurons. c. It is a precursor of dopamine that is converted to dopamine in the brain. d. It prevents the excessive breakdown of dopamine in the peripheral tissues.

29. c. Peripheral dopamine does not cross the blood-brain barrier but its precursor, levodopa, is able to enter the brain, where it is converted to dopamine, increasing the supply that is deficient in PD. Other drugs used to treat PD include bromocriptine, which stimulates dopamine receptors in the basal ganglia, and amantadine, which blocks the reuptake of dopamine into presynaptic neurons. Carbidopa is an agent that is usually administered with levodopa to prevent the levodopa from being metabolized in peripheral tissues before it can reach the brain.

3. During the admitting neurologic examination, the nurse determines the patient has speech difficulties as well as weakness of the right arm and lower face. The nurse would expect a CT scan to show pathology in the distribution of the a. basilar artery. b. left middle cerebral artery. c. right anterior cerebral artery. d. left posterior communicating artery.

3. b,

31. A patient with myasthenia gravis is admitted to the hospital with respiratory insufficiency and severe weakness. When is a diagnosis of cholinergic crisis made? a. The patient's respiration is impaired because of muscle weakness. b. Administration of edrophonium (Tensilon) increases muscle weakness. c. Administration of edrophonium (Tensilon) results in improved muscle contractility. d. EMG reveals decreased response to repeated stimulation of muscles.

31. b. The reduction of the acetylcholine (ACh) effect in myasthenia gravis (MG) is treated with anticholinesterase drugs, which prolong the action of ACh at the neuromuscular synapse, but too much of these drugs will cause a cholinergic crisis with symptoms very similar to those of MG. To determine whether the patient's manifestations are due to a deficiency of ACh or to too much anticholinesterase drug, the anticholinesterase drug edrophonium chloride (Tensilon) is administered. If the patient is in cholinergic crisis, the patient's symptoms will worsen; if the patient is in a myasthenic crisis, the patient will improve.

Priority Decision: During care of a patient in myasthenic crisis, maintenance of what is the nurse's first priority for the patient? a. Mobility b. Nutrition c. Respiratory function d. Verbal communication

32. c. The patient in myasthenic crisis has severe weakness and fatigability of all skeletal muscles, affecting the patient's ability to breathe, swallow, talk, and move. However, the priority of nursing care is monitoring and maintaining adequate ventilation.

33. When providing care for a patient with ALS, the nurse recognizes what as one of the most distressing problems experienced by the patient? a. Painful spasticity of the face and extremities b. Retention of cognitive function with total degeneration of motor function c. Uncontrollable writhing and twisting movements of the face, limbs, and body d. Knowledge that there is a 50% chance the disease has been passed to any offspring

33. b. In ALS there is gradual degeneration of motor neurons with extreme muscle wasting from lack of stimulation and use. However, cognitive function is not impaired and patients feel trapped in a dying body. Chorea manifested by writhing, involuntary movements is characteristic of HD. As an autosomal dominant genetic disease, HD also has a 50% chance of being passed to each offspring.

34. In providing care for patients with chronic, progressive neurologic disease, what is the major goal of treatment that the nurse works toward? a. Meet the patient's personal care needs. b. Return the patient to normal neurologic function. c. Maximize neurologic functioning for as long as possible. d. Prevent the development of additional chronic diseases.

34. c. Many chronic neurologic diseases involve progressive deterioration in physical or mental capabilities and have no cure, with devastating results for patients and families. Health care providers can only attempt to alleviate physical symptoms, prevent complications, and assist patients in maximizing function and self-care abilities for as long as possible.

4. A patient is seen in the emergency department after diving into the pool and hitting the bottom with a blow to the face that hyperextended the neck and scraped the skin off the nose. The patient also described "having double vision" when looking down. During the neurologic exam, the nurse finds the patient is unable to abduct either eye. The nurse recognizes this finding is related to a. a basal skull fracture. b. a stretch injury to bilateral CN VI. c. a stiff neck from the hyperextension injury. d. facial swelling from the scrape on the bottom of the pool.

4. b,

What is a normal CSF value?

5 to 13 mmHg

What is a normal ICP?

5 to 15 mmHg

5. Stimulation of the parasympathetic nervous system results in (select all that apply) a. constriction of the bronchi. b. dilation of skin blood vessels. c. increased secretion of insulin. d. increased blood glucose levels. e. relaxation of the urinary sphincters.

5. a, b, c, e,

Corticosteroids recommended for _________________-related ICP but not for ____________patients

tumor/abscess head-injured

7. A patient is admitted with a headache, fever, and general malaise. The HCP has asked that the patient be prepared for a lumbar puncture. What is a priority nursing action to avoid complications? a. Ensure that CT scan is performed prior to lumbar puncture. b. Assess laboratory results for changes in the white cell count. c. Provide acetaminophen for the headache and fever before the procedure. d. Administer antibiotics before the procedure to treat the potential meningitis.

7. a,

7. How do generalized seizures differ from focal seizures? a. Focal seizures are confined to one side of the brain and remain focal in nature. b. Generalized seizures result in loss of consciousness whereas focal seizures do not. c. Generalized seizures result in temporary residual deficits during the postictal phase. d. Generalized seizures have bilateral synchronous epileptic discharges affecting the whole brain at onset of the seizure.

7. d. Generalized seizures have bilateral synchronous epileptic discharge affecting the entire brain at onset of the seizure. Loss of consciousness is also characteristic but many focal seizures also include an altered consciousness. Focal seizures begin in one side of the brain but may spread to involve the entire brain. Focal seizures that start with a local focus and spread to the entire brain, causing a secondary generalized seizure, are associated with a transient residual neurologic deficit postictally known as Todd's paralysis.

brain tumors have the highest incidence in those aged over

70

8. During neurologic testing, the patient is able to perceive pain elicited by pinprick. Based on this finding, the nurse may omit testing for a. position sense. b. patellar reflexes. c. temperature perception. d. heel-to-shin movements.

8. c,

9. A patient's eyes jerk while the patient looks to the left. The nurse will record this finding as a. nystagmus. b. CN VI palsy. c. ophthalmic dyskinesia. d. oculocephalic response.

9. a,

9. The patient is diagnosed with complex focal seizures. Which characteristics are related to complex focal seizures (select all that apply)? a. Formerly known as grand mal seizure b. Often accompanied by incontinence or tongue or cheek biting c. Psychomotor seizures with repetitive behaviors and lip smacking d. Altered memory, sexual sensations, and distortions of visual or auditory sensations e. Loss of consciousness and stiffening of the body with subsequent jerking of extremities f. Often involves behavioral, emotional, and cognitive functions with altered consciousness

9. c, d, f. Complex focal seizures are psychomotor seizures with automatisms such as lip smacking. They cause altered consciousness or loss of consciousness producing a dreamlike state and may involve behavioral, emotional, or cognitive experiences without memory of what was done during the seizure. In generalized tonic-clonic seizures (previously known as grand mal seizures) there is loss of consciousness and stiffening of the body with subsequent jerking of extremities. Incontinence or tongue or cheek biting may also occur.

Sylvestri Questions from Ch 66-67 on other Set of Flashcards

:)

When assessing a patient with a traumatic brain injury, the nurse notes uncoordinated movement of the extremities. How should the nurse document this assessment? A. Ataxia B. Apraxia C. Anisocoria D. Anosognosia

A Ataxia is a lack of coordination of movement, possibly caused by lesions of sensory or motor pathways, cerebellum disorders, or certain medications. Apraxia is the inability to perform learned movements despite having the desire and physical ability to perform them related to a cerebral cortex lesion. Anisocoria is inequality of pupil size from an optic nerve injury. Anosognosia is the inability to recognize a bodily defect or disease related to lesions in the right parietal cortex.

A 72-year-old patient with kyphosis is scheduled for dual-energy x-ray absorptiometry (DXA) testing. The nurse will plan to a. explain the procedure. b. start an IV line for contrast medium injection. c. give an oral sedative 60 to 90 minutes before the procedure. d. screen the patient for allergies to shellfish or iodine products.

A. DXA testing is painless and noninvasive. No IV access is necessary. Contrast medium is not used. Because the procedure is painless, no antianxiety medications are required.

The nurse is caring for a 76-year-old man who has undergone left knee arthroplasty with prosthetic replacement of the knee joint to relieve the pain of severe osteoarthritis. Postoperatively the nurse expects what to be included in the care of the affected leg? A Progressive leg exercises to obtain 90-degree flexion B Early ambulation with full weight bearing on the left leg C Bed rest for 3 days with the left leg immobilized in extension D Immobilization of the left knee in 30-degree flexion for 2 weeks to prevent dislocation

A Progressive leg exercises to obtain 90-degree flexion Although early ambulation is not done, the patient is encouraged to engage in progressive leg exercises until 90-degree flexion is possible. Because this is painful after surgery, the patient requires good pain management and often the use of a CPM machine. The patient's knee is unlikely to dislocate.

Proprioception

A sensory receptor, found chiefly in muscles, tendons, joints, and the inner ear, that detects the motion or position of the body or a limb by responding to stimuli arising within the organism

A patient with Parkinson's disease is admitted to the hospital for treatment of pneumonia. Which nursing interventions will be included in the plan of care (select all that apply)? a. Use an elevated toilet seat. b. Cut patient's food into small pieces. c. Provide high-protein foods at each meal. d. Place an armchair at the patient's bedside. e. Observe for sudden exacerbation of symptoms.

A, B, D Because the patient with Parkinson's has difficulty chewing, food should be cut into small pieces. An armchair should be used when the patient is seated so that the patient can use the arms to assist with getting up from the chair. An elevated toilet seat will facilitate getting on and off the toilet. High-protein foods will decrease the effectiveness of L-dopa. Parkinson's is a steadily progressive disease without acute exacerbations.

The nurse is monitoring a patient for increased ICP following a head injury. Which of the following manifestations indicate an increased ICP (select all that apply) a. fever b. oriented to name only c. narrowing pulse pressure d. dilated right pupil > left pupil e. decorticate posturing to painful stimulus

A, B, D, E- The first sign of increased ICP is a change in LOC. Other manifestations are dilated ipsilateral pupil, changes in motor response such as posturing, and fever, which may indicate pressure on the hypothalamus. Changes in vital signs would be an increased systolic BP with widened pulse pressure and bradycardia

A patient with Parkinson's disease is admitted to the hospital for treatment of pneumonia. Which nursing interventions will be included in the plan of care (select all that apply)? a. Use an elevated toilet seat. b. Cut patient's food into small pieces. c. Provide high-protein foods at each meal. d. Place an armchair at the patient's bedside. e. Observe for sudden exacerbation of symptoms.

A, B, D. Because the patient with Parkinson's has difficulty chewing, food should be cut into small pieces. An armchair should be used when the patient is seated so that the patient can use the arms to assist with getting up from the chair. An elevated toilet seat will facilitate getting on and off the toilet. High-protein foods will decrease the effectiveness of L-dopa. Parkinson's is a steadily progressive disease without acute exacerbations.

A 27-year-old patient who has been treated for status epilepticus in the emergency department will be transferred to the medical nursing unit. Which equipment should the nurse have available in the patient's assigned room (select all that apply)? a. Side-rail pads b. Tongue blade c. Oxygen mask d. Suction tubing e. Urinary catheter f. Nasogastric tube

A, C, D The patient is at risk for further seizures, and oxygen and suctioning may be needed after any seizures to clear the airway and maximize oxygenation. The bed's side rails should be padded to minimize the risk for patient injury during a seizure. Use of tongue blades during a seizure is contraindicated. Insertion of a nasogastric (NG) tube is not indicated because the airway problem is not caused by vomiting or abdominal distention. A urinary catheter is not required unless there is urinary retention.

Which information obtained by the emergency department nurse when admitting a patient with a left femur fracture is most important to report to the health care provider? a. Bruising of the left thigh b. Complaints of left thigh pain c. Outward pointing toes on the left foot d. Prolonged capillary refill of the left foot

ANS: D Prolonged capillary refill may indicate complications such as arterial damage or compartment syndrome. The other findings are typical with a left femur fracture.

A 27-year-old patient who has been treated for status epilepticus in the emergency department will be transferred to the medical nursing unit. Which equipment should the nurse have available in the patient's assigned room (select all that apply)? a. Side-rail pads b. Tongue blade c. Oxygen mask d. Suction tubing e. Urinary catheter f. Nasogastric tube

A, C, D. The patient is at risk for further seizures, and oxygen and suctioning may be needed after any seizures to clear the airway and maximize oxygenation. The bed's side rails should be padded to minimize the risk for patient injury during a seizure. Use of tongue blades during a seizure is contraindicated. Insertion of a nasogastric (NG) tube is not indicated because the airway problem is not caused by vomiting or abdominal distention. A urinary catheter is not required unless there is urinary retention.

A patient who has a neurologic disease that affects the pyramidal tract is likely to manifest what sign? A. Impaired muscle movement B. Decreased deep tendon reflexes C. Decreased level of consciousness D. Impaired sensation of touch, pain, and temperature

A. Among the most important descending tracts are the corticobulbar and corticospinal tracts, collectively termed the pyramidal tract. These tracts carry volitional (voluntary) impulses from the cortex to the cranial and peripheral nerves. Dysfunction of the pyramidal tract is likely to manifest as impaired movement because of hypertonicity. Diseases affecting the pyramidal tract do not result in changes in LOC, impaired reflexes, or decreased sensation.

The patient with type 1 diabetes mellitus with hypoglycemia is having a seizure. Which medication should the nurse anticipate administering to stop the seizure? A. IV dextrose solution B. IV diazepam (Valium) C. IV phenytoin (Dilantin) D. Oral carbamazepine (Tegretol)

A. This patient's seizure is caused by low blood glucose, so IV dextrose solution should be given first to stop the seizure. IV diazepam, IV phenytoin, and oral carbamazepine would be used to treat seizures from other causes such as head trauma, drugs, and infections.

After change-of-shift report, which patient should the nurse assess first? a. Patient with myasthenia gravis who is reporting increased muscle weakness b. Patient with a bilateral headache described as "like a band around my head" c. Patient with seizures who is scheduled to receive a dose of phenytoin (Dilantin) d. Patient with Parkinson's disease who has developed cogwheel rigidity of the arms

A. Because increased muscle weakness may indicate the onset of a myasthenic crisis, the nurse should assess this patient first. The other patients should also be assessed, but do not appear to need immediate nursing assessments or actions to prevent life-threatening complications.

Which measure should the nurse prioritize when providing care for a patient with a diagnosis of multiple sclerosis (MS)? A. Vigilant infection control and adherence to standard precautions B. Careful monitoring of neurologic assessment and frequent reorientation C. Maintenance of a calorie count and hourly assessment of intake and output D. Assessment of blood pressure and monitoring for signs of orthostatic hypotension

A. Infection control is a priority in the care of patients with MS, since infection is the most common cause of an exacerbation of the disease. Decreases in cognitive function are less likely, and MS does not typically result in malnutrition, hypotension, or fluid volume excess or deficit.

The nurse advises a patient with myasthenia gravis (MG) to a. perform physically demanding activities early in the day. b. anticipate the need for weekly plasmapheresis treatments. c. do frequent weight-bearing exercise to prevent muscle atrophy. d. protect the extremities from injury due to poor sensory perception.

A. Muscles are generally strongest in the morning, and activities involving muscle activity should be scheduled then. Plasmapheresis is not routinely scheduled, but is used for myasthenia crisis or for situations in which corticosteroid therapy must be avoided. There is no decrease in sensation with MG, and muscle atrophy does not occur because although there is muscle weakness, they are still used.

A patient has been taking phenytoin (Dilantin) for 2 years. Which action will the nurse take when evaluating for adverse effects of the medication? a. Inspect the oral mucosa. b. Listen to the lung sounds. c. Auscultate the bowel tones. d. Check pupil reaction to light.

A. Phenytoin can cause gingival hyperplasia, but does not affect bowel tones, lung sounds, or pupil reaction to light.

A patient reports feeling numbness and tingling of the left arm before experiencing a tonic-clonic seizure. The nurse determines that this history is consistent with what type of seizure? a. Focal b. Atonic c. Absence d. Myoclonic

A. The initial symptoms of a focal seizure involve clinical manifestations that are localized to a particular part of the body or brain. Symptoms of an absence seizure are staring and a brief loss of consciousness. In an atonic seizure, the patient loses muscle tone and (typically) falls to the ground. Myoclonic seizures are characterized by a sudden jerk of the body or extremities.

The nurse observes a patient ambulating in the hospital hall when the patient's arms and legs suddenly jerk and the patient falls to the floor. The nurse will first a. assess the patient for a possible head injury. b. give the scheduled dose of divalproex (Depakote). c. document the timing and description of the seizure. d. notify the patient's health care provider about the seizure.

A. The patient who has had a myoclonic seizure and fall is at risk for head injury and should first be evaluated and treated for this possible complication. Documentation of the seizure, notification of the seizure, and administration of antiseizure medications are also appropriate actions, but the initial action should be assessment for injury.

A 54-year-old patient with acute osteomyelitis asks the nurse how this problem will be treated. Which response by the nurse is most appropriate? A. "IV antibiotics are usually required for several weeks." B. "Oral antibiotics are often required for several months." C. "Surgery is almost always necessary to remove the dead tissue that is likely to be present." D. "Drainage of the foot and instillation of antibiotics into the affected area is the usual therapy."

A. "IV antibiotics are usually required for several weeks." The standard treatment for acute osteomyelitis consists of several weeks of IV antibiotic therapy. This is because bone is denser and less vascular than other tissues, and it takes time for the antibiotic therapy to eradicate all of the microorganisms. Surgery may be used for chronic osteomyelitis, which may include debridement of the devitalized and infected tissue and irrigation of the affected bone with antibiotics

A 64-year-old patient who has amyotrophic lateral sclerosis (ALS) is hospitalized with pneumonia. Which nursing action will be included in the plan of care? a. Assist with active range of motion (ROM). b. Observe for agitation and paranoia. c. Give muscle relaxants as needed to reduce spasms. d. Use simple words and phrases to explain procedures.

A. ALS causes progressive muscle weakness, but assisting the patient to perform active ROM will help maintain strength as long as possible. Psychotic manifestations such as agitation and paranoia are not associated with ALS. Cognitive function is not affected by ALS, and the patient's ability to understand procedures will not be impaired. Muscle relaxants will further increase muscle weakness and depress respirations.

A hospitalized 31-year-old patient with a history of cluster headache awakens during the night with a severe stabbing headache. Which action should the nurse take first? a. Start the ordered PRN oxygen at 6 L/min. b. Put a moist hot pack on the patient's neck. c. Give the ordered PRN acetaminophen (Tylenol). d. Notify the patient's health care provider immediately.

A. Acute treatment for cluster headache is administration of 100% oxygen at 6 to 8 L/min. If the patient obtains relief with the oxygen, there is no immediate need to notify the health care provider. Cluster headaches last only 60 to 90 minutes, so oral pain medications have minimal effect. Hot packs are helpful for tension headaches but are not as likely to reduce pain associated with a cluster headache.

A patient has ICP monitoring with an intraventricular catheter. A priority nursing intervention for the patient is a. aseptic technique to prevent infection b. constant monitoring of ICP waveforms c. removal of CSF to maintain normal ICP d. sampling CSF to determine abnormalities

A. Aseptic technique to prevent infection- An intraventricular catheter is a fluid coupled system that can provide direct access for microorganisms to enter the ventricles of the brain, and aseptic technique is a very high nursing priority to decrease the risk for infection. Constant monitoring of ICP waveforms is not usually necessary, and removal of CSF for sampling or to maintain normal ICP is done only when specifically ordered

A patient has a nursing diagnosis of risk for ineffective cerebral tissue perfusion related to cerebral edema. An appropriate nursing intervention for the patient is a. avoiding positioning the patient with neck and hip flexion b. maintaining hyperventilation to a PaCO2 of 15 to 20 mm Hg c. clustering nursing activities to provide periods of uninterrupted rest d. routine suctioning to prevent accumulation of respiratory secretions

A. Avoiding positioning the patient with neck and hip flexion- Nursing care activities that increase ICP include hip and neck flexion, suctioning, clustering care activities, and noxious stimuli; they should be avoided or performed as little as possible in the patient with increased ICP. Lowering the PaCO2 below 20 mm Hg can cause ischemia and worsening of ICP; the PaCO2 should be maintained at 30 to 35 mm Hg.

After change-of-shift report, which patient should the nurse assess first? a. Patient with myasthenia gravis who is reporting increased muscle weakness b. Patient with a bilateral headache described as "like a band around my head" c. Patient with seizures who is scheduled to receive a dose of phenytoin (Dilantin) d. Patient with Parkinson's disease who has developed cogwheel rigidity of the arms

A. Because increased muscle weakness may indicate the onset of a myasthenic crisis, the nurse should assess this patient first. The other patients should also be assessed, but do not appear to need immediate nursing assessments or actions to prevent life-threatening complications.

Musculoskeletal assessment is an important component of care for patients on what type of long-term therapy? A. Corticosteroids B. β-Adrenergic blockers C. Antiplatelet aggregators D. Calcium-channel blockers

A. Corticosteroids Corticosteroids are associated with avascular necrosis and decreased bone and muscle mass. β-blockers, calcium-channel blockers, and antiplatelet aggregators are not commonly associated with damage to the musculoskeletal system.

For the patient undergoing a craniotomy, the nurse provides information about the use of wigs and hairpieces or other methods to disguise hair loss a. during pre operative teaching b. in the patient asks about their use c. in the immediate postoperative period d. when the patient expresses negative feelings about his or her appearance

A. During pre operative teaching- The prevent undue concern and anxiety about hair loss and postoperative self-esteem disturbances, a patient undergoing cranial surgery should be informed pre operatively that the head is usually shaved in surgery while the patient is anesthetized and that methods can be used after the dressings are removed postoperatively to disguise the hair loss. In the immediate postoperative period, the patient is very ill, and the focus is on maintaining neurologic function, bur preoperatively the nurse should anticipate the patient's postoperative need for self-esteem and maintenance of appearance.

A 31-year-old woman who has multiple sclerosis (MS) asks the nurse about risks associated with pregnancy. Which response by the nurse is accurate? a. "MS symptoms may be worse after the pregnancy." b. "Women with MS frequently have premature labor." c. "MS is associated with an increased risk for congenital defects." d. "Symptoms of MS are likely to become worse during pregnancy."

A. During the postpartum period, women with MS are at greater risk for exacerbation of symptoms. There is no increased risk for congenital defects in infants born of mothers with MS. Symptoms of MS may improve during pregnancy. Onset of labor is not affected by MS.

A patient is suspected of having a cranial tumor. The signs and symptoms include memory deficits, visual disturbances, weakness of right upper and lower extremities, and personality changes. The nurse recognizes that the tumor is most likely located in the a. frontal lobe b. parietal lobe c. occipital lobe d. temporal lobe

A. Frontal lobe

When a patient is admitted to the emergency department following a head injury, the nurse's first priority in management of the patient once a patent airway is confirmed is a. maintaining cervical spine precautions b. determining the presence of increased ICP c. monitoring for changes in neurologic status d. establishing IV access with a large-bore catheter

A. In addition to monitoring for a patent airway during emergency care of the patient with a head injury, the nurse must always assume that a patient with a head injury may have a cervical spine injury. Maintaining cervical spine precautions in all assessment and treatment activities with the patient is essential to prevent additional neurologic damage.

The nurse advises a patient with myasthenia gravis (MG) to a. perform physically demanding activities early in the day. b. anticipate the need for weekly plasmapheresis treatments. c. do frequent weight-bearing exercise to prevent muscle atrophy. d. protect the extremities from injury due to poor sensory perception.

A. Muscles are generally strongest in the morning, and activities involving muscle activity should be scheduled then. Plasmapheresis is not routinely scheduled, but is used for myasthenia crisis or for situations in which corticosteroid therapy must be avoided. There is no decrease in sensation with MG, and muscle atrophy does not occur because although there is muscle weakness, they are still used.

The nurse is performing a musculoskeletal assessment of an 81-year-old female patient whose mobility has been progressively decreasing in recent months. How should the nurse best assess the patient's range of motion (ROM) in the affected leg? A. Observe the patient's unassisted ROM in the affected leg. B. Perform passive ROM, asking the patient to report any pain. C. Ask the patient to lift progressive weights with the affected leg. D. Move both of the patient's legs from a supine position to full flexion.

A. Observe the patient's unassisted ROM in the affected leg. Passive ROM should be performed with extreme caution and may be best avoided when assessing older patients. Observing the patient's active ROM is more accurate and safe than asking the patient to lift weights with her legs.

During admission of a patient with a severe head injury to the ED, the nurse places highest priority on assessment for a. patency of of airway b. presence of a neck injury c. neurologic status with Glascow Coma Scale d. CSF leakage from ears and nose

A. Patency of airway is the #1 priority with all head injuries

A patient has a new order for magnetic resonance imaging (MRI) to evaluate for left femur osteomyelitis after a hip replacement surgery. Which information indicates that the nurse should consult with the health care provider before scheduling the MRI? a. The patient has a pacemaker. b. The patient is claustrophobic. c. The patient wears a hearing aid. d. The patient is allergic to shellfish.

A. Patients with permanent pacemakers cannot have MRI because of the force exerted by the magnetic field on metal objects. An open MRI will not cause claustrophobia. The patient will need to be instructed to remove the hearing aid before the MRI, but this does not require consultation with the health care provider. Because contrast medium will not be used, shellfish allergy is not a contraindication to MRI.

A patient has been taking phenytoin (Dilantin) for 2 years. Which action will the nurse take when evaluating for adverse effects of the medication? a. Inspect the oral mucosa. b. Listen to the lung sounds. c. Auscultate the bowel tones. d. Check pupil reaction to light.

A. Phenytoin can cause gingival hyperplasia, but does not affect bowel tones, lung sounds, or pupil reaction to light.

Which nursing intervention is most appropriate when turning a patient following spinal surgery? A. Placing a pillow between the patient's legs and turning the body as a unit B. Having the patient turn to the side by grasping the side rails to help turn over C. Elevating the head of bed 30 degrees and having the patient extend the legs while turning D. Turning the patient's head and shoulders and then the hips, keeping the patient's body centered in the bed

A. Placing a pillow between the patient's legs and turning the body as a unit Placing a pillow between the legs and turning the patient as a unit (logrolling) helps to keep the spine in good alignment and reduces pain and discomfort following spinal surgery. Having the patient turn by grasping the side rail to help, elevating the head of the bed, and turning with extended legs or turning the patient's head and shoulders and then the hips will not maintain proper spine alignment and may cause damage.

In which order will the nurse take these actions when caring for a patient with left leg fractures after a motor vehicle accident? Put a comma and space between each answer choice (a, b, c, d, etc.) ____________________ a. Obtain x-rays. b. Check pedal pulses. c. Assess lung sounds. d. Take blood pressure. e. Apply splint to the leg. f. Administer tetanus prophylaxis.

ANS: C, D, B, E, A, F The initial actions should be to ensure that airway, breathing, and circulation are intact. This should be followed by checking the neurovascular status of the leg (before and after splint application). Application of a splint to immobilize the leg should be done before sending the patient for x-rays. The tetanus prophylaxis is the least urgent of the actions.

A 46-year-old patient tells the nurse about using acetaminophen (Tylenol) several times every day for recurrent bilateral headaches. Which action will the nurse plan to take first? a. Discuss the need to stop taking the acetaminophen. b. Suggest the use of biofeedback for headache control. c. Describe the use of botulism toxin (Botox) for headaches. d. Teach the patient about magnetic resonance imaging (MRI).

A. The headache description suggests that the patient is experiencing medication overuse headache. The initial action will be withdrawal of the medication. The other actions may be needed if the headaches persist.

A patient reports feeling numbness and tingling of the left arm before experiencing a tonic-clonic seizure. The nurse determines that this history is consistent with what type of seizure? a. Focal b. Atonic c. Absence d. Myoclonic

A. The initial symptoms of a focal seizure involve clinical manifestations that are localized to a particular part of the body or brain. Symptoms of an absence seizure are staring and a brief loss of consciousness. In an atonic seizure, the patient loses muscle tone and (typically) falls to the ground. Myoclonic seizures are characterized by a sudden jerk of the body or extremities.

The nurse observes a patient ambulating in the hospital hall when the patient's arms and legs suddenly jerk and the patient falls to the floor. The nurse will first a. assess the patient for a possible head injury. b. give the scheduled dose of divalproex (Depakote). c. document the timing and description of the seizure. d. notify the patient's health care provider about the seizure.

A. The patient who has had a myoclonic seizure and fall is at risk for head injury and should first be evaluated and treated for this possible complication. Documentation of the seizure, notification of the seizure, and administration of antiseizure medications are also appropriate actions, but the initial action should be assessment for injury.

The nurse observes a patient ambulating in the hospital hall when the patient's arms and legs suddenly jerk and the patient falls to the floor. The nurse will first a. assess the patient for a possible injury. b. give the scheduled divalproex (Depakote). c. document the timing and description of the seizure. d. notify the patient's health care provider about the seizure.

ANS: A The patient who has had a myoclonic seizure and fall is at risk for head injury and should first be evaluated and treated for this possible complication. Documentation of the seizure, notification of the health care provider, and administration of antiseizure medications are also appropriate actions, but the initial action should be assessment for injury. DIF: Cognitive Level: Analyze (analysis) REF: 1376 OBJ: Special Questions: Prioritization TOP: Nursing Process: Implementation MSC: NCLEX: Physiological Integrity

A patient is admitted to the hospital with a left hemiplegia. To determine the size and location and to ascertain whether a stroke is ischemic or hemorrhagic, the nurse anticipates that the health care provider will request a a. CT scan b. lumbar puncture c. cerebral arteriogram d. positron emission tomography (PET)

A: CT scan- A CT scan is the most commonly used diagnostic test to determine the size and location of the lesion and to differentiate a thrombotic stroke from a hemorrhagic stroke. Positron emission tomography (PET) will show the metabolic activity of the brain and provide a depiction of the extent of tissue damage after a stroke. Lumbar punctures are not performed routinely because of the chance of increased intracranial pressure causing herniation. Cerebral arteriograms are invasive and may dislodge an embolism or cause further hemorrhage; they are performed only when no other test can provide the needed information.

During the acute phase of a stroke, the nurse assesses the patient's vital signs and neurologic status every 4 hours. A cardiovascular sign that the nurse would see as the body attempts to increase cerebral blood flow is a. hypertension b. fluid overload c. cardiac dysrhythmias d. S3 and S4 heart sounds

A: Hypertension- The body responds to the vasopasm and a decreased circulation to the brain that occurs with a stroke by increasing the BP, frequently resulting in hypertension. The other options are important cardiovascular factors to assess, but they do not result from impaired cerebral blood flow.

A newly admitted patient who has suffered a right sided brain stroke has a nursing diagnosis of disturbed visual sensory perception related to homonymous hemianopsia. Early in the care of the patient, the nurse should a. place objects on the right side within the patient's field of vision b. approach the patient from the left side to encourage the patient to turn the head c. place objects on the patient's left side to assess the patient's ability to compensate d. patch the affected eye to encourage the patient to turn the head to scan the environment

A: Place objects on the right side within the patient's field of vision- the presence of homonymous hemianopia in a patient with right-hemisphere brain damage causes a loss of vision in the left field. Early in the care of the patient, objects should be placed on the right side of the patient in the field of vision, and the nurse should approach the patient from the right side. Later in treatment, patients should be taught to turn the head and scan the environment and should be approached from the affected side to encourage head turning. Eye patches are used if patients have diplopia (double vision).

Four days following a stroke, a patient is to start oral fluids and feedings. Before feeding the patient, the nurse should first a. check the patient's gag reflex b. order a soft diet for the patient c. raise the head of the bed to sitting position d. evaluate the patient's ability to swallow small sips of ice water

A: check the patient's gag reflex- the first step in providing oral feedings for a patient with a stroke is ensuring that the patient has an intact gag reflex because oral feedings will not be provided if gag reflex is impaired. The nurse should then evaluate the patient's ability to swallow ice chips or ice water after placing the patient in an upright position

onset of meningitis is:

ABRUPT

A patient who is having an acute exacerbation of multiple sclerosis has a prescription for methylprednisolone (Solu-Medrol) 150 mg IV. The label on the vial reads: methylprednisolone 125 mg in 2 mL. How many mL will the nurse administer?

ANS: 2.4 With a concentration of 125 mg/2 mL, the nurse will need to administer 2.4 mL to obtain 150 mg of methylprednisolone. DIF: Cognitive Level: Understand (comprehension) REF: 1386 TOP: Nursing Process: Implementation MSC: NCLEX: Physiological Integrity

An unconscious patient with a traumatic head injury has a blood pressure of 126/72 mm Hg, and an intracranial pressure of 18 mm Hg. The nurse will calculate the cerebral perfusion pressure as ____________________.

ANS: 72 mm Hg The formula for calculation of cerebral perfusion pressure is [(Systolic pressure + Diastolic blood pressure 2)/3] = intracranial pressure. DIF: Cognitive Level: Application REF: 1452-1453

A patient is brought to the emergency department (ED) by ambulance after being found unconscious on the bathroom floor by the spouse. Which action will the nurse take first? a. Obtain oxygen saturation. b. Check pupil reaction to light. c. Palpate the head for hematoma. d. Assess Glasgow Coma Scale (GCS).

ANS: A Airway patency and breathing are the most vital functions and should be assessed first. The neurologic assessments should be accomplished next and the health and medication history last. DIF: Cognitive Level: Application REF: 1435-1437

Which of these patients is most appropriate for the intensive care unit (ICU) charge nurse to assign to an RN who has floated from the medical unit? a. A 44-year-old receiving IV antibiotics for meningococcal meningitis b. A 23-year-old who had a skull fracture and craniotomy the previous day c. A 30-year-old who has an intracranial pressure (ICP) monitor in place after a head injury a week ago d. A 61-year-old who has increased ICP and is receiving hyperventilation therapy

ANS: A An RN who works on a medical unit will be familiar with administration of IV antibiotics and with meningitis. The postcraniotomy patient, patient with an ICP monitor, and the patient on a ventilator should be assigned to an RN familiar with the care of critically ill patients. DIF: Cognitive Level: Application REF: 1435-1438

A 45-year-old patient has a dysfunction of the cerebellum. The nurse will plan interventions to a. prevent falls. b. stabilize mood. c. avoid aspiration. d. improve memory.

ANS: A Because functions of the cerebellum include coordination and balance, the patient with dysfunction is at risk for falls. The cerebellum does not affect memory, mood, or swallowing ability.

A patient has a tumor in the cerebellum. The nurse will plan interventions to a. prevent falls. c. avoid aspiration. b. stabilize mood. d. improve memory.

ANS: A Because functions of the cerebellum include coordination and balance, the patient with dysfunction is at risk for falls. The cerebellum does not affect memory, mood, or swallowing ability. DIF: Cognitive Level: Apply (application) REF: 1298 TOP: Nursing Process: Planning MSC: NCLEX: Physiological Integrity

After change-of-shift report, which patient should the nurse assess first? a. Patient with myasthenia gravis who is reporting increased muscle weakness b. Patient with a bilateral headache described as "like a band around my head" c. Patient with seizures who is scheduled to receive a dose of phenytoin (Dilantin) d. Patient with Parkinson's disease who has developed cogwheel rigidity of the arms

ANS: A Because increased muscle weakness may indicate the onset of a myasthenic crisis, the nurse should assess this patient first. The other patients should also be assessed but do not appear to need immediate nursing assessments or actions to prevent life-threatening complications. DIF: Cognitive Level: Analyze (analysis) REF: 1393 OBJ: Special Questions: Prioritization | Special Questions: Multiple Patients TOP: Nursing Process: Planning MSC: NCLEX: Safe and Effective Care Environment

After change-of-shift report, which patient should the nurse assess first? a. Patient with myasthenia gravis who is reporting increased muscle weakness b. Patient with a bilateral headache described as "like a band around my head" c. Patient with seizures who is scheduled to receive a dose of phenytoin (Dilantin) d. Patient with Parkinson's disease who has developed cogwheel rigidity of the arms

ANS: A Because increased muscle weakness may indicate the onset of a myasthenic crisis, the nurse should assess this patient first. The other patients should also be assessed, but do not appear to need immediate nursing assessments or actions to prevent life-threatening complications. DIF: Cognitive Level: Analyze (analysis) REF: 1438-1439 OBJ: Special Questions: Prioritization; Multiple Patients TOP: Nursing Process: Planning MSC: NCLEX: Safe and Effective Care Environment

A woman who has multiple sclerosis (MS) asks the nurse about risks associated with pregnancy. Which response by the nurse is accurate? a. "MS symptoms may be worse after the pregnancy." b. "Women with MS frequently have premature labor." c. "MS is associated with an increased risk for congenital defects." d. "Symptoms of MS are likely to become worse during pregnancy."

ANS: A During the postpartum period, women with MS are at greater risk for exacerbation of symptoms. There is no increased risk for congenital defects in infants born of mothers with MS. Symptoms of MS may improve during pregnancy. Onset of labor is not affected by MS. DIF: Cognitive Level: Understand (comprehension) REF: 1384 TOP: Nursing Process: Implementation MSC: NCLEX: Health Promotion and Maintenance

When a patient's intracranial pressure (ICP) is being monitored with an intraventricular catheter, which information obtained by the nurse is most important to communicate to the health care provider? a. Oral temperature 101.6° F b. Apical pulse 102 beats/min c. Intracranial pressure 15 mm Hg d. Mean arterial pressure 90 mm Hg

ANS: A Infection is a serious consideration with ICP monitoring, especially with intraventricular catheters. The temperature indicates the need for antibiotics or removal of the monitor. The ICP, arterial pressure, and apical pulse are all borderline high but require only ongoing monitoring at this time. DIF: Cognitive Level: Application REF: 1438-1440

The nurse advises a patient with myasthenia gravis (MG) to a. perform physically demanding activities early in the day. b. anticipate the need for weekly plasmapheresis treatments. c. do frequent weight-bearing exercise to prevent muscle atrophy. d. protect the extremities from injury due to poor sensory perception.

ANS: A Muscles are generally strongest in the morning, and activities involving muscle activity should be scheduled then. Plasmapheresis is not routinely scheduled but is used for myasthenia crisis or for situations in which corticosteroid therapy must be avoided. There is no decrease in sensation with MG, and muscle atrophy does not occur because although there is muscle weakness, they are still used. DIF: Cognitive Level: Apply (application) REF: 1393 TOP: Nursing Process: Implementation MSC: NCLEX: Physiological Integrity

he nurse advises a patient with myasthenia gravis (MG) to a. perform physically demanding activities early in the day. b. anticipate the need for weekly plasmapheresis treatments. c. do frequent weight-bearing exercise to prevent muscle atrophy. d. protect the extremities from injury due to poor sensory perception.

ANS: A Muscles are generally strongest in the morning, and activities involving muscle activity should be scheduled then. Plasmapheresis is not routinely scheduled, but is used for myasthenia crisis or for situations in which corticosteroid therapy must be avoided. There is no decrease in sensation with MG, and muscle atrophy does not occur because although there is muscle weakness, they are still used. DIF: Cognitive Level: Apply (application) REF: 1437 TOP: Nursing Process: Implementation MSC: NCLEX: Physiological Integrity

A patient with ulnar drift caused by rheumatoid arthritis (RA) is scheduled for an arthroplasty of the hand. Which patient statement to the nurse indicates realistic expectation for the surgery? a. "I will be able to use my fingers to grasp objects better." b. "I will not have to do as many hand exercises after the surgery." c. "This procedure will prevent further deformity in my hands and fingers." d. "My fingers will appear more normal in size and shape after this surgery."

ANS: A The goal of hand surgery in RA is to restore function, not to correct for cosmetic deformity or treat the underlying process. Hand exercises will be prescribed after the surgery.

A patient who has bacterial meningitis is disoriented and anxious. Which nursing action will be included in the plan of care? a. Encourage family members to remain at the bedside. b. Apply soft restraints to protect the patient from injury. c. Keep the room well-lighted to improve patient orientation. d. Minimize contact with the patient to decrease sensory input.

ANS: A Patients with meningitis and disorientation will be calmed by the presence of someone familiar at the bedside. Restraints should be avoided because they increase agitation and anxiety. The patient requires frequent assessment for complications; the use of touch and a soothing voice will decrease anxiety for most patients. The patient will have photophobia, so the light should be dim. DIF: Cognitive Level: Application REF: 1453-1455

A patient has been taking phenytoin (Dilantin) for 2 years. Which action will the nurse take when evaluating for adverse effects of the medication? a. Inspect the oral mucosa. c. Auscultate the bowel sounds. b. Listen to the lung sounds. d. Check pupil reaction to light.

ANS: A Phenytoin can cause gingival hyperplasia, but does not affect bowel sounds, lung sounds, or pupil reaction to light. DIF: Cognitive Level: Apply (application) REF: 1379 TOP: Nursing Process: Evaluation MSC: NCLEX: Physiological Integrity

A patient has been taking phenytoin (Dilantin) for 2 years. Which action will the nurse take when evaluating for adverse effects of the medication? a. Inspect the oral mucosa. b. Listen to the lung sounds. c. Auscultate the bowel tones. d. Check pupil reaction to light.

ANS: A Phenytoin can cause gingival hyperplasia, but does not affect bowel tones, lung sounds, or pupil reaction to light. DIF: Cognitive Level: Apply (application) REF: 1424 TOP: Nursing Process: Evaluation MSC: NCLEX: Physiological Integrity

A patient with lower leg fracture has an external fixation device in place and is scheduled for discharge. Which information will the nurse include in the discharge teaching? a. "You will need to assess and clean the pin insertion sites daily." b. "The external fixator can be removed during the bath or shower." c. "You will need to remain on bed rest until bone healing is complete." d. "Prophylactic antibiotics are used until the external fixator is removed."

ANS: A Pin insertion sites should be cleaned daily to decrease the risk for infection at the site. An external fixator allows the patient to be out of bed and avoid the risks of prolonged immobility. The device is surgically placed and is not removed until the bone is stable. Prophylactic antibiotics are not routinely given when an external fixator is used.

While admitting a patient with a basal skull fracture, the nurse notes clear drainage from the patient's nose. Which of these admission orders should the nurse question? a. Insert nasogastric tube. b. Turn patient every 2 hours. c. Keep the head of bed elevated. d. Apply cold packs for facial bruising.

ANS: A Rhinorrhea may indicate a dural tear with cerebrospinal fluid (CSF) leakage, and insertion of a nasogastric tube will increase the risk for infections such as meningitis. Turning the patient, elevating the head, and applying cold pack are appropriate orders. DIF: Cognitive Level: Application REF: 1440

The charge nurse observes an inexperienced staff nurse who is caring for a patient who has had a craniotomy for a brain tumor. Which action by the inexperienced nurse requires the charge nurse to intervene? a. The staff nurse suctions the patient every 2 hours. b. The staff nurse assesses neurologic status every hour. c. The staff nurse elevates the head of the bed to 30 degrees. d. The staff nurse administers a mild analgesic before turning the patient.

ANS: A Suctioning increases intracranial pressure and is done only when the patient's respiratory condition indicates it is needed. The other actions by the staff nurse are appropriate. DIF: Cognitive Level: Application REF: 1430-1431

A patient with a head injury has admission vital signs of blood pressure 128/68, pulse 110, and respirations 26. Which of these vital signs, if taken 1 hour after admission, will be of most concern to the nurse? a. Blood pressure 156/60, pulse 55, respirations 12 b. Blood pressure 130/72, pulse 90, respirations 32 c. Blood pressure 148/78, pulse 112, respirations 28 d. Blood pressure 110/70, pulse 120, respirations 30

ANS: A Systolic hypertension with widening pulse pressure, bradycardia, and respiratory changes represent Cushing's triad and indicate that the intracranial pressure (ICP) has increased, and brain herniation may be imminent unless immediate action is taken to reduce ICP. The other vital signs may indicate the need for changes in treatment, but they are not indicative of an immediately life-threatening process. DIF: Cognitive Level: Application REF: 1429-1430

When caring for a patient who has had a head injury, which assessment information requires the most rapid action by the nurse? a. The patient is more difficult to arouse. b. The patient's pulse is slightly irregular. c. The patient's blood pressure increases from 120/54 to 136/62 mm Hg. d. The patient complains of a headache at pain level 5 of a 10-point scale.

ANS: A The change in level of consciousness (LOC) is an indicator of increased intracranial pressure (ICP) and suggests that action by the nurse is needed to prevent complications. The change in BP should be monitored but is not an indicator of a need for immediate nursing action. Headache is not unusual in a patient after a head injury. A slightly irregular apical pulse is not unusual. DIF: Cognitive Level: Application REF: 1431-1433

When admitting a patient who has a tumor of the right frontal lobe, the nurse would expect to find a. judgment changes. b. expressive aphasia. c. right-sided weakness. d. difficulty swallowing.

ANS: A The frontal lobes control intellectual activities such as judgment. Speech is controlled in the parietal lobe. Weakness and hemiplegia occur on the contralateral side from the tumor. Swallowing is controlled by the brainstem. DIF: Cognitive Level: Application REF: 1447 | 1448

Which action will the nurse include in the plan of care for a patient with impaired functioning of the left glossopharyngeal nerve (CN IX) and the vagus nerve (CN X)? a. Withhold oral fluid or foods. b. Provide highly seasoned foods. c. Insert an oropharyngeal airway. d. Apply artificial tears every hour.

ANS: A The glossopharyngeal and vagus nerves innervate the pharynx and control the gag reflex. A patient with impaired function of these nerves is at risk for aspiration. An oral airway may be needed when a patient is unconscious and unable to maintain the airway, but it will not decrease aspiration risk. Taste and eye blink are controlled by the facial nerve

A patient tells the nurse about using acetaminophen (Tylenol) several times every day for recurrent bilateral headaches. Which action will the nurse plan to take first? a. Discuss the need to stop taking the acetaminophen. b. Suggest the use of biofeedback for headache control. c. Describe the use of botulism toxin (Botox) for headaches. d. Teach the patient about magnetic resonance imaging (MRI).

ANS: A The headache description suggests that the patient is experiencing medication overuse headache. The initial action will be withdrawal of the medication. The other actions may be needed if the headaches persist. DIF: Cognitive Level: Analyze (analysis) REF: 1372 OBJ: Special Questions: Prioritization TOP: Nursing Process: Planning MSC: NCLEX: Physiological Integrity

The nurse obtains these assessment findings for a patient who has a head injury. Which finding should be reported rapidly to the health care provider? a. Urine output of 800 mL in the last hour b. Intracranial pressure of 16 mm Hg when patient is turned c. Ventriculostomy drains 10 mL of cerebrospinal fluid per hour d. LICOX brain tissue oxygenation catheter shows PbtO2 of 38 mm Hg

ANS: A The high urine output indicates that diabetes insipidus may be developing and interventions to prevent dehydration need to be rapidly implemented. The other data do not indicate a need for any change in therapy. DIF: Cognitive Level: Application REF: 1434-1435

A patient reports feeling numbness and tingling of the left arm before experiencing a seizure. The nurse determines that this history is consistent with what type of seizure? a. Focal c. Absence b. Atonic d. Myoclonic

ANS: A The initial symptoms of a focal seizure involve clinical manifestations that are localized to a particular part of the body or brain. Symptoms of an absence seizure are staring and a brief loss of consciousness. In an atonic seizure, the patient loses muscle tone and (typically) falls to the ground. Myoclonic seizures are characterized by a sudden jerk of the body or extremities. DIF: Cognitive Level: Understand (comprehension) REF: 1376 TOP: Nursing Process: Assessment MSC: NCLEX: Physiological Integrity

A patient reports feeling numbness and tingling of the left arm before experiencing a tonic-clonic seizure. The nurse determines that this history is consistent with what type of seizure? a. Focal b. Atonic c. Absence d. Myoclonic

ANS: A The initial symptoms of a focal seizure involve clinical manifestations that are localized to a particular part of the body or brain. Symptoms of an absence seizure are staring and a brief loss of consciousness. In an atonic seizure, the patient loses muscle tone and (typically) falls to the ground. Myoclonic seizures are characterized by a sudden jerk of the body or extremities. DIF: Cognitive Level: Understand (comprehension) REF: 1421 TOP: Nursing Process: Assessment MSC: NCLEX: Physiological Integrity

When doing discharge teaching for a patient who has had a repair of a fractured mandible, the nurse will include information about a. when and how to cut the immobilizing wires. b. self-administration of nasogastric tube feedings. c. the use of sterile technique for dressing changes. d. the importance of including high-fiber foods in the diet.

ANS: A The jaw will be wired for stabilization, and the patient should know what emergency situations require that the wires be cut to protect the airway. There are no dressing changes for this procedure. The diet is liquid, and patients are not able to chew high fiber foods. Initially, the patient may receive nasogastric tube feedings, but by discharge the patient will swallow liquid through a straw.

Before assisting a patient with ambulation on the day after a total hip replacement, which action is most important for the nurse to take? a. Administer the ordered oral opioid pain medication. b. Instruct the patient about the benefits of ambulation. c. Ensure that the incisional drain has been discontinued. d. Change the hip dressing and document the wound appearance.

ANS: A The patient should be adequately medicated for pain before any attempt to ambulate. Instructions about the benefits of ambulation may increase the patient's willingness to ambulate, but decreasing pain with ambulation is more important. The presence of an incisional drain or timing of dressing change will not affect ambulation.

The nurse observes a patient ambulating in the hospital hall when the patient's arms and legs suddenly jerk and the patient falls to the floor. The nurse will first a. assess the patient for a possible head injury. b. give the scheduled dose of divalproex (Depakote). c. document the timing and description of the seizure. d. notify the patient's health care provider about the seizure.

ANS: A The patient who has had a myoclonic seizure and fall is at risk for head injury and should first be evaluated and treated for this possible complication. Documentation of the seizure, notification of the seizure, and administration of antiseizure medications are also appropriate actions, but the initial action should be assessment for injury. DIF: Cognitive Level: Apply (application) REF: 1423 OBJ: Special Questions: Prioritization TOP: Nursing Process: Implementation MSC: NCLEX: Physiological Integrity

An unconscious patient has a nursing diagnosis of ineffective cerebral tissue perfusion related to cerebral tissue swelling. Which nursing intervention will be included in the plan of care? a. Keep the head of the bed elevated to 30 degrees. b. Position the patient with the knees and hips flexed. c. Encourage coughing and deep breathing to improve oxygenation. d. Cluster nursing interventions to provide uninterrupted rest periods.

ANS: A The patient with increased intracranial pressure (ICP) should be maintained in the head-up position to help reduce ICP. Flexion of the hips and knees increases abdominal pressure, which increases ICP. Because the stimulation associated with nursing interventions increases ICP, clustering interventions will progressively elevate ICP. Coughing increases intrathoracic pressure and ICP. DIF: Cognitive Level: Application REF: 1436-1437

A patient who has had an open reduction and internal fixation (ORIF) of left lower leg fractures complains of constant severe pain in the leg, which is unrelieved by the prescribed morphine. Pulses are faintly palpable and the foot is cool. Which action should the nurse take next? a. Notify the health care provider. b. Assess the incision for redness. c. Reposition the left leg on pillows. d. Check the patient's blood pressure.

ANS: A The patient's clinical manifestations suggest compartment syndrome and delay in diagnosis and treatment may lead to severe functional impairment. The data do not suggest problems with blood pressure or infection. Elevation of the leg will decrease arterial flow and further reduce perfusion.

Intervention for Chronic phase of Stroke

Airway Fluid therapy Treatment of cerebral edema Prevention of secondary injury

To assess the functioning of the trigeminal and facial nerves (CNs V and VII), the nurse should a. shine a light into the patient's pupil. b. check for unilateral eyelid drooping. c. touch a cotton wisp strand to the cornea. d. have the patient read a magazine or book.

ANS: A The trigeminal and facial nerves are responsible for the corneal reflex. The optic nerve is tested by having the patient read a Snellen chart or a newspaper. Assessment of pupil response to light and ptosis are used to check function of the oculomotor nerve

When admitting a patient with a possible coup-contracoup injury after a car accident to the emergency department, the nurse obtains the following information. Which finding is most important to report to the health care provider? a. The patient takes warfarin (Coumadin) daily. b. The patient's blood pressure is 162/94 mm Hg. c. The patient is unable to remember the accident. d. The patient complains of a severe dull headache.

ANS: A The use of anticoagulants increases the risk for intracranial hemorrhage and should be immediately reported. The other information would not be unusual in a patient with a head injury who had just arrived to the ED. DIF: Cognitive Level: Application REF: 1437-1438

When admitting an acutely confused 20-year-old patient with a head injury, which action should the nurse take? a. Ask family members about the patient's health history. b. Ask leading questions to assist in obtaining health data. c. Wait until the patient is better oriented to ask questions. d. Obtain only the physiologic neurologic assessment data.

ANS: A When admitting a patient who is likely to be a poor historian, the nurse should obtain health history information from others who have knowledge about the patient's health. Waiting until the patient is oriented or obtaining only physiologic data will result in incomplete assessment data, which could adversely affect decision making about treatment. Asking leading questions may result in inaccurate or incomplete information

When admitting an acutely confused patient with a head injury, which action should the nurse take? a. Ask family members about the patient's health history. b. Ask leading questions to assist in obtaining health data. c. Wait until the patient is better oriented to ask questions. d. Obtain only the physiologic neurologic assessment data.

ANS: A When admitting a patient who is likely to be a poor historian, the nurse should obtain health history information from others who have knowledge about the patient's health. Waiting until the patient is oriented or obtaining only physiologic data will result in incomplete assessment data, which could adversely affect decision making about treatment. Asking leading questions may result in inaccurate or incomplete information. DIF: Cognitive Level: Apply (application) REF: 1301 TOP: Nursing Process: Assessment MSC: NCLEX: Physiological Integrity

A patient with Parkinson's disease is admitted to the hospital for treatment of pneumonia. Which nursing interventions will be included in the plan of care (select all that apply)? a. Provide an elevated toilet seat. b. Cut patient's food into small pieces. c. Serve high-protein foods at each meal. d. Place an armchair at the patient's bedside. e. Observe for sudden exacerbation of symptoms.

ANS: A, B, D Because the patient with Parkinson's disease has difficulty chewing, food should be cut into small pieces. An armchair should be used when the patient is seated so that the patient can use the arms to assist with getting up from the chair. An elevated toilet seat will facilitate getting on and off the toilet. High-protein foods will decrease the effectiveness of L-dopa. Parkinson's disease is a steadily progressive disease without acute exacerbations. DIF: Cognitive Level: Apply (application) REF: 1391 TOP: Nursing Process: Planning MSC: NCLEX: Physiological Integrity

A patient with Parkinson's disease is admitted to the hospital for treatment of pneumonia. Which nursing interventions will be included in the plan of care (select all that apply)? a. Use an elevated toilet seat. b. Cut patient's food into small pieces. c. Provide high-protein foods at each meal. d. Place an armchair at the patient's bedside. e. Observe for sudden exacerbation of symptoms.

ANS: A, B, D Because the patient with Parkinson's has difficulty chewing, food should be cut into small pieces. An armchair should be used when the patient is seated so that the patient can use the arms to assist with getting up from the chair. An elevated toilet seat will facilitate getting on and off the toilet. High-protein foods will decrease the effectiveness of L-dopa. Parkinson's is a steadily progressive disease without acute exacerbations. DIF: Cognitive Level: Apply (application) REF: 1436-1437 TOP: Nursing Process: Planning MSC: NCLEX: Physiological Integrity

A patient who has been treated for status epilepticus in the emergency department will be transferred to the medical nursing unit. Which equipment should the nurse have available in the patient's assigned room (select all that apply)? a. Side-rail pads d. Suction tubing b. Tongue blade e. Urinary catheter c. Oxygen mask f. Nasogastric tube

ANS: A, C, D The patient is at risk for further seizures, and O2 and suctioning may be needed after any seizures to clear the airway and maximize oxygenation. The bed's side rails should be padded to minimize the risk for patient injury during a seizure. Use of tongue blades during a seizure is contraindicated. Insertion of a nasogastric (NG) tube is not indicated because the airway problem is not caused by vomiting or abdominal distention. A urinary catheter is not required unless there is urinary retention. DIF: Cognitive Level: Apply (application) REF: 1381 TOP: Nursing Process: Planning MSC: NCLEX: Physiological Integrity

A 27-year-old patient who has been treated for status epilepticus in the emergency department will be transferred to the medical nursing unit. Which equipment should the nurse have available in the patient's assigned room (select all that apply)? a. Side-rail pads b. Tongue blade c. Oxygen mask d. Suction tubing e. Urinary catheter f. Nasogastric tube

ANS: A, C, D The patient is at risk for further seizures, and oxygen and suctioning may be needed after any seizures to clear the airway and maximize oxygenation. The bed's side rails should be padded to minimize the risk for patient injury during a seizure. Use of tongue blades during a seizure is contraindicated. Insertion of a nasogastric (NG) tube is not indicated because the airway problem is not caused by vomiting or abdominal distention. A urinary catheter is not required unless there is urinary retention .DIF: Cognitive Level: Apply (application) REF: 1426 TOP: Nursing Process: Planning MSC: NCLEX: Physiological Integrity

Which assessment information will the nurse collect to determine whether a patient is developing postconcussion syndrome? a. Muscle resistance b. Short-term memory c. Glasgow coma scale d. Pupil reaction to light

ANS: B Decreased short-term memory is one indication of postconcussion syndrome. The other data may be assessed but are not indications of postconcussion syndrome. DIF: Cognitive Level: Application REF: 1440

A hospitalized patient complains of a bilateral headache (4/10 on the pain scale) that radiates from the base of the skull. Which prescribed PRN medications should the nurse administer initially? a. Lorazepam (Ativan) c. Morphine sulfate (MS Contin) b. Acetaminophen (Tylenol) d. Butalbital and aspirin (Fiorinal)

ANS: B The patient's symptoms are consistent with a tension headache, and initial therapy usually involves a nonopioid analgesic such as acetaminophen, which is sometimes combined with a sedative or muscle relaxant. Lorazepam may be used in conjunction with acetaminophen but would not be appropriate as the initial monotherapy. Morphine sulfate and butalbital and aspirin would be more appropriate for a headache that did not respond to a nonopioid analgesic. DIF: Cognitive Level: Analyze (analysis) REF: 1372 OBJ: Special Questions: Prioritization TOP: Nursing Process: Implementation MSC: NCLEX: Physiological Integrity

When obtaining a health history and physical assessment for a 36-yr-old female patient with possible multiple sclerosis (MS), the nurse should a. assess for the presence of chest pain. b. inquire about urinary tract problems. c. inspect the skin for rashes or discoloration. d. ask the patient about any increase in libido.

ANS: B Urinary tract problems with incontinence or retention are common symptoms of MS. Chest pain and skin rashes are not symptoms of MS. A decrease in libido is common with MS. DIF: Cognitive Level: Apply (application) REF: 1384 TOP: Nursing Process: Assessment MSC: NCLEX: Physiological Integrity

A 39-yr-old patient with a suspected herniated intervertebral disc is scheduled for a myelogram. Which information communicated by the nurse to the health care provider before the procedure would change the procedural plans? a. The patient is anxious about the test results. b. The patient reports a previous allergy to shellfish. c. The patient has back pain when lying flat for more than 4 hours. d. The patient drank apple juice 4 hours before the scheduled procedure.

ANS: B A contrast medium containing iodine is injected into the subarachnoid space during a myelogram. The patient's allergy would contraindicate the use of this medium. The health care provider may need to modify the orders to prevent back pain, but this can be done after the procedure. Clear liquids are usually considered safe up to 4 hours before a diagnostic or surgical procedure. The patient's anxiety should be addressed, but procedural plans would not need to be changed. DIF: Cognitive Level: Apply (application) REF: 1310 TOP: Nursing Process: Assessment MSC: NCLEX: Physiological Integrity

A patient who has a head injury is diagnosed with a concussion. Which action will the nurse plan to take? a. Coordinate the transfer of the patient to the operating room. b. Provide discharge instructions about monitoring neurologic status. c. Transport the patient to radiology for magnetic resonance imaging (MRI) of the brain. d. Arrange to admit the patient to the neurologic unit for observation for 24 hours.

ANS: B A patient with a minor head trauma is usually discharged with instructions about neurologic monitoring and the need to return if neurologic status deteriorates. MRI, hospital admission, or surgery are not indicated in a patient with a concussion. DIF: Cognitive Level: Application REF: 1440

Which nursing diagnosis is expected to be appropriate for a patient who has a positive Romberg test? a. Acute pain b. Risk for falls c. Acute confusion d. Ineffective thermoregulation

ANS: B A positive Romberg test indicates that the patient has difficulty maintaining balance with the eyes closed. The Romberg does not test for orientation, thermoregulation, or discomfort

Which problem can the nurse expect for a patient who has a positive Romberg test result? a. Pain b. Falls c. Aphasia d. Confusion

ANS: B A positive Romberg test result indicates that the patient has difficulty maintaining balance when standing with the eyes closed. The Romberg does not test for orientation, thermoregulation, or discomfort. DIF: Cognitive Level: Apply (application) REF: 1307 TOP: Nursing Process: Planning MSC: NCLEX: Physiological Integrity

A patient is seen at the urgent care center after falling on the right arm and shoulder. Which finding is most important for the nurse to communicate to the health care provider? a. There is bruising at the shoulder area. b. The right arm appears shorter than the left. c. There is decreased range of motion of the shoulder. d. The patient is complaining of arm and shoulder pain.

ANS: B A shorter limb after a fall indicates a possible dislocation, which is an orthopedic emergency. Bruising, pain, and decreased range of motion also should be reported, but these do not indicate that emergent treatment is needed to preserve function.

A patient who has amyotrophic lateral sclerosis (ALS) is hospitalized with pneumonia. Which nursing action will be included in the plan of care? a. Observe for agitation and paranoia. b. Assist with active range of motion (ROM). c. Give muscle relaxants as needed to reduce spasms. d. Use simple words and phrases to explain procedures.

ANS: B ALS causes progressive muscle weakness, but assisting the patient to perform active ROM will help maintain strength as long as possible. Psychotic manifestations such as agitation and paranoia are not associated with ALS. Cognitive function is not affected by ALS, and the patient's ability to understand procedures will not be impaired. Muscle relaxants will further increase muscle weakness and depress respirations. DIF: Cognitive Level: Apply (application) REF: 1395 TOP: Nursing Process: Planning MSC: NCLEX: Physiological Integrity

A hospitalized patient with a history of cluster headache awakens during the night with a severe stabbing headache. Which action should the nurse take first? a. Put a moist hot pack on the patient's neck. b. Start the prescribed PRN O2 at 6 L/min. c. Give the ordered PRN acetaminophen (Tylenol). d. Notify the patient's health care provider immediately.

ANS: B Acute treatment for cluster headache is administration of 100% O2 at 6 to 8 L/min. If the patient obtains relief with the O2, there is no immediate need to notify the health care provider. Cluster headaches last only 60 to 90 minutes, so oral pain medications have minimal effect. Hot packs are helpful for tension headaches but are not as likely to reduce pain associated with a cluster headache. DIF: Cognitive Level: Analyze (analysis) REF: 1372 OBJ: Special Questions: Prioritization TOP: Nursing Process: Implementation MSC: NCLEX: Physiological Integrity

Which equipment will the nurse obtain to assess vibration sense in a patient with diabetes who has peripheral nerve dysfunction? a. Sharp pin b. Tuning fork c. Reflex hammer d. Calibrated compass

ANS: B Vibration sense is testing by touching the patient with a vibrating tuning fork. The other equipment is needed for testing of pain sensation, reflexes, and two-point discrimination. DIF: Cognitive Level: Understand (comprehension) REF: 1307 TOP: Nursing Process: Assessment MSC: NCLEX: Physiological Integrity

An unconscious male patient has just arrived in the emergency department with a head injury caused by a motorcycle crash. Which order should the nurse question? a. Obtain x-rays of the skull and spine. b. Prepare the patient for lumbar puncture. c. Send for computed tomography (CT) scan. d. Perform neurologic checks every 15 minutes.

ANS: B After a head injury, the patient may be experiencing intracranial bleeding and increased intracranial pressure, and herniation of the brain could result if lumbar puncture is performed. The other orders are appropriate. DIF: Cognitive Level: Apply (application) REF: 1310 TOP: Nursing Process: Implementation MSC: NCLEX: Physiological Integrity

An unconscious male patient has just arrived in the emergency department after a head injury caused by a motorcycle crash. Which order should the nurse question? a. Obtain x-rays of the skull and spine. b. Prepare the patient for lumbar puncture. c. Send for computed tomography (CT) scan. d. Perform neurologic checks every 15 minutes.

ANS: B After a head injury, the patient may be experiencing intracranial bleeding and increased intracranial pressure, which could lead to herniation of the brain if a lumbar puncture is performed. The other orders are appropriate

Which information about a 76-yr-old patient should the nurse report as uncharacteristic of normal aging? a. Triceps reflex response graded at 1/5 b. Unintended weight loss of 15 pounds c. 10 mm Hg orthostatic drop in systolic blood pressure d. Patient complaint of chronic difficulty in falling asleep

ANS: B Although changes in appetite are normal with aging, a 15-lb weight loss requires further investigation. Orthostatic drops in blood pressure, changes in sleep patterns, and slowing of reflexes are normal changes in aging. DIF: Cognitive Level: Apply (application) REF: 1301 TOP: Nursing Process: Assessment MSC: NCLEX: Health Promotion and Maintenance

Which information about a 76-year-old patient is most important for the admitting nurse to report to the patient's health care provider? a. Triceps reflex response graded at 1/5 b. Unintended weight loss of 20 pounds c. 10 mm Hg orthostatic drop in systolic blood pressure d. Patient complaint of chronic difficulty in falling asleep

ANS: B Although changes in appetite are normal with aging, a 20-pound weight loss requires further investigation. Orthostatic drops in blood pressure, changes in sleep patterns, and slowing of reflexes are normal changes in aging

Several patients have been hospitalized for diagnosis of neurologic problems. Which patient will the nurse assess first? a. Patient with a transient ischemic attack (TIA) returning from carotid duplex studies b. Patient with a brain tumor who has just arrived on the unit after a cerebral angiogram c. Patient with a seizure disorder who has just completed an electroencephalogram (EEG) d. Patient prepared for a lumbar puncture whose health care provider is waiting for assistance

ANS: B Because cerebral angiograms require insertion of a catheter into the femoral artery, bleeding is a possible complication. The nurse will need to check the pulse, blood pressure, and the catheter insertion site in the groin as soon as the patient arrives. Carotid duplex studies and EEG are noninvasive. The nurse will need to assist with the lumbar puncture as soon as possible, but monitoring for hemorrhage after cerebral angiogram has a higher priority

Several patients have been hospitalized for diagnosis of neurologic problems. Which patient will the nurse assess first? a. A patient with a transient ischemic attack (TIA) returning from carotid duplex studies b. A patient with a brain tumor who has just arrived on the unit after a cerebral angiogram c. A patient with a seizure disorder who has just completed an electroencephalogram (EEG) d. A patient prepared for a lumbar puncture whose health care provider is waiting for assistance

ANS: B Because cerebral angiograms require insertion of a catheter into the femoral artery, bleeding is a possible complication. The nurse will need to check the pulse, blood pressure, and the catheter insertion site in the groin as soon as the patient arrives. Carotid duplex studies and EEG are noninvasive. The nurse will need to assist with the lumbar puncture as soon as possible, but monitoring for hemorrhage after cerebral angiogram has a higher priority. DIF: Cognitive Level: Analyze (analysis) REF: 1310 OBJ: Special Questions: Prioritization | Special Questions: Multiple Patients TOP: Nursing Process: Planning MSC: NCLEX: Physiological Integrity

Which finding would the nurse expect when assessing the legs of a patient who has a lower motor neuron lesion? a. Spasticity b. Flaccidity c. No sensation d. Hyperactive reflexes

ANS: B Because the cell bodies of lower motor neurons are located in the spinal cord, damage to the neuron will decrease motor activity of the affected muscles. Spasticity and hyperactive reflexes are caused by upper motor neuron damage. Sensation is not impacted by motor neuron lesions

Which finding would the nurse expect when assessing the legs of a patient who has a lower motor neuron lesion? a. Spasticity b. Flaccidity c. Impaired sensation d. Hyperactive reflexes

ANS: B Because the cell bodies of lower motor neurons are located in the spinal cord, damage to the neuron will decrease motor activity of the affected muscles. Spasticity and hyperactive reflexes are caused by upper motor neuron damage. Sensation is not impacted by motor neuron lesions. DIF: Cognitive Level: Understand (comprehension) REF: 1296 TOP: Nursing Process: Assessment MSC: NCLEX: Physiological Integrity

A patient who has a cast in place after fracturing the radius asks when the cast can be removed. The nurse will instruct the patient that the cast will need to remain in place a. for several months. b. for at least 3 weeks. c. until swelling of the wrist has resolved. d. until x-rays show complete bony union.

ANS: B Bone healing starts immediately after the injury, but since ossification does not begin until 3 weeks postinjury, the cast will need to be worn for at least 3 weeks. Complete union may take up to a year. Resolution of swelling does not indicate bone healing.

After noting that a patient with a head injury has clear nasal drainage, which action should the nurse take? a. Have the patient blow the nose. b. Check the nasal drainage for glucose. c. Assure the patient that rhinorrhea is normal after a head injury. d. Obtain a specimen of the fluid to send for culture and sensitivity.

ANS: B Clear nasal drainage in a patient with a head injury suggests a dural tear and cerebrospinal fluid (CSF) leakage. If the drainage is CSF, it will test positive for glucose. Fluid leaking from the nose will have normal nasal flora, so culture and sensitivity will not be useful. Blowing the nose is avoided to prevent CSF leakage. DIF: Cognitive Level: Application REF: 1438-1439

Which statement by a patient who is being discharged from the emergency department (ED) after a head injury indicates a need for intervention by the nurse? a. "I will return if I feel dizzy or nauseated." b. "I am going to drive home and go to bed." c. "I do not even remember being in an accident." d. "I can take acetaminophen (Tylenol) for my headache."

ANS: B Following a head injury, the patient should avoid operating heavy machinery. Retrograde amnesia is common after a concussion. The patient can take acetaminophen for headache and should return if symptoms of increased intracranial pressure such as dizziness or nausea occur. DIF: Cognitive Level: Application REF: 1444

Propranolol (Inderal), a b-adrenergic blocker that inhibits sympathetic nervous system activity, is prescribed for a patient who has extreme anxiety about public speaking. The nurse monitors the patient for a. dry mouth. b. bradycardia. c. constipation. d. urinary retention.

ANS: B Inhibition of the fight or flight response leads to a decreased heart rate. Dry mouth, constipation, and urinary retention are associated with peripheral nervous system blockade

Propranolol (Inderal), a β-adrenergic blocker that inhibits sympathetic nervous system activity, is prescribed for a patient who has extreme anxiety about public speaking. The nurse monitors the patient for a. dry mouth. b. bradycardia. c. constipation. d. urinary retention.

ANS: B Inhibition of the fight-or-flight response leads to a decreased heart rate. Dry mouth, constipation, and urinary retention are associated with peripheral nervous system blockade. DIF: Cognitive Level: Understand (comprehension) REF: 1299 TOP: Nursing Process: Evaluation MSC: NCLEX: Physiological Integrity

A 39-year-old patient with a suspected herniated intervertebral disc is scheduled for a myelogram. Which information is most important for the nurse to communicate to the health care provider before the procedure? a. The patient is anxious about the test. b. The patient has an allergy to shellfish. c. The patient has back pain when lying flat. d. The patient drank apple juice 4 hours earlier.

ANS: B Iodine-containing contrast medium is injected into the subarachnoid space during a myelogram. The health care provider may need to modify the postmyelogram orders to prevent back pain, but this can be done after the procedure. Clear liquids are usually considered safe up to 4 hours before a diagnostic or surgical procedure. The patient's anxiety should be addressed, but this is not as important as the iodine allergy

The home health registered nurse (RN) is planning care for a patient with a seizure disorder related to a recent head injury. Which nursing action can be delegated to a licensed practical/vocational nurse (LPN/LVN)? a. Make referrals to appropriate community agencies. b. Place medications in the home medication organizer. c. Teach the patient and family how to manage seizures. d. Assess for use of medications that may precipitate seizures.

ANS: B LPN/LVN education includes administration of medications. The other activities require RN education and scope of practice. DIF: Cognitive Level: Apply (application) REF: 1381 OBJ: Special Questions: Delegation TOP: Nursing Process: Planning MSC: NCLEX: Safe and Effective Care Environment

The home health registered nurse (RN) is planning care for a patient with a seizure disorder related to a recent head injury. Which nursing action can be delegated to a licensed practical/vocational nurse (LPN/LVN)? a. Make referrals to appropriate community agencies. b. Place medications in the home medication organizer. c. Teach the patient and family how to manage seizures. d. Assess for use of medications that may precipitate seizures.

ANS: B LPN/LVN education includes administration of medications. The other activities require RN education and scope of practice. DIF: Cognitive Level: Apply (application) REF: 1426 OBJ: Special Questions: Delegation TOP: Nursing Process: Planning MSC: NCLEX: Safe and Effective Care Environment

A patient has hip replacement surgery using the posterior approach. Which patient action requires rapid intervention by the nurse? a. The patient uses crutches with a swing-to gait. b. The patient leans over to pull shoes and socks on. c. The patient sits straight up on the edge of the bed. d. The patient bends over the sink while brushing the teeth.

ANS: B Leaning over would flex the hip at greater than 90 degrees and predispose the patient to hip dislocation. The other patient actions are appropriate and do not require any immediate action by the nurse to protect the patient.

Which intervention will the nurse include in the plan of care for a patient with primary restless legs syndrome (RLS) who is having difficulty sleeping? a. Teach about the use of antihistamines to improve sleep. b. Suggest that the patient exercise regularly during the day. c. Make a referral to a massage therapist for deep massage of the legs. d. Assure the patient that the problem is transient and likely to resolve.

ANS: B Nondrug interventions such as getting regular exercise are initially suggested to improve sleep quality in patients with RLS. Antihistamines may aggravate RLS. Massage does not alleviate RLS symptoms, and RLS is likely to progress in most patients. DIF: Cognitive Level: Apply (application) REF: 1382 TOP: Nursing Process: Planning MSC: NCLEX: Physiological Integrity

When planning care for a patient who has had hip replacement surgery, which nursing action can the nurse delegate to experienced nursing assistive personnel (NAP)? a. Teach quadriceps-setting exercises. b. Reposition the patient every 1 to 2 hours. c. Assess for skin irritation on the patient's back. d. Determine the patient's pain level and tolerance.

ANS: B Repositioning of patients is within the scope of practice of NAP (after they have been trained and evaluated in this skill). The other actions should be done by licensed nursing staff members.

A patient with Parkinson's disease has bradykinesia. Which action will the nurse include in the plan of care? a. Instruct the patient in activities that can be done while lying or sitting. b. Suggest that the patient rock from side to side to initiate leg movement. c. Have the patient take small steps in a straight line directly in front of the feet. d. Teach the patient to keep the feet in contact with the floor and slide them forward.

ANS: B Rocking the body from side to side stimulates balance and improves mobility. The patient will be encouraged to continue exercising because this will maintain functional abilities. Maintaining a wide base of support will help with balance. The patient should lift the feet and avoid a shuffling gait. DIF: Cognitive Level: Apply (application) REF: 1392 TOP: Nursing Process: Planning MSC: NCLEX: Physiological Integrity

A 73-year-old patient with Parkinson's disease has a nursing diagnosis of impaired physical mobility related to bradykinesia. Which action will the nurse include in the plan of care? a. Instruct the patient in activities that can be done while lying or sitting. b. Suggest that the patient rock from side to side to initiate leg movement. c. Have the patient take small steps in a straight line directly in front of the feet. d. Teach the patient to keep the feet in contact with the floor and slide them forward.

ANS: B Rocking the body from side to side stimulates balance and improves mobility. The patient will be encouraged to continue exercising because this will maintain functional abilities. Maintaining a wide base of support will help with balance. The patient should lift the feet and avoid a shuffling gait. DIF: Cognitive Level: Apply (application) REF: 1437 TOP: Nursing Process: Planning MSC: NCLEX: Physiological Integrity

When family members ask the nurse about the purpose of the ventriculostomy system being used for intracranial pressure monitoring for a patient, which response by the nurse is best? a. "This type of monitoring system is complex and highly skilled staff are needed." b. "The monitoring system helps show whether blood flow to the brain is adequate." c. "The ventriculostomy monitoring system helps check for alterations in cerebral perfusion pressure." d. "This monitoring system has multiple benefits including facilitation of cerebrospinal fluid drainage."

ANS: B Short and simple explanations should be given to patients and family members. The other explanations are either too complicated to be easily understood or may increase the family member's anxiety. DIF: Cognitive Level: Application REF: 1438

After suctioning, the nurse notes that the intracranial pressure for a patient with a traumatic head injury has increased from 14 to 16 mm Hg. Which action should the nurse take first? a. Document the increase in intracranial pressure. b. Assure that the patient's neck is not in a flexed position. c. Notify the health care provider about the change in pressure. d. Increase the rate of the prescribed propofol (Diprovan) infusion.

ANS: B Since suctioning will cause a transient increase in intracranial pressure, the nurse should initially check for other factors that might be contributing to the increase and observe the patient for a few minutes. Documentation is needed, but this is not the first action. There is no need to notify the health care provider about this expected reaction to suctioning. Propofol is used to control patient anxiety or agitation; there is no indication that anxiety has contributed to the increase in intracranial pressure. DIF: Cognitive Level: Application REF: 1426 | 1435-1437 | 1436-1437

Which action will the nurse plan to take for a patient with multiple sclerosis who has urinary retention caused by a flaccid bladder? a. Encourage a decreased evening intake of fluid. b. Teach the patient how to use the Credé method. c. Suggest the use of adult incontinence briefs for nighttime only. d. Assist the patient to the commode every 2 hours during the day.

ANS: B The Credé method can be used to improve bladder emptying. Decreasing fluid intake will not improve bladder emptying and may increase risk for urinary tract infection and dehydration. The use of incontinence briefs and frequent toileting will not improve bladder emptying. DIF: Cognitive Level: Apply (application) REF: 1387 TOP: Nursing Process: Planning MSC: NCLEX: Physiological Integrity

When the nurse is caring for a patient who is on bed rest after having a complex pelvic fracture, which assessment finding is most important to report to the health care provider? a. The patient states that the pelvis feels unstable. b. Abdominal distention is present and bowel tones are absent. c. There are ecchymoses on the abdomen and hips. d. The patient complains of pelvic pain with palpation.

ANS: B The abdominal distention and absent bowel tones may be due to complications of pelvic fractures such as paralytic ileus or hemorrhage or trauma to the bladder, urethra, or colon. Pelvic instability, abdominal pain with palpation, and abdominal bruising would be expected with this type of injury.

When a 74-yr-old patient is seen in the health clinic with new development of a stooped posture, shuffling gait, and pill rolling-type tremor, the nurse will anticipate teaching the patient about a. oral corticosteroids. b. antiparkinsonian drugs. c. magnetic resonance imaging (MRI). d. electroencephalogram (EEG) testing.

ANS: B The clinical diagnosis of Parkinson's is made when tremor, rigidity, and akinesia, and postural instability are present. The confirmation of the diagnosis is made on the basis of improvement when antiparkinsonian drugs are administered. MRI and EEG are not useful in diagnosing Parkinson's disease, and corticosteroid therapy is not used to treat it. DIF: Cognitive Level: Apply (application) REF: 1389 TOP: Nursing Process: Planning MSC: NCLEX: Physiological Integrity

When a 74-year-old patient is seen in the health clinic with new development of a stooped posture, shuffling gait, and pill rolling-type tremor, the nurse will anticipate teaching the patient about a. oral corticosteroids. b. antiparkinsonian drugs. c. magnetic resonance imaging (MRI). d. electroencephalogram (EEG) testing.

ANS: B The diagnosis of Parkinson's is made when two of the three characteristic manifestations of tremor, rigidity, and bradykinesia are present. The confirmation of the diagnosis is made on the basis of improvement when antiparkinsonian drugs are administered. This patient has symptoms of tremor and bradykinesia. The next anticipated step will be treatment with medications. MRI and EEG are not useful in diagnosing Parkinson's disease, and corticosteroid therapy is not used to treat it. DIF: Cognitive Level: Apply (application) REF: 1434 TOP: Nursing Process: Planning MSC: NCLEX: Physiological Integrity

Which action will the nurse include in the plan of care for a patient with impaired functioning of the left glossopharyngeal nerve (CN IX) and vagus nerve (CN X)? a. Assist to stand and ambulate. b. Withhold oral fluids and food. c. Insert an oropharyngeal airway. d. Apply artificial tears every hour.

ANS: B The glossopharyngeal and vagus nerves innervate the pharynx and control the gag reflex. A patient with impaired function of these nerves is at risk for aspiration. An oral airway may be needed when a patient is unconscious and unable to maintain the airway, but it will not decrease aspiration risk. Taste and eye blink are controlled by the facial nerve. Balance and coordination are cerebellar functions. DIF: Cognitive Level: Apply (application) REF: 1305 TOP: Nursing Process: Planning MSC: NCLEX: Physiological Integrity

Which medication taken by a patient with restless legs syndrome should the nurse discuss with the patient? a. Ibuprofen b. Multivitamin c. Acetaminophen d. Diphenhydramine

ANS: D Antihistamines can aggravate restless legs syndrome. The other medications will not contribute to restless legs syndrome. DIF: Cognitive Level: Apply (application) REF: 1383 TOP: Nursing Process: Implementation MSC: NCLEX: Physiological Integrity

Following a head injury, an unconscious 32-year-old patient is admitted to the emergency department (ED). The patient's spouse and children stay at the patient's side and constantly ask about the treatment being given. What action is best for the nurse to take? a. Ask the family to stay in the waiting room until the initial assessment is completed. b. Allow the family to stay with the patient and briefly explain all procedures to them. c. Call the family's pastor or spiritual advisor to support them while initial care is given. d. Refer the family members to the hospital counseling service to deal with their anxiety.

ANS: B The need for information about the diagnosis and care is very high in family members of acutely ill patients, and the nurse should allow the family to observe care and explain the procedures. A pastor or counseling service can offer some support, but research supports information as being more effective. Asking the family to stay in the waiting room will increase their anxiety. DIF: Cognitive Level: Application REF: 1438

The nurse is preparing to assist a patient who has had an open reduction and internal fixation (ORIF) of a hip fracture out of bed for the first time. Which action should the nurse take? a. Use a mechanical lift to transfer the patient from the bed to the chair. b. Check the postoperative orders for the patient's weight-bearing status. c. Avoid administration of pain medications before getting the patient up. d. Delegate the transfer of the patient out of bed to nursing assistive personnel (NAP).

ANS: B The nurse should be familiar with the weight-bearing orders for the patient before attempting the transfer. Mechanical lifts are not typically needed after this surgery. Pain medications should be given, since the movement is likely to be painful for the patient. The RN should supervise the patient during the initial transfer to evaluate how well the patient is able to accomplish this skill.

A high school teacher who has been diagnosed with epilepsy after having a generalized tonic-clonic seizure tells the nurse, "I cannot teach any more. It will be too upsetting if I have a seizure at work." Which response by the nurse specifically addresses the patient's concern? a. "You might benefit from some psychologic counseling." b. "Epilepsy usually can be well controlled with medications." c. "You will want to contact the Epilepsy Foundation for assistance." d. "The Department of Vocational Rehabilitation can help with work retraining."

ANS: B The nurse should inform the patient that most patients with seizure disorders are controlled with medication. The other information may be necessary if the seizures persist after treatment with antiseizure medications is implemented. DIF: Cognitive Level: Apply (application) REF: 1381 TOP: Nursing Process: Implementation MSC: NCLEX: Psychosocial Integrity

A high school teacher who has just been diagnosed with epilepsy after having a generalized tonic-clonic seizure tells the nurse, "I cannot teach anymore, it will be too upsetting if I have a seizure at work." Which response by the nurse is best? a. "You might benefit from some psychologic counseling." b. "Epilepsy usually can be well controlled with medications." c. "You will want to contact the Epilepsy Foundation for assistance." d. "The Department of Vocational Rehabilitation can help with work retraining."

ANS: B The nurse should inform the patient that most patients with seizure disorders are controlled with medication. The other information may be necessary if the seizures persist after treatment with antiseizure medications is implemented. DIF: Cognitive Level: Apply (application) REF: 1422 TOP: Nursing Process: Implementation MSC: NCLEX: Psychosocial Integrity

A patient with a head injury opens the eyes to verbal stimulation, curses when stimulated, and does not respond to a verbal command to move but attempts to remove a painful stimulus. The nurse records the patient's Glasgow Coma Scale score as a. 9. b. 11. c. 13. d. 15.

ANS: B The patient has a score of 3 for eye opening, 3 for best verbal response, and 5 for best motor response. DIF: Cognitive Level: Application REF: 1434

A patient who has been hospitalized for 3 days with a hip fracture has sudden onset shortness of breath and tachypnea. The patient tells the nurse, "I feel like I am going to die!" Which action should the nurse take first? a. Stay with the patient and offer reassurance. b. Administer the prescribed PRN oxygen at 4 L/min. c. Check the patient's legs for swelling or tenderness. d. Notify the health care provider about the symptoms.

ANS: B The patient's clinical manifestations and history are consistent with a pulmonary embolus, and the nurse's first action should be to ensure adequate oxygenation. The nurse should offer reassurance to the patient, but meeting the physiologic need for oxygen is a higher priority. The health care provider should be notified after the oxygen is started and pulse oximetry and assessment for fat embolus or venous thromboembolism (VTE) are obtained.

After a thymectomy, a patient with myasthenia gravis receives the usual dose of pyridostigmine (Mestinon). An hour later, the patient complains of nausea and severe abdominal cramps. Which action should the nurse take first? a. Auscultate the patient's bowel sounds. b. Notify the patient's health care provider. c. Administer the prescribed PRN antiemetic drug. d. Give the scheduled dose of prednisone (Deltasone).

ANS: B The patient's history and symptoms indicate a possible cholinergic crisis. The health care provider should be notified immediately, and it is likely that atropine will be prescribed. The other actions will be appropriate if the patient is not experiencing a cholinergic crisis. DIF: Cognitive Level: Analyze (analysis) REF: 1394 OBJ: Special Questions: Prioritization TOP: Nursing Process: Implementation MSC: NCLEX: Physiological Integrity

ollowing a thymectomy, a 62-year-old male patient with myasthenia gravis receives the usual dose of pyridostigmine (Mestinon). An hour later, the patient complains of nausea and severe abdominal cramps. Which action should the nurse take first? a. Auscultate the patient's bowel sounds. b. Notify the patient's health care provider. c. Administer the prescribed PRN antiemetic drug. d. Give the scheduled dose of prednisone (Deltasone).

ANS: B The patient's history and symptoms indicate a possible cholinergic crisis. The health care provider should be notified immediately, and it is likely that atropine will be prescribed. The other actions will be appropriate if the patient is not experiencing a cholinergic crisis. DIF: Cognitive Level: Apply (application) REF: 1438-1439 OBJ: Special Questions: Prioritization TOP: Nursing Process: Implementation MSC: NCLEX: Physiological Integrity

A patient with possible cerebral edema has a serum sodium level of 115 mEq/L (115 mmol/L) and a decreasing level of consciousness (LOC) and complains of a headache. Which of these prescribed interventions should the nurse implement first? a. Draw blood for arterial blood gases (ABGs). b. Administer 5% hypertonic saline intravenously. c. Administer acetaminophen (Tylenol) 650 mg orally. d. Send patient for computed tomography (CT) of the head.

ANS: B The patient's low sodium indicates that hyponatremia may be causing the cerebral edema, and the nurse's first action should be to correct the low sodium level. Acetaminophen (Tylenol) will have minimal effect on the headache because it is caused by cerebral edema and increased intra-cranial pressure (ICP). Drawing ABGs and obtaining a CT scan may add some useful information, but the low sodium level may lead to seizures unless it is addressed quickly. DIF: Cognitive Level: Application REF: 1452-1455

Which cerebrospinal fluid analysis result will be most important for the nurse to communicate to the health care provider? a. Specific gravity 1.007 b. Protein 65 mg/dL (0.65 g/L) c. Glucose 45 mg/dL (1.7 mmol/L) d. White blood cell (WBC) count 4 cells/mL

ANS: B The protein level is high. The specific gravity, WBCs, and glucose values are normal

Which cerebrospinal fluid analysis result should the nurse recognize as abnormal and communicate to the health care provider? a. Specific gravity of 1.007 b. Protein of 65 mg/dL (0.65 g/L) c. Glucose of 45 mg/dL (1.7 mmol/L) d. White blood cell (WBC) count of 4 cells/μL

ANS: B The protein level is high. The specific gravity, WBCs, and glucose values are normal. DIF: Cognitive Level: Understand (comprehension) REF: 1298 TOP: Nursing Process: Implementation MSC: NCLEX: Physiological Integrity

The health care provider is considering the use of sumatriptan (Imitrex) for a 54-yr-old male patient with migraine headaches. Which information obtained by the nurse is most important to report to the health care provider? a. The patient drinks 1 to 2 cups of coffee daily. b. The patient had a recent acute myocardial infarction. c. The patient has had migraine headaches for 30 years. d. The patient has taken topiramate (Topamax) for 2 months.

ANS: B The triptans cause coronary artery vasoconstriction and should be avoided in patients with coronary artery disease. The other information will be reported to the health care provider, but none of it indicates that sumatriptan would be an inappropriate treatment. DIF: Cognitive Level: Analyze (analysis) REF: 1372 OBJ: Special Questions: Prioritization TOP: Nursing Process: Assessment MSC: NCLEX: Physiological Integrity

The nurse expects the assessment of a patient who is experiencing a cluster headache to include a. nuchal rigidity. b. unilateral ptosis. c. projectile vomiting. d. throbbing, bilateral facial pain.

ANS: B Unilateral eye edema, tearing, and ptosis are characteristic of cluster headaches. Nuchal rigidity suggests meningeal irritation, such as occurs with meningitis. Although nausea and vomiting may occur with migraine headaches, projectile vomiting is more consistent with increased intracranial pressure. Unilateral sharp, stabbing pain, rather than throbbing pain, is characteristic of cluster headaches. DIF: Cognitive Level: Understand (comprehension) REF: 1371 TOP: Nursing Process: Assessment MSC: NCLEX: Physiological Integrity

A patient has a long-arm plaster cast applied for immobilization of a fractured left radius. Until the cast has completely dried, the nurse should a. keep the left arm in a dependent position. b. handle the cast with the palms of the hands. c. place gauze around the cast edge to pad any roughness. d. cover the cast with a small blanket to absorb the dampness.

ANS: B Until a plaster cast has dried, placing pressure on the cast should be avoided to prevent creating areas inside the cast that could place pressure on the arm. The left arm should be elevated to prevent swelling. The edges of the cast may be petaled once the cast is dry, but padding the edges before that may cause the cast to be misshapen. The cast should not be covered until it is dry because heat builds up during drying.

Which equipment will the nurse obtain to assess vibration sense in a diabetic patient who has peripheral nerve dysfunction? a. Sharp pin b. Tuning fork c. Reflex hammer d. Calibrated compass

ANS: B Vibration sense is testing by touching the patient with a vibrating tuning fork. The other equipment is needed for testing of pain sensation, reflexes, and two-point discrimination

The priority nursing assessment for a patient being admitted with a brainstem infarction is a. pupil reaction. b. respiratory rate. c. reflex reaction time. d. level of consciousness.

ANS: B Vital centers that control respiration are located in the medulla and part of the brainstem, and will require priority assessments because changes in respiratory function may be life threatening. The other information will also be obtained by the nurse but is not as urgent. DIF: Cognitive Level: Apply (application) REF: 1297 OBJ: Special Questions: Prioritization TOP: Nursing Process: Assessment MSC: NCLEX: Physiological Integrity

Which assessments will the nurse make to monitor a patient's cerebellar function (select all that apply)? a. Assess for graphesthesia. b. Observe arm swing with gait. c. Perform the finger-to-nose test. d. Check ability to push against resistance. e. Determine ability to sense heat and cold.

ANS: B, C The cerebellum is responsible for coordination and is assessed by looking at the patient's gait and the finger-to-nose test. The other assessments will be used for other parts of the neurologic assessment

Which assessments will the nurse make to monitor a patient's cerebellar function (select all that apply)? a. Test for graphesthesia. b. Observe arm swing with gait. c. Perform the finger-to-nose test. d. Assess heat and cold sensation. e. Measure strength against resistance.

ANS: B, C The cerebellum is responsible for coordination and is assessed by looking at the patient's gait and the finger-to-nose test. The other assessments will be used for other parts of the neurologic assessment. DIF: Cognitive Level: Apply (application) REF: 1306 TOP: Nursing Process: Assessment MSC: NCLEX: Health Promotion and Maintenance

A 40-yr-old patient is diagnosed with early Huntington's disease (HD). When teaching the patient, spouse, and adult children about this disorder, the nurse will provide information about the a. use of levodopa-carbidopa (Sinemet) to help reduce HD symptoms. b. prophylactic antibiotics to decrease the risk for aspiration pneumonia. c. option of genetic testing for the patient's children to determine their own HD risks. d. lifestyle changes of improved nutrition and exercise that delay disease progression.

ANS: C Genetic testing is available to determine whether an asymptomatic individual has the HD gene. The patient and family should be informed of the benefits and problems associated with genetic testing. Sinemet will increase symptoms of HD because HD involves an increase in dopamine. Antibiotic therapy will not reduce the risk for aspiration. There are no effective treatments or lifestyle changes that delay the progression of symptoms in HD. DIF: Cognitive Level: Apply (application) REF: 1396 TOP: Nursing Process: Implementation MSC: NCLEX: Physiological Integrity

A patient admitted with bacterial meningitis and a temperature of 102° F (38.8° C) has orders for all of these collaborative interventions. Which action should the nurse take first? a. Administer ceftizoxime (Cefizox) 1 g IV. b. Use a cooling blanket to lower temperature. c. Swap the nasopharyngeal mucosa for cultures. d. Give acetaminophen (Tylenol) 650 mg PO.

ANS: C Antibiotic therapy should be instituted rapidly in bacterial meningitis, but cultures must be done before antibiotics are started. As soon as the cultures are done, the antibiotic should be started. Hypothermia therapy and acetaminophen administration are appropriate but can be started after the other actions are implemented. DIF: Cognitive Level: Application REF: 1440-1441

After a patient with a left femur fracture has a hip spica cast applied, which nursing intervention will be included in the plan of care? a. Avoid placing the patient in the prone position. b. Use the cast support bar to reposition the patient. c. Ask the patient about any abdominal discomfort or nausea. d. Discuss the reasons for remaining on bed rest for several weeks.

ANS: C Assessment of bowel tones, abdominal pain, and nausea and vomiting will detect the development of cast syndrome. To avoid breakage, the support bar should not be used for repositioning. After the cast dries, the patient can begin ambulating with the assistance of physical therapy personnel and may be turned to the prone position.

Which nursing action will be included in the plan of care for a patient who has had cerebral angiography? a. Monitor for headache and photophobia. b. Keep patient NPO until gag reflex returns. c. Check pulse and blood pressure frequently. d. Assess orientation to person, place, and time.

ANS: C Because a catheter is inserted into an artery (e.g., the femoral artery) during cerebral angiography, the nurse should assess for bleeding after this procedure that can affect pulse and blood pressure. The other nursing assessments are not needed after angiography. DIF: Cognitive Level: Apply (application) REF: 1310 TOP: Nursing Process: Planning MSC: NCLEX: Physiological Integrity

Which nursing action will be included in the care for a patient who has had cerebral angiography? a. Monitor for headache and photophobia. b. Keep patient NPO until gag reflex returns. c. Check pulse and blood pressure frequently. d. Assess orientation to person, place, and time.

ANS: C Because a catheter is inserted into an artery (such as the femoral artery) during cerebral angiography, the nurse should assess for bleeding after this procedure. The other nursing assessments are not necessary after angiography

Which assessment is most important for the nurse to make regarding a patient with myasthenia gravis? a. Pupil size c. Respiratory effort b. Grip strength d. Level of consciousness

ANS: C Because respiratory insufficiency may be life threatening, it will be most important to monitor respiratory function. The other data also will be assessed but are not as critical. DIF: Cognitive Level: Analyze (analysis) REF: 1393 OBJ: Special Questions: Prioritization TOP: Nursing Process: Assessment MSC: NCLEX: Physiological Integrity

Which assessment is most important for the nurse to make regarding a patient with myasthenia gravis? a. Pupil size b. Grip strength c. Respiratory effort d. Level of consciousness

ANS: C Because respiratory insufficiency may be life threatening, it will be most important to monitor respiratory function. The other data also will be assessed but are not as critical. DIF: Cognitive Level: Apply (application) REF: 1438-1439 OBJ: Special Questions: Prioritization TOP: Nursing Process: Assessment MSC: NCLEX: Physiological Integrity

Which action will the nurse take in order to evaluate the effectiveness of Buck's traction for a patient who has an intracapsular fracture of the left femur? a. Assess for hip contractures. b. Monitor for hip dislocation. c. Check the peripheral pulses. d. Ask about left hip pain level.

ANS: D Buck's traction keeps the leg immobilized and reduces painful muscle spasm. Hip contractures and dislocation are unlikely to occur in this situation. The peripheral pulses will be assessed, but this does not help in evaluating the effectiveness of Buck's traction.

While the nurse is transporting a patient on a stretcher to the radiology department, the patient begins having a tonic-clonic seizure. Which action should the nurse take? a. Insert an oral airway during the seizure to maintain a patent airway. b. Restrain the patient's arms and legs to prevent injury during the seizure. c. Time and observe and record the details of the seizure and postictal state. d. Avoid touching the patient to prevent further nervous system stimulation.

ANS: C Because the diagnosis and treatment of seizures frequently are based on the description of the seizure, recording the length and details of the seizure is important. Insertion of an oral airway and restraining the patient during the seizure are contraindicated. The nurse may need to move the patient to decrease the risk of injury during the seizure. DIF: Cognitive Level: Apply (application) REF: 1381 TOP: Nursing Process: Implementation MSC: NCLEX: Physiological Integrity

While the nurse is transporting a patient on a stretcher to the radiology department, the patient begins having a tonic-clonic seizure. Which action should the nurse take? a. Insert an oral airway during the seizure to maintain a patent airway. b. Restrain the patient's arms and legs to prevent injury during the seizure. c. Time and observe and record the details of the seizure and postictal state. d. Avoid touching the patient to prevent further nervous system stimulation.

ANS: C Because the diagnosis and treatment of seizures frequently are based on the description of the seizure, recording the length and details of the seizure is important. Insertion of an oral airway and restraining the patient during the seizure are contraindicated. The nurse may need to move the patient to decrease the risk of injury during the seizure. DIF: Cognitive Level: Apply (application) REF: 1422 TOP: Nursing Process: Implementation MSC: NCLEX: Physiological Integrity

When counseling an older patient about ways to prevent fractures, which information will the nurse include? a. Tack down scatter rugs in the home. b. Most falls happen outside the home. c. Buy shoes that provide good support and are comfortable to wear. d. Range-of-motion exercises should be taught by a physical therapist.

ANS: C Comfortable shoes with good support will help decrease the risk for falls. Scatter rugs should be eliminated, not just tacked down. Activities of daily living provide range of motion exercise; these do not need to be taught by a physical therapist. Falls inside the home are responsible for many injuries.

A 33-yr-old patient with multiple sclerosis (MS) is to begin treatment with glatiramer acetate (Copaxone). Which information will the nurse include in patient teaching? a. Recommendation to drink at least 4 L of fluid daily b. Need to avoid driving or operating heavy machinery c. How to draw up and administer injections of the medication d. Use of contraceptive methods other than oral contraceptives

ANS: C Copaxone is administered by self-injection. Oral contraceptives are an appropriate choice for birth control. There is no need to avoid driving or drink large fluid volumes when taking glatiramer. DIF: Cognitive Level: Apply (application) REF: 1385 TOP: Nursing Process: Implementation MSC: NCLEX: Physiological Integrity

Which information about a 60-yr-old patient with multiple sclerosis indicates that the nurse should consult with the health care provider before giving the prescribed dose of dalfampridine (Ampyra)? a. The patient walks a mile each day for exercise. b. The patient complains of pain with neck flexion. c. The patient has an increased serum creatinine level. d. The patient has the relapsing-remitting form of MS.

ANS: C Dalfampridine should not be given to patients with impaired renal function. The other information will not impact whether the dalfampridine should be administered. DIF: Cognitive Level: Apply (application) REF: 1386 TOP: Nursing Process: Assessment MSC: NCLEX: Physiological Integrity

During the neurologic assessment, the patient is unable to respond verbally to the nurse but cooperates with the nurse's directions to move his hands and feet. The nurse will suspect a. cerebellar injury. b. a brainstem lesion. c. frontal lobe damage. d. a temporal lobe lesion.

ANS: C Expressive speech (ability to express the self in language) is controlled by Broca's area in the frontal lobe. The temporal lobe contains Wernicke's area, which is responsible for receptive speech (ability to understand language input). The cerebellum and brainstem do not affect higher cognitive functions such as speech. DIF: Cognitive Level: Apply (application) REF: 1296 TOP: Nursing Process: Assessment MSC: NCLEX: Physiological Integrity

During the neurologic assessment, the patient is unable to respond verbally to the nurse but cooperates with the nurse's directions to move his hands and feet. The nurse will suspect a. cerebellar injury. b. a brainstem lesion. c. frontal lobe damage. d. a temporal lobe lesion.

ANS: C Expressive speech is controlled by Broca's area in the frontal lobe. The temporal lobe contains Wernicke's area, which is responsible for receptive speech. The cerebellum and brainstem do not affect higher cognitive functions such as speech

After a patient has a short-arm plaster cast applied in the emergency department, which statement by the patient indicates a good understanding of the nurse's discharge teaching? a. "I can get the cast wet as long as I dry it right away with a hair dryer." b. "I should avoid moving my fingers and elbow until the cast is removed." c. "I will apply an ice pack to the cast over the fracture site for the next 24 hours." d. "I can use a cotton-tipped applicator to rub lotion on any dry areas under the cast."

ANS: C Ice application for the first 24 hours after a fracture will help reduce swelling and can be placed over the cast. Plaster casts should not get wet. The patient should be encouraged to move the joints above and below the cast. Patients should not insert objects inside the cast.

When the nurse applies a painful stimulus to the nail beds of an unconscious patient, the patient responds with internal rotation, adduction, and flexion of the arms. The nurse documents this as a. flexion withdrawal. b. localization of pain. c. decorticate posturing. d. decerebrate posturing.

ANS: C Internal rotation, adduction, and flexion of the arms in an unconscious patient is documented as decorticate posturing. Extension of the arms and legs is decerebrate posturing. Because the flexion is generalized, it does not indicate localization of pain or flexion withdrawal. DIF: Cognitive Level: Comprehension REF: 1429-1430

A patient is being treated with carbidopa/levodopa (Sinemet) for Parkinson's disease. Which information indicates a need for change in the medication or dosage? a. Shuffling gait c. Cogwheel rigidity of limbs b. Tremor at rest d. Uncontrolled head movement

ANS: D Dyskinesia is an adverse effect of the Sinemet, indicating a need for a change in medication or decrease in dose. The other findings are typical with Parkinson's disease. DIF: Cognitive Level: Apply (application) REF: 1390 TOP: Nursing Process: Planning MSC: NCLEX: Physiological Integrity

The nurse determines that teaching about management of migraine headaches has been effective when the patient says which of the following? a. "I can take the (Topamax) as soon as a headache starts." b. "A glass of wine might help me relax and prevent a headache." c. "I will lie down someplace dark and quiet when the headaches begin." d. "I should avoid taking aspirin and sumatriptan (Imitrex) at the same time."

ANS: C It is recommended that the patient with a migraine rest in a dark, quiet area. Topiramate (Topamax) is used to prevent migraines and must be taken for several months to determine effectiveness. Aspirin or other nonsteroidal antiinflammatory medications can be taken with the triptans. Alcohol may precipitate migraine headaches. DIF: Cognitive Level: Apply (application) REF: 1373 TOP: Nursing Process: Evaluation MSC: NCLEX: Physiological Integrity

Which information about a patient who has a new prescription for phenytoin (Dilantin) indicates that the nurse should consult with the health care provider before administration of the medication? a. Patient has tonic-clonic seizures. b. Patient experiences an aura before seizures. c. Patient has minor elevations in the liver function tests. d. Patient's most recent blood pressure is 156/92 mm Hg.

ANS: C Many older patients (especially with compromised liver function) may not be able to metabolize phenytoin. The health care provider may need to choose another antiseizure medication. Phenytoin is an appropriate medication for patients with tonic-clonic seizures, with or without an aura. Hypertension is not a contraindication for phenytoin therapy. DIF: Cognitive Level: Apply (application) REF: 1379 TOP: Nursing Process: Implementation MSC: NCLEX: Physiological Integrity

While caring for a patient who has just been admitted with meningococcal meningitis, the RN observes all of the following. Which one requires action by the RN? a. The bedrails at the head and foot of the bed are both elevated. b. The patient receives a regular diet from the dietary department. c. The nursing assistant goes into the patient's room without a mask. d. The lights in the patient's room are turned off and the blinds are shut.

ANS: C Meningococcal meningitis is spread by respiratory secretions, so it is important to maintain respiratory isolation as well as standard precautions. Because the patient may be confused and weak, bedrails should be elevated at both the food and head of the bed. Low light levels in the room decrease pain caused by photophobia. Nutrition is an important aspect of care in a patient with meningitis. DIF: Cognitive Level: Application REF: 1453-1455

The community health nurse is developing a program to decrease the incidence of meningitis in adolescents and young adults. Which nursing action is most important? a. Vaccinate 11- and 12-year-old children against Haemophilus influenzae. b. Emphasize the importance of hand washing to prevent spread of infection. c. Immunize adolescents and college freshman against Neisseria meningitides. d. Encourage adolescents and young adults to avoid crowded areas in the winter.

ANS: C The Neisseria meningitides vaccination is recommended for children ages 11 and 12, unvaccinated teens entering high school, and college freshmen. Hand washing may help decrease the spread of bacteria, but it is not as effective as immunization. Vaccination with Haemophilus influenzae is for infants and toddlers. Because adolescents and young adults are in school or the workplace, avoiding crowds is not realistic. DIF: Cognitive Level: Application REF: 1453-1455

Which information about a patient who is hospitalized after a traumatic brain injury requires the most rapid action by the nurse? a. Intracranial pressure of 15 mm Hg b. Cerebrospinal fluid (CSF) drainage of 15 mL/hour c. Pressure of oxygen in brain tissue (PbtO2) is 14 mm Hg d. Cardiac monitor shows sinus tachycardia, with a heart rate of 126 beats/min

ANS: C The PbtO2 should be 20 to 40 mm Hg. Lower levels indicate brain ischemia. An intracranial pressure (ICP) of 15 mm Hg is at the upper limit of normal. CSF is produced at a rate of 20 to 30 mL/hour. The reason for the sinus tachycardia should be investigated, but the elevated heart rate is not as concerning as the decrease in PbtO2. DIF: Cognitive Level: Application REF: 1430-1432

A patient who has a proximal humerus fracture that is immobilized with a left-sided long-arm cast and a sling is admitted to the medical-surgical unit. Which nursing intervention will be included in the plan of care? a. Use surgical net dressing to hang the arm from an IV pole. b. Immobilize the fingers on the left hand with gauze dressings. c. Assess the left axilla and change absorbent dressings as needed. d. Assist the patient in passive range of motion (ROM) for the right arm.

ANS: C The axilla can become excoriated when a sling is used to support the arm, and the nurse should check the axilla and apply absorbent dressings to prevent this. A patient with a sling would not have traction applied by hanging. The patient will be encouraged to move the fingers on the injured arm to maintain function and to help decrease swelling. The patient will do active ROM on the uninjured side.

A 22-yr-old patient seen at the health clinic with a severe migraine headache tells the nurse about having similar headaches recently. Which initial action should the nurse take? a. Teach about the use of triptan drugs. b. Refer the patient for stress counseling. c. Ask the patient to keep a headache diary. d. Suggest the use of muscle-relaxation techniques.

ANS: C The initial nursing action should be further assessment of the precipitating causes of the headaches, quality, and location of pain. Stress reduction, muscle relaxation, and the triptan drugs may be helpful, but more assessment is needed first. DIF: Cognitive Level: Analyze (analysis) REF: 1373 OBJ: Special Questions: Prioritization TOP: Nursing Process: Implementation MSC: NCLEX: Physiological Integrity

A patient has a systemic BP of 108/51 mm Hg and an intracranial pressure (ICP) of 14 mm Hg. Which action should the nurse take first? a. Elevate the head of the patient's bed to 60 degrees. b. Document the BP and ICP in the patient's record. c. Report the BP and ICP to the health care provider. d. Continue to monitor the patient's vital signs and ICP.

ANS: C The patient's cerebral perfusion pressure is 56 mm Hg, below the normal of 60 to 100 mm Hg and approaching the level of ischemia and neuronal death. Immediate changes in the patient's therapy such as fluid infusion or vasopressor administration are needed to improve the cerebral perfusion pressure. Adjustments in the head elevation should only be done after consulting with the health care provider. Continued monitoring and documentation also will be done, but they are not the first actions that the nurse should take. DIF: Cognitive Level: Analysis REF: 1426

The nurse performing a focused assessment of left posterior temporal lobe functions will assess the patient for a. sensation on the left side of the body. b. reasoning and problem-solving ability. c. ability to understand written and oral language. d. voluntary movements on the right side of the body.

ANS: C The posterior temporal lobe integrates the visual and auditory input for language comprehension. Reasoning and problem solving are functions of the anterior frontal lobe. Sensation on the left side of the body is located in the right postcentral gyrus. Voluntary movement on the right side is controlled in the left precentral gyrus. DIF: Cognitive Level: Apply (application) REF: 1298 TOP: Nursing Process: Assessment MSC: NCLEX: Physiological Integrity

To assess the functions of the trigeminal and facial nerves (CNs V and VII), the nurse should a. check for unilateral eyelid droop. b. shine a light into the patient's pupil. c. touch a cotton wisp strand to the cornea. d. have the patient read a magazine or book.

ANS: C The trigeminal and facial nerves are responsible for the corneal reflex. The optic nerve is tested by having the patient read a Snellen chart or a newspaper. Assessment of pupil response to light and ptosis are used to evaluate function of the oculomotor nerve. DIF: Cognitive Level: Understand (comprehension) REF: 1305 TOP: Nursing Process: Assessment MSC: NCLEX: Physiological Integrity

Which prescribed intervention will the nurse implement first for a patient in the emergency department who is experiencing continuous tonic-clonic seizures? a. Give phenytoin (Dilantin) 100 mg IV. b. Monitor level of consciousness (LOC). c. Administer lorazepam (Ativan) 4 mg IV. d. Obtain computed tomography (CT) scan.

ANS: C To prevent ongoing seizures, the nurse should administer rapidly acting antiseizure medications such as the benzodiazepines. A CT scan is appropriate, but prevention of any seizure activity during the CT scan is necessary. Phenytoin will also be administered, but it is not rapidly acting. Patients who are experiencing tonic-clonic seizures are nonresponsive, although the nurse should assess LOC after the seizure. DIF: Cognitive Level: Analyze (analysis) REF: 1378 OBJ: Special Questions: Prioritization TOP: Nursing Process: Implementation MSC: NCLEX: Physiological Integrity

The charge nurse is observing a new staff nurse who is assessing a patient with a traumatic spinal cord injury for sensation. Which action indicates a need for further teaching of the new nurse about neurologic assessment? a. The new nurse tests for light touch before testing for pain. b. The new nurse has the patient close the eyes during testing. c. The new nurse asks the patient if the instrument feels sharp. d. The new nurse uses an irregular pattern to test for intact touch.

ANS: C When performing a sensory assessment, the nurse should not provide verbal clues. The other actions by the new nurse are appropriate. DIF: Cognitive Level: Apply (application) REF: 1306 OBJ: Special Questions: Delegation TOP: Nursing Process: Evaluation MSC: NCLEX: Safe and Effective Care Environment

In developing a care plan for a patient with an open reduction and internal fixation (ORIF) of an open, displaced fracture of the tibia, the priority nursing diagnosis is a. activity intolerance related to deconditioning. b. risk for constipation related to prolonged bed rest. c. risk for impaired skin integrity related to immobility. d. risk for infection related to disruption of skin integrity.

ANS: D A patient having an ORIF is at risk for problems such as wound infection and osteomyelitis. After an ORIF, patients typically are mobilized starting the first postoperative day, so problems caused by immobility are not as likely.

A 76-year-old patient is being treated with carbidopa/levodopa (Sinemet) for Parkinson's disease. Which information is most important for the nurse to report to the health care provider? a. Shuffling gait b. Tremor at rest c. Cogwheel rigidity of limbs d. Uncontrolled head movement

ANS: D Dyskinesia is an adverse effect of the Sinemet, indicating a need for a change in medication or decrease in dose. The other findings are typical with Parkinson's disease. DIF: Cognitive Level: Apply (application) REF: 1435 OBJ: Special Questions: Prioritization TOP: Nursing Process: Planning MSC: NCLEX: Physiological Integrity

The care plan for a patient who has increased intracranial pressure and a ventriculostomy includes the following nursing actions. Which action can the nurse delegate to nursing assistive personnel (NAP) who regularly work in the intensive care unit? a. Monitor cerebrospinal fluid color hourly. b. Document intracranial pressure every hour. c. Turn and reposition the patient every 2 hours. d. Check capillary blood glucose level every 6 hours.

ANS: D Experienced NAP can obtain capillary blood glucose levels when they have been trained and evaluated in the skill. Monitoring and documentation of cerebrospinal fluid (CSF) color and intracranial pressure (ICP) require RN-level education and scope of practice. Although repositioning patients is frequently delegated to NAP, repositioning a patient with a ventriculostomy is complex and should be done by the RN. DIF: Cognitive Level: Application REF: 1442

A patient with suspected meningitis is scheduled for a lumbar puncture. Before the procedure, the nurse will plan to a. enforce NPO status for 4 hours. b. transfer the patient to radiology. c. administer a sedative medication. d. help the patient to a lateral position.

ANS: D For a lumbar puncture, the patient lies in the lateral recumbent position. The procedure does not usually require a sedative, is done in the patient room, and has no risk for aspiration

A patient with suspected meningitis is scheduled for a lumbar puncture. Before the procedure, the nurse will plan to a. enforce NPO status for 4 hours. b. transfer the patient to radiology. c. administer a sedative medication. d. help the patient to a lateral position.

ANS: D For a lumbar puncture, the patient lies in the lateral recumbent position. The procedure does not usually require a sedative, is done in the patient room, and has no risk for aspiration. DIF: Cognitive Level: Apply (application) REF: 1310 TOP: Nursing Process: Planning MSC: NCLEX: Physiological Integrity

A 62-yr-old patient who has Parkinson's disease is taking bromocriptine (Parlodel). Which information obtained by the nurse may indicate a need for a decrease in the dosage? a. The patient has a chronic dry cough. b. The patient has four loose stools in a day. c. The patient develops a deep vein thrombosis. d. The patient's blood pressure is 92/52 mm Hg.

ANS: D Hypotension is an adverse effect of bromocriptine, and the nurse should check with the health care provider before giving the medication. Diarrhea, cough, and deep vein thrombosis are not associated with bromocriptine use. DIF: Cognitive Level: Apply (application) REF: 1390 TOP: Nursing Process: Evaluation MSC: NCLEX: Physiological Integrity

A 62-year-old patient who has Parkinson's disease is taking bromocriptine (Parlodel). Which information obtained by the nurse may indicate a need for a decrease in the dose? a. The patient has a chronic dry cough. b. The patient has four loose stools in a day. c. The patient develops a deep vein thrombosis. d. The patient's blood pressure is 92/52 mm Hg.

ANS: D Hypotension is an adverse effect of bromocriptine, and the nurse should check with the health care provider before giving the medication. Diarrhea, cough, and deep vein thrombosis are not associated with bromocriptine use. DIF: Cognitive Level: Apply (application) REF: 1435 TOP: Nursing Process: Evaluation MSC: NCLEX: Physiological Integrity

When giving home care instructions to a patient who has multiple forearm fractures and a long-arm cast on the right arm, which information should the nurse include? a. Keep the hand immobile to prevent soft tissue swelling. b. Keep the right shoulder elevated on a pillow or cushion. c. Avoid the use of nonsteroidal anti-inflammatory drugs (NSAIDs) for the first 48 hours after the injury. d. Call the health care provider for increased swelling or numbness.

ANS: D Increased swelling or numbness may indicate increased pressure at the injury, and the health care provider should be notified immediately to avoid damage to nerves and other tissues. The patient should be encouraged to move the joints above and below the cast to avoid stiffness. There is no need to elevate the shoulder, although the forearm should be elevated to reduce swelling. NSAIDs are appropriate to treat pain after a fracture.

Which parameter is best for the nurse to monitor to determine whether the prescribed IV mannitol (Osmitrol) has been effective for an unconscious patient? a. Hematocrit b. Blood pressure c. Oxygen saturation d. Intracranial pressure

ANS: D Mannitol is an osmotic diuretic and will reduce cerebral edema and intracranial pressure. It may initially reduce hematocrit and increase blood pressure, but these are not the best parameters for evaluation of the effectiveness of the drug. Oxygen saturation will not directly improve as a result of mannitol administration. DIF: Cognitive Level: Application REF: 1432-1433

Which information about a 72-year-old patient who has a new prescription for phenytoin (Dilantin) indicates that the nurse should consult with the health care provider before administration of the medication? a. Patient has generalized tonic-clonic seizures. b. Patient experiences an aura before seizures. c. Patient's most recent blood pressure is 156/92 mm Hg. d. Patient has minor elevations in the liver function tests.

ANS: D Many older patients (especially with compromised liver function) may not be able to metabolize phenytoin. The health care provider may need to choose another antiseizure medication. Phenytoin is an appropriate medication for patients with tonic-clonic seizures, with or without an aura. Hypertension is not a contraindication for phenytoin therapy. DIF: Cognitive Level: Apply (application) REF: 1424 TOP: Nursing Process: Implementation MSC: NCLEX: Physiological Integrity

Which assessment finding in a patient who was admitted the previous day with a basilar skull fracture is most important to report to the health care provider? a. Bruising under both eyes b. Complaint of severe headache c. Large ecchymosis behind one ear d. Temperature of 101.5° F (38.6° C)

ANS: D Patients who have basilar skull fractures are at risk for meningitis, so the elevated temperature should be reported to the health care provider. The other findings are typical of a patient with a basilar skull fracture. DIF: Cognitive Level: Application REF: 1440

After having a craniectomy and left anterior fossae incision, a patient has a nursing diagnosis of impaired physical mobility related to decreased level of consciousness and weakness. An appropriate nursing intervention is to a. position the bed flat and log roll the patient. b. cluster nursing activities to allow longer rest periods. c. turn and reposition the patient side to side every 2 hours. d. perform range-of-motion (ROM) exercises every 4 hours.

ANS: D ROM exercises will help to prevent the complications of immobility. Patients with anterior craniotomies are positioned with the head elevated. The patient with a craniectomy should not be turned to the operative side. When the patient is weak, clustering nursing activities may lead to more fatigue and weakness. DIF: Cognitive Level: Application REF: 1450-1451

The nurse will anticipate teaching a patient with a possible seizure disorder about which test? a. Cerebral angiography b. Evoked potential studies c. Electromyography (EMG) d. Electroencephalography (EEG)

ANS: D Seizure disorders are usually assessed using EEG testing. Evoked potential is used for diagnosing problems with the visual or auditory systems. Cerebral angiography is used to diagnose vascular problems. EMG is used to evaluate electrical innervation to skeletal muscle

The nurse will anticipate teaching a patient with a possible seizure disorder about which test? a. Cerebral angiography b. Evoked potential studies c. Electromyography (EMG) d. Electroencephalography (EEG)

ANS: D Seizure disorders are usually assessed using EEG testing. Evoked potential is used to diagnose problems with the visual or auditory systems. Cerebral angiography is used to diagnose vascular problems. EMG is used to evaluate electrical innervation to skeletal muscle. DIF: Cognitive Level: Understand (comprehension) REF: 1311 TOP: Nursing Process: Planning MSC: NCLEX: Physiological Integrity

When assessing a patient with bacterial meningitis, the nurse obtains the following data. Which finding should be reported immediately to the health care provider? a. The patient has a positive Kernig's sign. b. The patient complains of having a stiff neck. c. The patient's temperature is 101° F (38.3° C). d. The patient's blood pressure is 86/42 mm Hg.

ANS: D Shock is a serious complication of meningitis, and the patient's low blood pressure indicates the need for interventions such as fluids or vasopressors. Nuchal rigidity and a positive Kernig's sign are expected with bacterial meningitis. The nurse should intervene to lower the temperature, but this is not as life threatening as the hypotension. DIF: Cognitive Level: Application REF: 1452-1453

Which nursing diagnosis is of highest priority for a patient with Parkinson's disease who is unable to move the facial muscles? a. Activity intolerance b. Self-care deficit: toileting c. Ineffective self-health management d. Imbalanced nutrition: less than body requirements

ANS: D The data about the patient indicate that poor nutrition will be a concern because of decreased swallowing. The other diagnoses may also be appropriate for a patient with Parkinson's disease, but the data do not indicate that they are current problems for this patient. DIF: Cognitive Level: Analyze (analysis) REF: 1391 OBJ: Special Questions: Prioritization TOP: Nursing Process: Analysis MSC: NCLEX: Physiological Integrity

hich nursing diagnosis is of highest priority for a patient with Parkinson's disease who is unable to move the facial muscles? a. Activity intolerance b. Self-care deficit: toileting c. Ineffective self-health management d. Imbalanced nutrition: less than body requirements

ANS: D The data about the patient indicate that poor nutrition will be a concern because of decreased swallowing. The other diagnoses may also be appropriate for a patient with Parkinson's disease, but the data do not indicate that they are current problems for this patient. DIF: Cognitive Level: Apply (application) REF: 1436 OBJ: Special Questions: Prioritization TOP: Nursing Process: Analysis MSC: NCLEX: Physiological Integrity

After the emergency department nurse has received a status report on the following patients who have been admitted with head injuries, which patient should the nurse assess first? a. A patient whose cranial x-ray shows a linear skull fracture b. A patient who has an initial Glasgow Coma Scale score of 13 c. A patient who lost consciousness for a few seconds after a fall d. A patient whose right pupil is 10 mm and unresponsive to light

ANS: D The dilated and nonresponsive pupil may indicate an intracerebral hemorrhage and increased intracranial pressure. The other patients are not at immediate risk for complications such as herniation. DIF: Cognitive Level: Analysis REF: 1432-1433 | 1437-1438

A patient is admitted to the emergency department with possible left lower leg fractures. The initial action by the nurse should be to a. elevate the left leg. b. splint the lower leg. c. obtain information about the tetanus immunization status. d. check the popliteal, dorsalis pedis, and posterior tibial pulses.

ANS: D The initial nursing action should be assessment of the neurovascular status of the injured leg. After assessment, the nurse may need to splint and elevate the leg, based on the assessment data. Information about tetanus immunizations should be done if there is an open wound.

A patient with a comminuted fracture of the right femur has Buck's traction in place while waiting for surgery. To assess for pressure areas on the patient's back and sacral area and to provide skin care, the nurse should a. loosen the traction and have the patient turn onto the unaffected side. b. place a pillow between the patient's legs and turn gently to each side. c. turn the patient partially to each side with the assistance of another nurse. d. have the patient lift the buttocks by bending and pushing with the left leg.

ANS: D The patient can lift the buttocks off the bed by using the left leg without changing the right-leg alignment. Turning the patient will tend to move the leg out of alignment. Disconnecting the traction will interrupt the weight needed to immobilize and align the fracture.

The second day after admission with a fractured pelvis, a patient develops acute onset confusion. Which action should the nurse take first? a. Take the blood pressure. b. Assess patient orientation. c. Check pupil reaction to light. d. Assess the oxygen saturation.

ANS: D The patient's history and clinical manifestations suggest a fat embolus. The most important assessment is oxygenation. The other actions also are appropriate but will be done after the nurse assesses gas exchange.

The nurse performing a focused assessment of left posterior temporal lobe functions will assess the patient for a. sensation on the left side of the body. b. voluntary movements on the right side. c. reasoning and problem-solving abilities. d. understanding written and oral language.

ANS: D The posterior temporal lobe integrates the visual and auditory input for language comprehension. Reasoning and problem solving are functions of the anterior frontal lobe. Sensation on the left side of the body is located in the right postcentral gyrus. Voluntary movement on the right side is controlled in the left precentral gyrus.

A patient who is suspected of having an epidural hematoma is admitted to the emergency department. Which action will the nurse plan to take? a. Administer IV furosemide (Lasix). b. Initiate high-dose barbiturate therapy. c. Type and crossmatch for blood transfusion. d. Prepare the patient for immediate craniotomy.

ANS: D The principal treatment for epidural hematoma is rapid surgery to remove the hematoma and prevent herniation. If intracranial pressure (ICP) is elevated after surgery, furosemide or high-dose barbiturate therapy may be needed, but these will not be of benefit unless the hematoma is removed. Minimal blood loss occurs with head injuries, and transfusion is usually not necessary. DIF: Cognitive Level: Application REF: 1440-1441

Which prescribed intervention will the nurse implement first for a patient in the emergency department who is experiencing continuous tonic-clonic seizures? a. Give phenytoin (Dilantin) 100 mg IV. b. Monitor level of consciousness (LOC). c. Obtain computed tomography (CT) scan. d. Administer lorazepam (Ativan) 4 mg IV.

ANS: D To prevent ongoing seizures, the nurse should administer rapidly acting antiseizure medications such as the benzodiazepines. A CT scan is appropriate, but prevention of any seizure activity during the CT scan is necessary. Phenytoin will also be administered, but it is not rapidly acting. Patients who are experiencing tonic-clonic seizures are nonresponsive, although the nurse should assess LOC after the seizure. DIF: Cognitive Level: Apply (application) REF: 1424 OBJ: Special Questions: Prioritization TOP: Nursing Process: Implementation MSC: NCLEX: Physiological Integrity

The priority nursing assessment for a 72-year-old patient being admitted with a brainstem infarction is a. reflex reaction time. b. pupil reaction to light. c. level of consciousness. d. respiratory rate and rhythm.

ANS: D Vital centers that control respiration are located in the medulla, and these are the priority assessments because changes in respiratory function may be life threatening. The other information will also be collected by the nurse, but it is not as urgent

Cranial nerve VI

Abducens-connection with pons (motor) lateral rectus of the eye

Cranial nerve XI

Accessory; neck and shoulder movement

Increased ICP symptoms

Altered LOC Headache Slowing pulse Rising BP with widening pulse pressure Elevated temp Vomiting Pupil changes

A patient with amyotrophic lateral sclerosis (ALS) is hospitalized with pneumonia. Which nursing action will be included in the plan of care?

Assist with active range of motion. rational: ALS causes progressive muscle weakness, but assisting the patient to perform active ROM will help to maintain strength as long as possible. Psychotic symptoms such as agitation and paranoia are not associated with ALS. Cognitive function is not affected by ALS, and the patient's ability to understand procedures will not be impaired. Muscle relaxants will further increase muscle weakness and depress respirations.

The nurse cares for a 34-year-old woman after a lumbar puncture. Which action by the nurse is most appropriate? A. Assess for drainage or bleeding from the puncture site. B. Monitor for bladder dysfunction and bowel incontinence. C. Maintain bed rest until lower extremities move normally. D. Check for loss of muscle strength in the upper extremities.

Answer: A After a lumbar puncture the nurse should monitor the puncture site for drainage or bleeding. Other assessments include headache intensity, meningeal irritation (nuchal rigidity), signs and symptoms of local trauma (e.g., hematoma, pain), neurologic signs, and vital signs. A lumbar puncture does not affect bowel or bladder function or upper extremity muscle strength. Bed rest until lower extremity movement returns is indicated for the patient after spinal anesthesia. Awarded 1.0 points out of 1.0 possible points.

The nurse is completing a health assessment for an obese 62-year-old man who wants to begin a diet and exercise program. Which assessment should the nurse perform to determine the cognitive function of the patient during the physical examination? A. Ask the patient a question such as, "Who were the last three presidents?" B. Determine the level of consciousness, body posture, and facial expressions. C. Observe for signs of agitation, anger, or depression during the health check. D. Request that the patient mimic rapid, alternating movements with both hands.

Answer: A Cognition is one component of the mental status examination to determine cerebral functioning. Cognition is assessed by determining orientation, memory, general knowledge, insight, judgment, problem solving, and calculation. A question often used to determine cognition for adults living in the United States is, "Who were the last three presidents?" General appearance and behavior is another component and includes level of consciousness, body posture, and facial expressions. Mood and affect are assessed by observing for agitation, anger, or depression. Cerebellar function is determined by assessing balance and coordination and may include testing rapid alternating movements of the upper and lower extremities.

A patient with heart failure and type 1 diabetes mellitus is scheduled for a positron emission tomography (PET) of the brain. Which medication prescribed by the health care provider should the nurse expect to administer before the diagnostic study? A. Regular insulin 6 units (SQ) B. Furosemide (Lasix) 20 mg (IV) C. Alprazolam (Xanax) 0.5 mg (PO) D. Ciprofloxacin (Cipro) 500 mg (PO)

Answer: A Patients with type 1 diabetes mellitus must have insulin administered the day of the PET study if glucose metabolism is the focus of the PET. Diuretics should not be administered before the PET scan unless a urinary catheter is inserted. The patient must remain still during the procedure (1 to 2 hours). Sedatives and tranquilizers (e.g., alprazolam) should not be administered before a PET study of the brain because the patient may need to perform mental activities and these medications may affect glucose metabolism. Prophylactic antibiotics are not necessary. Patients are NPO before a PET study of the brain and should not receive oral medications (alprazolam and ciprofloxacin)

In which patient would it be the most important for the nurse to assess the glossopharyngeal and vagus nerves? A. A 50-year-old woman with lethargy from a drug overdose B. A 40-year-old man with a complete lumbar spinal cord injury C. A 60-year-old man with severe pain from trigeminal neuralgia D. A 30-year-old woman with a high fever and bacterial meningitis

Answer: A The glossopharyngeal and vagus nerves innervate the pharynx and are tested by the gag reflex. It is important to assess the gag reflex in patients who have a decreased level of consciousness, a brainstem lesion, or a disease involving the throat musculature. If the reflex is weak or absent, the patient is in danger of aspirating food or secretions.

The nurse is caring for a group of well older people at a community day center. Which neurologic finding associated with aging would the nurse expect to find in older adults? A. Longer reaction time B. Improved sense of taste C. Orthostatic hypotension D. Hyperactive deep tendon reflexes

Answer: C Older adults are more likely to experience orthostatic hypotension related to altered coordination of neuromuscular activity. Other neurologic changes in the older adult include atrophy of taste buds with decreased sense of taste, below average reflex score (and diminished deep tendon reflexes), and slowed reaction times.

Beta-Interferon (Avenox, Betaseron, Rebif)

Anti multiple sclerosis agent Reduce incidnece of relapse & slow physical disability Avoid echinacea & melatonin

Topiramate (Topamax)

Anticonvulsant / mood stabilizer Partial onset & tonic-clonic/ prevents migraine headache

Carbamazepine (Tegretol)

Anticonvulsant/mood stabilizer prevents seizures, trigeminal neuralgia NO GRAPEFRUIT

Tensilon Test Antidote

Atropine sulfate

Temporal Lobe

Auditory center Wernicke's area for sensory and speech

The ability of the brain to maintain relatively constant blood flow despite changes in perfusion pressure by adjusting the diameter of blood vessels

Autoregulation (of cerebral blood flow)

When assessing the accessory nerve, what should the nurse do? A. Assess the gag reflex by stroking the posterior pharynx. B. Ask the patient to shrug the shoulders against resistance. C. Ask the patient to push the tongue to either side against resistance. D. Have the patient say "ah" while visualizing elevation of soft palate.

B The spinal accessory nerve is tested by asking the patient to shrug the shoulders against resistance and to turn the head to either side against resistance while observing the sternocleidomastoid muscles and the trapezius muscles. Assessing the gag reflex and saying "ah" are used to assess the glossopharyngeal and vagus nerves. Asking the patient to push the tongue to either side against resistance and to stick out the tongue are used to assess the hypoglossal nerve

While completing an admission history for a 73-year-old man with osteoarthritis admitted for knee arthroplasty, the nurse asks about the patient's perception of the reason for admission. The nurse expects the patient to relate which response to this question? A Recent knee trauma B Debilitating joint pain C Repeated knee infections D Onset of "frozen" knee joint

B Debilitating joint pain The most common reason for knee arthroplasty is debilitating joint pain despite attempts to manage it with exercise and drug therapy. Recent knee trauma, repeated knee infections, and onset of "frozen" knee joint are not primary indicators for a knee arthroplasty.

When establishing a diagnosis of MS, the nurse should teach the patient about what diagnostic studies (select all that apply)? A. EEG B. CT scan C. Carotid duplex scan D. Evoked response testing E. Cerebrospinal fluid analysis

B, D, E There is no definitive diagnostic test for MS. CT scan, evoked response testing, cerebrospinal fluid analysis, and MRI along with history and physical examination are used to establish a diagnosis for MS. EEG and carotid duplex scan are not used for diagnosing MS.

Which normal nervous system changes of aging put the geriatric person at higher risk of falls (select all that apply)? A. Memory deficit B. Sensory deficit C. Motor function deficit D. Cranial and spinal nerves E. Reticular activation system F. Central nervous system changes

B,C,F An older person is at a higher risk for falls because the changes in the nervous system decrease the sensory function that leads to poor ability to maintain balance and a widened gait. The motor function deficit decreases muscle strength and agility. The central nervous system changes in the brain lead to a diminished kinesthetic sense or position sense. Memory deficits, normal changes of cranial and spinal nerves, and the reticular activation system do not contribute to the increased risk of falls.

The nurse provides information to the caregiver of a 68-year-old man with epilepsy who has tonic-clonic seizures. Which statement, if made by the caregiver, requires further teaching? A. "It is normal for a person to be sleepy after a seizure." B. "I should call 911 if breathing stops during the seizure." C. "The jerking movements may last for 30 to 40 seconds." D. "Objects should not be placed in the mouth during a seizure."

B. Caregivers do not need to call an ambulance or send a person to the hospital after a single seizure unless the seizure is prolonged, another seizure immediately follows, or extensive injury has occurred. Altered breathing is a clinical manifestation of a tonic-clonic seizure. Contact emergency medical services (or call 911) if breathing stops for more than 30 seconds. No objects (e.g., oral airway, padded tongue blade) should be placed in the mouth. Lethargy is common in the postictal phase of a seizure. Jerking of the extremities occurs during the clonic phase of a tonic-clonic seizure. The clonic phase may last 30 to 40 seconds.

A patient has been receiving scheduled doses of phenytoin (Dilantin) and begins to experience diplopia. The nurse immediately assesses the patient for A. an aura or focal seizure. B. nystagmus or confusion. C. abdominal pain or cramping. D. irregular pulse or palpitations.

B. Diplopia is a sign of phenytoin toxicity. The nurse should assess for other signs of toxicity, which include neurologic changes, such as nystagmus, ataxia, confusion, dizziness, or slurred speech. An aura, focal seizure, abdominal pain or cramping, irregular pulse, or palpitations are not associated with phenytoin toxicity.

The nurse observes a 74-year-old man with Parkinson's disease rocking side to side while sitting in the chair. Which action by the nurse is most appropriate? A. Provide the patient with diversional activities. B. Document the activity in the patient's health record. C. Take the patient's blood pressure sitting and standing. D. Ask if the patient is feeling either anxious or depressed.

B. Patients with Parkinson's disease are instructed to rock from side to side to stimulate balance mechanisms and decrease akinesia.

A 50-year-old male patient has been diagnosed with amyotrophic lateral sclerosis (ALS). What nursing intervention is most important to help prevent a common cause of death for patients with ALS? A. Reduce fat intake. B. Reduce the risk of aspiration. C. Decrease injury related to falls. D. Decrease pain secondary to muscle weakness.

B. Reducing the risk of aspiration can help prevent respiratory infections that are a common cause of death from deteriorating muscle function. Reducing fat intake may reduce cardiovascular disease, but this is not a common cause of death for patients with ALS. Decreasing injury related to falls and decreasing pain secondary to muscle weakness are important nursing interventions for patients with ALS but are unrelated to causes of death for these patients.

The home health registered nurse (RN) is planning care for a patient with a seizure disorder related to a recent head injury. Which nursing action can be delegated to a licensed practical/vocational nurse (LPN/LVN)? a. Make referrals to appropriate community agencies. b. Place medications in the home medication organizer. c. Teach the patient and family how to manage seizures. d. Assess for use of medications that may precipitate seizures.

B. LPN/LVN education includes administration of medications. The other activities require RN education and scope of practice.

A 73-year-old patient with Parkinson's disease has a nursing diagnosis of impaired physical mobility related to bradykinesia. Which action will the nurse include in the plan of care? a. Instruct the patient in activities that can be done while lying or sitting. b. Suggest that the patient rock from side to side to initiate leg movement. c. Have the patient take small steps in a straight line directly in front of the feet. d. Teach the patient to keep the feet in contact with the floor and slide them forward.

B. Rocking the body from side to side stimulates balance and improves mobility. The patient will be encouraged to continue exercising because this will maintain functional abilities. Maintaining a wide base of support will help with balance. The patient should lift the feet and avoid a shuffling gait.

How should the nurse most accurately assess the position sense of a patient with a recent traumatic brain injury? A. Ask the patient to close his or her eyes and slowly bring the tips of the index fingers together. B. Ask the patient to stand with the feet together and eyes closed and observe for balance maintainance. C. Ask the patient to close his or her eyes and identify the presence of a common object on the forearm. D. Place the two points of a calibrated compass on the tips of the fingers and toes and ask the patient to discriminate the points.

B. The Romberg test is an assessment of position sense in which the patient stands with the feet together and then closes his or her eyes while attempting to maintain balance. The other cited tests of neurologic function do not directly assess position sense.

When a 74-year-old patient is seen in the health clinic with new development of a stooped posture, shuffling gait, and pill rolling-type tremor, the nurse will anticipate teaching the patient about a. oral corticosteroids. b. antiparkinsonian drugs. c. magnetic resonance imaging (MRI). d. electroencephalogram (EEG) testing.

B. The diagnosis of Parkinson's is made when two of the three characteristic manifestations of tremor, rigidity, and bradykinesia are present. The confirmation of the diagnosis is made on the basis of improvement when antiparkinsonian drugs are administered. This patient has symptoms of tremor and bradykinesia. The next anticipated step will be treatment with medications. MRI and EEG are not useful in diagnosing Parkinson's disease, and corticosteroid therapy is not used to treat it.

Which characteristic of a patient's recent seizure is consistent with a focal seizure? A. The patient lost consciousness during the seizure. B. The seizure involved lip smacking and repetitive movements. C. The patient fell to the ground and became stiff for 20 seconds. D. The etiology of the seizure involved both sides of the patient's brain.

B. The most common complex focal seizure involves lip smacking and automatisms (repetitive movements that may not be appropriate). Loss of consciousness, bilateral brain involvement, and a tonic phase are associated with generalized seizure activity.

A high school teacher who has just been diagnosed with epilepsy after having a generalized tonic-clonic seizure tells the nurse, "I cannot teach anymore, it will be too upsetting if I have a seizure at work." Which response by the nurse is best? a. "You might benefit from some psychologic counseling." b. "Epilepsy usually can be well controlled with medications." c. "You will want to contact the Epilepsy Foundation for assistance." d. "The Department of Vocational Rehabilitation can help with work retraining."

B. The nurse should inform the patient that most patients with seizure disorders are controlled with medication. The other information may be necessary if the seizures persist after treatment with antiseizure medications is implemented.

Following a thymectomy, a 62-year-old male patient with myasthenia gravis receives the usual dose of pyridostigmine (Mestinon). An hour later, the patient complains of nausea and severe abdominal cramps. Which action should the nurse take first? a. Auscultate the patient's bowel sounds. b. Notify the patient's health care provider. c. Administer the prescribed PRN antiemetic drug. d. Give the scheduled dose of prednisone (Deltasone).

B. The patient's history and symptoms indicate a possible cholinergic crisis. The health care provider should be notified immediately, and it is likely that atropine will be prescribed. The other actions will be appropriate if the patient is not experiencing a cholinergic crisis.

A 57-year-old postmenopausal woman is scheduled for dual-energy x-ray absorptiometry (DXA). Which statement, if made by the patient to the nurse, indicates understanding of the procedure? A. "The bone density in my heel will be measured." B. "This procedure will not cause any pain or discomfort." C. "I will not be exposed to any radiation during the procedure." D. "I will need to remove my hearing aids before the procedure."

B. "This procedure will not cause any pain or discomfort." Dual-energy x-ray absorptiometry (DXA) is painless and measures the bone mass of spine, femur, forearm, and total body with minimal radiation exposure. A quantitative ultrasound (QUS) evaluates density, elasticity, and strength of bone using ultrasound of the calcaneus (heel). Magnetic resonance imaging would require removal of objects such as hearing aids that have metal parts.

A patient with an intracranial problem does not open his eyes to any stimulus, has no verbal response except moaning and muttering when stimulated, and flexes his arm in response to painful stimuli. The nurse records the patients GCS score as a. 6 b. 7 c. 9 d. 11

B. 7- no opening of eyes = 1; incomprehensible words= 2, flexion withdrawal = 4 Total = 7

Which information in a 67-year-old woman's health history will alert the nurse to the need for a more focused assessment of the musculoskeletal system? a. The patient sprained her ankle at age 13. b. The patient's mother became shorter with aging. c. The patient takes ibuprofen (Advil) for occasional headaches. d. The patient's father died of complications of miliary tuberculosis.

B. A family history of height loss with aging may indicate osteoporosis, and the nurse should perform a more thorough assessment of the patient's current height and other risk factors for osteoporosis. A sprained ankle during adolescence does not place the patient at increased current risk for musculoskeletal problems. A family history of tuberculosis is not a risk factor. Occasional nonsteroidal antiinflammatory drug (NSAID) use does not indicate any increased musculoskeletal risk.

During a health screening event which assessment finding would alert the nurse to the possible presence of osteoporosis in a white 61-year-old female? A. The presence of bowed legs B. A measurable loss of height C. Poor appetite and aversion to dairy products D. Development of unstable, wide-gait ambulation

B. A measurable loss of height A gradual but measurable loss of height and the development of kyphosis or "dowager's hump" are indicative of the presence of osteoporosis in which the rate of bone resorption is greater than bone deposition. Bowed legs may be caused by abnormal bone development or rickets but is not indicative of osteoporosis. Lack of calcium and Vitamin D intake may cause osteoporosis but are not indicative it is present. A wide gait is used to support balance and does not indicate osteoporosis.

Vasogenic cerebral edema increases ICP by a. shifting fluid in gray matter b. altering the endothelial lining of cerebral capillaries c. leaking molecules from the intracellular fluid to the capillaries d. altering the osmotic gradient flow inot the intravascular component

B. Altering the endothelial lining of cerebral capillaries

A female patient with a long-standing history of rheumatoid arthritis has sought care because of increasing stiffness in her right knee that has culminated in complete fixation of the joint. The nurse would document the presence of which problem? A. Atrophy B. Ankylosis C. Crepitation D. Contracture

B. Ankylosis Ankylosis is stiffness or fixation of a joint, whereas contracture is reduced movement as a consequence of fibrosis of soft tissue (muscles, ligaments, or tendons). Atrophy is a flabby appearance of muscle leading to decreased function and tone. Crepitation is a grating or crackling sound that accompanies movement.

A nursing measure that is indicated to reduce the potential for seizures and increased ICP in the patient with bacterial meningitis is a. administering codeine for relief of head and neck pain b. controlling fever with prescribed drugs and cooling techniques c. keeping the room darkened and quiet to minimize environmental stimulation d. maintaining the patient on strict bed rest with the HOB slightly elevated

B. Controlling fever with prescribed drugs and cooling techniques

The nurse recognizes the presence of Cushing's triad in the patient with a. Increased pulse, irregular respiration, increased BP b. decreased pulse, irregular respiration, increased pulse pressure c. increased pulse, decreased respiration, increased pulse pressure d. decreased pulse, increased respiration, decreased systolic BP

B. Cushing's triad consists of three vital sign measures that reflect ICP and its effect on the medulla, the hypothalamus, the pons, and the thalamus. Because these structures are very deep, Cushing's triad is usually a late sign of ICP. The signs include an increasing systolic BP with a widening pulse pressure, a bradycardia with a full and bounding pulse, and irregular respirations.

Which action can the nurse delegate to unlicensed assistive personnel (UAP) who are working in the orthopedic clinic? a. Grade leg muscle strength for a patient with back pain. b. Obtain blood sample for uric acid from a patient with gout. c. Perform straight-leg-raise testing for a patient with sciatica. d. Check for knee joint crepitation before arthroscopic surgery.

B. Drawing blood specimens is a common skill performed by UAP in clinic settings. The other actions are assessments and require registered nurse (RN)-level judgment and critical thinking.

Assisting the family to understand what is happening to the patient is an especially important role of the nurse when the patient has a tumor of the a. ventricles b. frontal lobe c. parietal lobe d. occipital lobe

B. Frontal lobe- frontal lobe tumors often lead to loss of emotional control, confusion, memory loss, disorientation, and personality changes that are very disturbing and frightening to the family. Physical symptoms, such as blindness, disturbances in sensation and perception, and even seizures, that occur with other tumors are more likely to be understood and accepted by the family

Cranial nerve IX

Glossopharyngeal; taste in posterior one-third of tongue, sensation of pharynx and tongue, gag reflex, swallowing, secretions of parotid gland

Classic symptoms of bacterial meningitis include a. papilledema and psychomotor seizures b. high fever, nuchal rigidity, and severe headache c. behavioral changes with memory loss and lethargy d. positive Kernig's and Brudzinski's signs and hemiparesis

B. High fever, severe headache, nuchal rigidity, and positive Brudzinski's and Kernig's signs are such classic symptoms of meningitis that they are usually considered diagnostic for meningitis. Other symptoms, such as papilledema, generalized seizures, hemiparesis, and decreased LOC, may occur as complications of increased ICP and cranial nerve dysfunction.

The nurse is alerted to possible acute subdural hematoma in the patient who a. has a linear skull fracture crossing a major artery b. has focal symptoms of brain damage with no recollection of a head injury c. develops decreased LOC and a headache within 48 hours of head injury d. has an immediate loss of consciousness with a brief lucid interval followed by decreasing LOC

C. develops decreased LOC and a headache within 48 hours of head injury

The home health registered nurse (RN) is planning care for a patient with a seizure disorder related to a recent head injury. Which nursing action can be delegated to a licensed practical/vocational nurse (LPN/LVN)? a. Make referrals to appropriate community agencies. b. Place medications in the home medication organizer. c. Teach the patient and family how to manage seizures. d. Assess for use of medications that may precipitate seizures.

B. LPN/LVN education includes administration of medications. The other activities require RN education and scope of practice.

The nurse is caring for a patient admitted to the nursing unit with osteomyelitis of the tibia. Which symptom will the nurse most likely find on physical examination of the patient? A. Nausea and vomiting B. Localized pain and warmth C. Paresthesia in the affected extremity D. Generalized bone pain throughout the leg

B. Localized pain and warmth Osteomyelitis is an infection of bone and bone marrow that can occur with trauma, surgery, or spread from another part of the body. Because it is an infection, the patient will exhibit typical signs of inflammation and infection, including localized pain and warmth. Nausea and vomiting and paresthesia of the extremity are not expected to occur. Pain occurs, but it is localized, not generalized throughout the leg.

Which intervention will the nurse include in the plan of care for a patient with primary restless legs syndrome (RLS) who is having difficulty sleeping? a. Teach about the use of antihistamines to improve sleep. b. Suggest that the patient exercise regularly during the day. c. Make a referral to a massage therapist for deep massage of the legs. d. Assure the patient that the problem is transient and likely to resolve.

B. Nondrug interventions such as getting regular exercise are initially suggested to improve sleep quality in patients with RLS. Antihistamines may aggravate RLS. Massage does not alleviate RLS symptoms and RLS is likely to progress in most patients.

In reviewing bone remodeling, what should the nurse know about the involvement of bone cells? A. Osteoclasts add canaliculi. B. Osteoblasts deposit new bone. C. Osteocytes are mature bone cells. D. Osteons create a dense bone structure.

B. Osteoblasts deposit new bone. Bone remodeling is achieved when osteoclasts remove old bone and osteoblasts deposit new bone. Osteocytes are mature bone cells, and osteons or Haversian systems create a dense bone structure, but these are not involved with bone remodeling.

3 classifications of brain tumors

benign malignant (rarely metastasize outside CNS) metastatic

A 73-year-old patient with Parkinson's disease has a nursing diagnosis of impaired physical mobility related to bradykinesia. Which action will the nurse include in the plan of care? a. Instruct the patient in activities that can be done while lying or sitting. b. Suggest that the patient rock from side to side to initiate leg movement. c. Have the patient take small steps in a straight line directly in front of the feet. d. Teach the patient to keep the feet in contact with the floor and slide them forward.

B. Rocking the body from side to side stimulates balance and improves mobility. The patient will be encouraged to continue exercising because this will maintain functional abilities. Maintaining a wide base of support will help with balance. The patient should lift the feet and avoid a shuffling gait.

A patient with a head injury has bloody drainage from the ear. To determine whether CSF is present in the drainage, the nurse a. examines the tympanic membrane for a tear b. tests the fluid for a halo sign on a white dressing c. tests the fluid with a glucose identifying strip or stick d. collects 5 mL of fluid in a test tube and sends it to the laboratory for analysis

B. Tests the fluid for a halo sing on a white dressing- Testing clear drainage for CSF in nasal or ear drainage may be done with a Dextrostik or Tes-Tape strip, but if blood is present, the glucose in the blood will produce and unreliable result. To test bloody drainage, the nurse should test the fluid for a halo or ring that occurs when a yellowish ring encircles blood dripped onto a white pad or towel

Which action will the nurse plan to take for a 40-year-old patient with multiple sclerosis (MS) who has urinary retention caused by a flaccid bladder? a. Decrease the patient's evening fluid intake. b. Teach the patient how to use the Credé method. c. Suggest the use of adult incontinence briefs for nighttime only. d. Assist the patient to the commode every 2 hours during the day.

B. The Credé method can be used to improve bladder emptying. Decreasing fluid intake will not improve bladder emptying and may increase risk for urinary tract infection (UTI) and dehydration. The use of incontinence briefs and frequent toileting will not improve bladder emptying.

When a 74-year-old patient is seen in the health clinic with new development of a stooped posture, shuffling gait, and pill rolling-type tremor, the nurse will anticipate teaching the patient about a. oral corticosteroids. b. antiparkinsonian drugs. c. magnetic resonance imaging (MRI). d. electroencephalogram (EEG) testing.

B. The diagnosis of Parkinson's is made when two of the three characteristic manifestations of tremor, rigidity, and bradykinesia are present. The confirmation of the diagnosis is made on the basis of improvement when antiparkinsonian drugs are administered. This patient has symptoms of tremor and bradykinesia. The next anticipated step will be treatment with medications. MRI and EEG are not useful in diagnosing Parkinson's disease, and corticosteroid therapy is not used to treat it.

A high school teacher who has just been diagnosed with epilepsy after having a generalized tonic-clonic seizure tells the nurse, "I cannot teach anymore, it will be too upsetting if I have a seizure at work." Which response by the nurse is best? a. "You might benefit from some psychologic counseling." b. "Epilepsy usually can be well controlled with medications." c. "You will want to contact the Epilepsy Foundation for assistance." d. "The Department of Vocational Rehabilitation can help with work retraining."

B. The nurse should inform the patient that most patients with seizure disorders are controlled with medication. The other information may be necessary if the seizures persist after treatment with antiseizure medications is implemented.

Following a thymectomy, a 62-year-old male patient with myasthenia gravis receives the usual dose of pyridostigmine (Mestinon). An hour later, the patient complains of nausea and severe abdominal cramps. Which action should the nurse take first? a. Auscultate the patient's bowel sounds. b. Notify the patient's health care provider. c. Administer the prescribed PRN antiemetic drug. d. Give the scheduled dose of prednisone (Deltasone).

B. The patient's history and symptoms indicate a possible cholinergic crisis. The health care provider should be notified immediately, and it is likely that atropine will be prescribed. The other actions will be appropriate if the patient is not experiencing a cholinergic crisis.

A hospitalized patient complains of a bilateral headache, 4/10 on the pain scale, that radiates from the base of the skull. Which prescribed PRN medications should the nurse administer initially? a. Lorazepam (Ativan) b. Acetaminophen (Tylenol) c. Morphine sulfate (Roxanol) d. Butalbital and aspirin (Fiorinal)

B. The patient's symptoms are consistent with a tension headache, and initial therapy usually involves a nonopioid analgesic such as acetaminophen, which is sometimes combined with a sedative or muscle relaxant. Lorazepam may be used in conjunction with acetaminophen but would not be appropriate as the initial monotherapy. Morphine sulfate and butalbital and aspirin would be more appropriate for a headache that did not respond to a nonopioid analgesic.

The health care provider is considering the use of sumatriptan (Imitrex) for a 54-year-old male patient with migraine headaches. Which information obtained by the nurse is most important to report to the health care provider? a. The patient drinks 1 to 2 cups of coffee daily. b. The patient had a recent acute myocardial infarction. c. The patient has had migraine headaches for 30 years. d. The patient has taken topiramate (Topamax) for 2 months.

B. The triptans cause coronary artery vasoconstriction and should be avoided in patients with coronary artery disease. The other information will be reported to the health care provider, but none of it indicates that sumatriptan would be an inappropriate treatment.

The nurse will assess a 67-year-old patient who is experiencing a cluster headache for a. nuchal rigidity. b. unilateral ptosis. c. projectile vomiting. d. throbbing, bilateral facial pain.

B. Unilateral eye edema, tearing, and ptosis are characteristic of cluster headaches. Nuchal rigidity suggests meningeal irritation, such as occurs with meningitis. Although nausea and vomiting may occur with migraine headaches, projectile vomiting is more consistent with increased intracranial pressure (ICP). Unilateral sharp, stabbing pain, rather than throbbing pain, is characteristic of cluster headaches.

When obtaining a health history and physical assessment for a 36-year-old female patient with possible multiple sclerosis (MS), the nurse should a. assess for the presence of chest pain. b. inquire about urinary tract problems. c. inspect the skin for rashes or discoloration. d. ask the patient about any increase in libido.

B. Urinary tract problems with incontinence or retention are common symptoms of MS. Chest pain and skin rashes are not symptoms of MS. A decrease in libido is common with MS.

The nurse plans care for a patient with increased ICP with the knowledge that the best way to position the patient is to a. keep the head of the bed flat b. elevate the head of the bed to 30 degrees c. maintain patient on the left side with the head supported on a pillow d. use a continuous rotation bed to continuously change patient position

B. elevate the head of the bed to 30 degrees

A diagnosis of a ruptured cerebral aneurysm has been made in a patient with manifestations of a stroke. The nurse anticipates that treatment options that would be evaluated for the patient include a. hyperventilation therapy b. surgical clipping of the aneurysm c. administration of hyperosmotic agents d. administration of thrombolytic therapy

B: Surgical clipping of they aneurysm- Surgical management with clipping of an aneurysm to decrease re bleeding and vasospasm is an option for a stroke cause by rupture of a cerebral aneurysm. Placement of coils into the lumens of the aneurysm by intercentional radiologists is increasing in popularity. Hyperventilation therapy would increase vasodilation and the potential for hemorrhage. Thrombolytic therapy would be absolutely contraindicated, and if a vessel is patent, osmotic diuretics may leak into tissue, pulling fluid out of the vessel and increasing edema.

To promote communication during rehabilitation of the patient with aphasia, an appropriate nursing intervention is to a. use gestures, pictures, and music to stimulate patient responses b. talk about activities of daily living (ADLs) that are familiar to the patient c. structure statements so that patient does not have to respond verbally d. use flashcards with simple words and pictures to promote language recall

B: Talk about ADLs that are familiar to the patient- during rehabilitation, the patient with aphasia needs frequent, meaningful verbal stimulation that has relevance for him. Conversation by the nurse and family should address ADLs that are familiar to the patient. Gestures, pictures, and simple statements are more appropriate in the acute phase, when patients may be overwhelmed with verbal stimuli. Flashcards are often perceived by the patient as childish and meaningless.

This has been proven to result in better outcomes for patients on corticosteroid therapy:

BG monitoring at least every 6 hours (Strict glycemic control)

monitor these when administering hypertonic saline for ICP treatment:

BP and serum Na levels

Subarachnoid hemorrhage

Bleeding occurs directly into the brain, ventricles, or subarachnoid space

Frontal Lobe

Broca's are for speech Emotions, reasoning, and judgement

A patient's sudden onset of hemiplegia has necessitated a computed tomography (CT) of her head. Which assessment should the nurse complete prior to this diagnostic study? A. Assess the patient's immunization history. B. Screen the patient for any metal parts or a pacemaker. C. Assess the patient for allergies to shellfish, iodine, or dyes. D. Assess the patient's need for tranquilizers or antiseizure medications.

C Allergies to shellfish, iodine, or dyes contraindicate the use of contrast media in CT. The patient's immunization history is not a central consideration, and the presence of metal in the body does not preclude the use of CT as a diagnostic tool. The need to assess for allergies supersedes the need for tranquilizers or antiseizure medications in the majority of patients.

The new patient has a diagnosis of frontal lobe dementia. What functional difficulties should the nurse expect in this patient? A. The lack of reflexes B. Endocrine problems C. Higher cognitive function abnormalities D. Respiratory, vasomotor, and cardiac dysfunction

C Because the frontal lobe is responsible for higher cognitive function, this patient may have difficulty with memory retention, voluntary eye movements, voluntary motor movement, and expressive speech. The lack of reflexes would occur if the patient had problems with the reflex arcs in the spinal cord. Endocrine problems would be evident if the hypothalamus or pituitary gland were affected. Respiratory, vasomotor, and cardiac dysfunction would occur if there were a problem in the medulla.

The nurse is preparing the patient for an electromyography (EMG). What should the nurse include in teaching the patient before the test? A.The patient will be tilted on a table during the test. B. It is noninvasive, and there is no risk of electric shock. C.The pain that occurs is from the insertion of the needles. D.The passive sensor does not make contact with the patient.

C With an EMG, pain may occur when needles are inserted to record the electrical activity of nerve and skeletal muscle. The patient is tilted on a table during a myelogram. The electroencephalography (EEG) is noninvasive without a danger of electric shock. The magnetoencephalography (MEG) is done with a passive sensor that does not make contact with the patient.

The nurse formulates a nursing diagnosis of impaired physical mobility related to decreased muscle strength for a 78-year-old patient following left total knee replacement. What would be an appropriate nursing intervention for this patient? A Promote vitamin C and calcium intake in the diet. B Provide passive range of motion to all of the joints q4hr. C Encourage isometric quadriceps-setting exercises at least qid. D Keep the left leg in extension and abduction to prevent contractures.

C Encourage isometric quadriceps-setting exercises at least qid. Emphasis is placed on postoperative exercise of the affected leg, with isometric quadriceps setting beginning on the first day after surgery along with a continuous passive motion (CPM) machine. Vitamin C and calcium do not improve muscle strength, but they will facilitate healing. The patient should be able to do active range of motion to all joints. Keeping the leg in one position (extension and abduction) potentially will result in contractures.

The nurse is caring for a patient with osteoarthritis who is about to undergo total left knee arthroplasty. The nurse assesses the patient carefully to be sure that there is no evidence of what in the preoperative period? A Pain B Left knee stiffness C Left knee infection D Left knee instability

C Left knee infection It is critical that the patient be free of infection before a total knee arthroplasty. An infection in the joint could lead to even greater pain and joint instability, requiring extensive surgery. For this reason, the nurse monitors the patient for signs of infection, such as redness, swelling, fever, and elevated white blood cell count. Pain, knee stiffness, or instability may be present with osteoarthritis.

Nursing management of a patient with a brain tumor includes (select all that apply) a. discussing with the patient methods to control appropriate behavior b. using diversion techniques to leep the patient stimulated and motivated c. assisting and supporting the family in understanding any changes in behavior d. limiting self care activities until the patient has regained maximal physical functioning e. plan for seizure precautions and teaching the patient and caregiver about antiseizure drugs.

C, E

A male patient with a diagnosis of Parkinson's disease (PD) has been admitted recently to a long-term care facility. Which action should the health care team take in order to promote adequate nutrition for this patient? A. Provide multivitamins with each meal. B. Provide a diet that is low in complex carbohydrates and high in protein. C. Provide small, frequent meals throughout the day that are easy to chew and swallow. D. Provide the patient with a minced or pureed diet that is high in potassium and low in sodium.

C. Nutritional support is a priority in the care of individuals with PD. Such patients may benefit from meals that are smaller and more frequent than normal and that are easy to chew and swallow. Multivitamins are not necessary at each meal, and vitamin intake, along with protein intake, must be monitored to prevent contraindications with medications. It is likely premature to introduce a minced or pureed diet, and a low carbohydrate diet is not indicated.

The nurse is caring for a group of patients on a medical unit. After receiving report, which patient should the nurse see first? A. A 42-year-old patient with multiple sclerosis who was admitted with sepsis B. A 72-year-old patient with Parkinson's disease who has aspiration pneumonia C. A 38-year-old patient with myasthenia gravis who declined prescribed medications D. A 45-year-old patient with amyotrophic lateral sclerosis who refuses enteral feedings

C. Patients with myasthenia gravis who discontinue pyridostigmine (Mestinon) will develop a myasthenic crisis. Myasthenia crisis results in severe muscle weakness and can lead to a respiratory arrest.

Which assessment is most important for the nurse to make regarding a patient with myasthenia gravis? a. Pupil size b. Grip strength c. Respiratory effort d. Level of consciousness

C. Because respiratory insufficiency may be life threatening, it will be most important to monitor respiratory function. The other data also will be assessed but are not as critical.

While the nurse is transporting a patient on a stretcher to the radiology department, the patient begins having a tonic-clonic seizure. Which action should the nurse take? a. Insert an oral airway during the seizure to maintain a patent airway. b. Restrain the patient's arms and legs to prevent injury during the seizure. c. Time and observe and record the details of the seizure and postictal state. d. Avoid touching the patient to prevent further nervous system stimulation.

C. Because the diagnosis and treatment of seizures frequently are based on the description of the seizure, recording the length and details of the seizure is important. Insertion of an oral airway and restraining the patient during the seizure are contraindicated. The nurse may need to move the patient to decrease the risk of injury during the seizure.

Which nursing diagnosis is likely to be a priority in the care of a patient with myasthenia gravis (MG)? A. Acute confusion B. Bowel incontinence C. Activity intolerance D. Disturbed sleep pattern

C. The primary feature of MG is fluctuating weakness of skeletal muscle. Bowel incontinence and confusion are unlikely signs of MG, and although sleep disturbance is likely, activity intolerance is usually of primary concern.

The nurse receives report from the licensed practical nurse about care provided to patients on the orthopedic surgical unit. It is most important for the nurse to follow up on which statement? A. "The patient who had a spinal fusion 12 hours ago has hypoactive bowel sounds and is not passing flatus." B. "The patient who had cervical spine surgery 2 days ago wants to wear her soft cervical collar when out of bed." C. "The patient who had spinal surgery 3 hours ago is complaining of a headache and has clear drainage on the dressing." D. "The patient who had a laminectomy 24 hours ago is using patient-controlled analgesia with morphine for pain management."

C. "The patient who had spinal surgery 3 hours ago is complaining of a headache and has clear drainage on the dressing." After spinal surgery there is potential for cerebrospinal fluid (CSF) leakage. Severe headache or leakage of CSF (clear or slightly yellow) on the dressing should be reported immediately. The drainage is CSF if a dipstick test is positive for glucose. Patients after spinal surgery may experience paralytic ileus and interference with bowel function for several days. Postoperatively most patients require opioids such as morphine IV for 24 to 48 hours. Patient-controlled analgesia is the preferred method for pain management during this time. After cervical spine surgery patients often wear a soft or hard cervical collar to immobilize the neck.

A 54-year-old patient admitted with cellulitis and probable osteomyelitis received an injection of radioisotope at 9:00 AM before a bone scan. The nurse should plan to send the patient for the bone scan at what time? A. 9:30 PM B. 10:00 AM C. 11:00 AM D. 1:00 PM

C. 11:00 AM A technician usually administers a calculated dose of a radioisotope 2 hours before a bone scan. If the patient was injected at 9:00 AM, the procedure should be done at 11:00 AM. 10:00 AM would be too early; 1:00 PM and 9:30 PM would be too late.

The nurse is caring for patients in a primary care clinic. Which individual is most at risk to develop osteomyelitis caused by Staphylococcus aureus? A. 22-year-old female with gonorrhea who is an IV drug user B. 48-year-old male with muscular dystrophy and acute bronchitis C. 32-year-old male with type 1 diabetes mellitus and a stage IV pressure ulcer D. 68-year-old female with hypertension who had a knee arthroplasty 3 years ago

C. 32-year-old male with type 1 diabetes mellitus and a stage IV pressure ulcer Osteomyelitis caused by Staphylococcus aureus is usually associated with a pressure ulcer or vascular insufficiency related to diabetes mellitus. Osteomyelitis caused by Staphylococcus epidermidis is usually associated with indwelling prosthetic devices such as joint replacements. Osteomyelitis caused by Neisseria gonorrhoeae is usually associated with gonorrhea. Osteomyelitis caused by Pseudomonas is usually associated with IV drug use. Muscular dystrophy is not associated with osteomyelitis.

The nurse determines that dietary teaching for a 75-year-old patient with osteoporosis has been successful when the patient selects which highest-calcium meal? A. Chicken stir-fry with 1 cup each onions and green peas, and 1 cup of steamed rice B. Ham and Swiss cheese sandwich on whole wheat bread, steamed broccoli, and an apple C. A sardine (3 oz) sandwich on whole wheat bread, 1 cup of fruit yogurt, and 1 cup of skim milk D. A two-egg omelet with 2 oz of American cheese, one slice of whole wheat toast, and a half grapefruit

C. A sardine (3 oz) sandwich on whole wheat bread, 1 cup of fruit yogurt, and 1 cup of skim milk The highest calcium content is present in the lunch containing milk and milk products (yogurt) and small fish with bones (sardines). Chicken, onions, green peas, rice, ham, whole wheat bread, broccoli, apple, eggs, and grapefruit each have less than 75 mg of calcium per 100 g of food. Swiss cheese and American cheese have more calcium, but not as much as the sardines, yogurt, and milk

A 63-year-old woman has been taking prednisone (Deltasone) daily for several years after a kidney transplant to prevent organ rejection. What is most important for the nurse to assess? A. Staggering gait B. Ruptured tendon C. Back or neck pain D. Tardive dyskinesia

C. Back or neck pain Osteoporosis with resultant fractures is a frequent and serious complication of systemic corticosteroid therapy. The ribs and vertebrae are affected the most, and patients should be observed for signs of compression fractures (back and neck pain). Phenytoin (Dilantin) is an antiseizure medication. An adverse effect of phenytoin is an ataxic (or staggering) gait. A rare adverse effect of ciprofloxacin (Cipro) and other fluoroquinolones is tendon rupture, usually of the Achilles tendon. The highest risk is in people age 60 and older and in people taking corticosteroids. Antipsychotics and antidepressants may cause tardive dyskinesia, which is characterized by involuntary movements of the tongue and face.

Which assessment is most important for the nurse to make regarding a patient with myasthenia gravis? a. Pupil size b. Grip strength c. Respiratory effort d. Level of consciousness

C. Because respiratory insufficiency may be life threatening, it will be most important to monitor respiratory function. The other data also will be assessed but are not as critical.

While the nurse is transporting a patient on a stretcher to the radiology department, the patient begins having a tonic-clonic seizure. Which action should the nurse take? a. Insert an oral airway during the seizure to maintain a patent airway. b. Restrain the patient's arms and legs to prevent injury during the seizure. c. Time and observe and record the details of the seizure and postictal state. d. Avoid touching the patient to prevent further nervous system stimulation.

C. Because the diagnosis and treatment of seizures frequently are based on the description of the seizure, recording the length and details of the seizure is important. Insertion of an oral airway and restraining the patient during the seizure are contraindicated. The nurse may need to move the patient to decrease the risk of injury during the seizure.

Increased ICP in the left cerebral cortex, caused by intracranial bleeding causes displacement of brain tissue to the right hemisphere beneath the falx cerebri. The nurse knows that this is referred to as a. uncal herniation b. tentorial herniation c. cingulate herniation d. temporal lobe herniation

C. Cingulate herniation- the dural structures that separate the two hemispheres and the cerebral hemispheres from the cerebellum influence the patterns of cerebral herniation. A cingulated herniation occurs where there is lateral displacement of brain tissue beneath the falx cerebri.

A 49-year-old patient with multiple sclerosis (MS) is to begin treatment with glatiramer acetate (Copaxone). Which information will the nurse include in patient teaching? a. Recommendation to drink at least 4 L of fluid daily b. Need to avoid driving or operating heavy machinery c. How to draw up and administer injections of the medication d. Use of contraceptive methods other than oral contraceptives

C. Copaxone is administered by self-injection. Oral contraceptives are an appropriate choice for birth control. There is no need to avoid driving or drink large fluid volumes when taking glatiramer.

Which medication information will the nurse identify as a concern for a patient's musculoskeletal status? a. The patient takes a daily multivitamin and calcium supplement. b. The patient takes hormone therapy (HT) to prevent "hot flashes." c. The patient has severe asthma and requires frequent therapy with oral corticosteroids. d. The patient has migraine headaches treated with nonsteroidal antiinflammatory drugs (NSAIDs).

C. Frequent or chronic corticosteroid use may lead to skeletal problems such as avascular necrosis and osteoporosis. The use of HT and calcium supplements will help prevent osteoporosis. NSAID use does not increase the risk for musculoskeletal problems.

A 40-year-old patient is diagnosed with early Huntington's disease (HD). When teaching the patient, spouse, and children about this disorder, the nurse will provide information about the a. use of levodopa-carbidopa (Sinemet) to help reduce HD symptoms. b. prophylactic antibiotics to decrease the risk for aspiration pneumonia. c. option of genetic testing for the patient's children to determine their own HD risks. d. lifestyle changes of improved nutrition and exercise that delay disease progression.

C. Genetic testing is available to determine whether an asymptomatic individual has the HD gene. The patient and family should be informed of the benefits and problems associated with genetic testing. Sinemet will increase symptoms of HD because HD involves an increase in dopamine. Antibiotic therapy will not reduce the risk for aspiration. There are no effective treatments or lifestyle changes that delay the progression of symptoms in HD.

The nurse determines that teaching about management of migraine headaches has been effective when the patient says which of the following? a. "I can take the (Topamax) as soon as a headache starts." b. "A glass of wine might help me relax and prevent a headache." c. "I will lie down someplace dark and quiet when the headaches begin." d. "I should avoid taking aspirin and sumatriptan (Imitrex) at the same time."

C. It is recommended that the patient with a migraine rest in a dark, quiet area. Topiramate (Topamax) is used to prevent migraines and must be taken for several months to determine effectiveness. Aspirin or other nonsteroidal antiinflammatory medications can be taken with the triptans. Alcohol may precipitate migraine headaches.

The earliest signs of increased ICP the nurse should assess for include a. Cushing's triad b. unexpected vomiting c. decreasing level of consciousness (LOC) d. dilated pupil with sluggish response to light

C. One of the most sensitive signs of increased intracranial pressure (ICP) is a decreasing LOC. A decrease in LOC will occur before changes in vital signs, ocular signs, and projectile vomiting occur

An unconscious patient with increased ICP in on ventilatory support. The nurse notifies the health care provider when arterial blood gas measurement results reveal a a. pH of 7.43 b. SaO2 of 94% c. PaO2 of 50 mm Hg d. PaCO2 of 30 mm Hg

C. PaO2 of 50 mm Hg- A PaO2 of 50 mm Hg reflects a hypoxemia that may lead to further decreased cerebral perfusion and hypoxia and must be corrected. The pH of SaO2 are within normal range, and a PaCO2 of 30 mm Hg reflects acceptable value for the patient with increased ICP

While the nurse performs ROM on an unconscious patient with increased ICP, the patient experiences severe decerebrate posturing reflexes. The nurse should a. use restraints to protect the patient from injury b. administer CNS depressants to lightly sedate the patient c. perform the exercises less frequently because posturing can increase ICP d. continue the exercises because they are necessary to maintain musculoskeletal function

C. Perform the exercises less frequently because posturing can increase ICP- If reflex posturing occurs during ROM or positioning of the patient, these activities should be done less frequently until the patient's condition stabilizes, because posturing can case increases in ICP. Neither restraints nor CNS depressants would be indicated.

A 54-year old man is recovering from a skull fracture with a subacute subdural hematoma. He has return of motor control and orientation but appears apathetic and has reduced awareness of his environment. When planning discharge or the patient, the nurse explains to the patient and the family that a. continuous improvement in the patient's condition should occur until he has returned to pre trauma status b. the patient's complete recovery may take years, and the family should plan for his long term dependent care c. the patient is likely to have long term emotional and mental changes that may require continued professional help d. role changes in family members will be necessary because the patient will be dependent on his family for care and support

C. Residual mental and emotional changes of brain trauma with personality changes are often the most incapacitating problems following head injury and are common in patients who have been comatose longer than 6 hours. Families must be prepared for changes in the patient's behavior to avoid family-patient friction and maintain family functioning, and professional assistance may be required. There is no indication he will be dependent on others for care, but he likely will not return to pre trauma status

The nurse prepares to administer IV ibandronate (Boniva) to a 67-year-old woman with osteoporosis. What is a priority laboratory assessment to make before the administration of ibandronate? A. Serum calcium B. Serum creatinine C. Serum phosphate D. Serum alkaline phosphatase

C. Serum phosphate Ibandronate is a bisphosphonate that is administered IV every 3 months and is administered slowly over 15 to 30 seconds to prevent renal damage. Ibandronate should not be used by patients taking other nephrotoxic drugs or by those with severe renal impairment (defined as serum creatinine above 2.3 mg/dL or creatinine clearance less than 30 mL/min).

CN III originating in the midbrain is assessed by the nurse for an early indication of pressure on the brainstem by a. assessing for nystagmus b. testing the corneal reflex c. testing pupillary reaction to light d. testing for oculocephalic (doll's eye) reflex

C. Testing pupillary reaction to light- One of the functions of CN III, the oculomotor nerve, is pupillary constriction, and testing for pupillary constriction is important to identify patients at risk for brainstem herniation caused by increased ICP. The corneal reflex is used to assess the functions of CN V and VII, and the oculocephalic reflex tests all cranial nerves involved with eye movement. Nystagmus is commonly associatted with specific lesions or chemical toxicities and is not a definitive sign of ICP

A 22-year-old patient seen at the health clinic with a severe migraine headache tells the nurse about having other similar headaches recently. Which initial action should the nurse take? a. Teach about the use of triptan drugs. b. Refer the patient for stress counseling. c. Ask the patient to keep a headache diary. d. Suggest the use of muscle-relaxation techniques.

C. The initial nursing action should be further assessment of the precipitating causes of the headaches, quality, and location of pain, etc. Stress reduction, muscle relaxation, and the triptan drugs may be helpful, but more assessment is needed first.

On physical examination of a patient with headache and fever, the nurse would suspect a brain abscess when the patient has a. seizures b. nuchal rigidity c. focal symptoms d. signs of increased ICP

C. The symptoms of brain abscess closely resemble those of meningitis and encephalitis, including fever, headache, and increased ICP, except the patient also usually has some focal symptoms that reflect the local are of the abscess.

The nurse on the clinical unit is assigned to four patients. Which patient should she assess first? a. patient with a skull fracture whose nose is bleeding b. elderly patient with a stroke who is confused and whose daughter is present c. patient with meningitis who is suddenly agitated and reporting a HA of 10 on a 0 to 10 scale d. patient who had a craniotomy for a brain tumor who is now 3 days postoperative and has had continued emesis

C. patient with meningitis who is suddenly agitated and reporting a HA of 10 on a 0 to 10 scale

Which intervention should the nurse delegate to the LPN when caring for a patient following an acute stroke? a. assess the patient's neurologic status b. assess the patient's gag reflex before beginning feeding c. administer ordered antihypertensives and platelet inhibitors d. teach the patient's caregivers strategies to minimize unilateral neglect

C: Administer ordered antihypertensives and platelet inhibitors- medication administration is within the scope of practice for an LPN. Assessment and teaching are within the scope of practice for the RN.

A thrombus that develops in a cerebral artery does not always cause a loss of neurologic function because a. the body can dissolve the atherosclerotic plaques as they form b. some tissues of the brain do not require constant blood supply to prevent damage c. circulation through the circle of Willis may provide blood supply to the affected area of the brain d. neurologic deficits occur only when major arteries are occluded by thrombus formation around an atherosclerotic plaque

C: Circulation through the circle of Willis may provide blood supply to the affected area of the brain. The communication between cerebral arteries in the circle of Willing provides a collateral circulation, which may maintain circulation to an area of the brain if its original blood supply is obstructed. ALl areas of the brain require constant blood supply, and atherosclerotic plaques are not readily reversed. Neurologic deficits can result from ischemia cause by many factors.

The incidence of ischemic stroke in patients with TIAs and other risk factors is reduced with administration of a. furosemide (Lasix) b. lovastatin (Mevacor) c. daily low dose aspirin d. nimodipine (Nimotop)

C: Daily low dose aspirin- the administration of antiplatelet agents, such as aspirin, dipyridamole (Persantine), and ticlopdipine (Ticlid), reduces the incidence of stroke in those at risk. Anticoagulants are also used for prevention of embolic strokes but increase the risk for hemorrhage. Diuretics are not indicated for stroke prevention other than for their role in controlling BP, and antilipemic agents have bot been found to have a significant effect on stroke prevention. The calcium channel blocker nimodipine is used in patients with subarachnoid hemorrhage to decrease the effects of vasospasm and minimize tissue damage. P.S. I freaking love you and good luck on the final!!

A patient with a stroke has a right sided hemiplegia. The nurse prepares family members to help control behavior changes seen with this type of stroke by teaching them to a. ignore undesirable behaviors manifested by the patient b. provide directions to the patient verbally in small steps c. distract the patient from inappropriate emotional responses d. supervise all activities before allowing the patient to pursue them independently

C: Distract the patient from inappropriate emotional responses- patients with left-sided brain damage from stroke often experience emotional lability, inappropriate emotional responses, mood swings, and uncontrolled tears or laughter disproportionate or out of context with the situation. The behavior is upsetting and embarrassing to both the patient and the family, and the patient should be distracted to minimize its presence. Patients with right-brain damage often have impulsive, rapid behavior that supervision and direction.

In promoting health maintenance for prevention of strokes, the nurse understands that the highest risk for the most common type of stroke is present in a. African Americans b. women who smoke c.individuals with hypertension and diabetes d. those who are obese with high dietary fat intake

C: Individuals with hypertension and diabetes- The highest risk factors for thrombotic stroke are hypertension and diabetes. African Americans have a higher risk for stroke than do white persons but probably because they have a greater incidence of hypertension. Factors such as obesity, diet high in saturated fats and cholesterol, cigarette smoking, and excessive alcohol use are also risk factors but carry less risk than hypertension.

A carotid endarterectomy is being considered as a treatment for a patient who has had several TIAs. The nurse explains to the patient that this surgery a. is used to restore blood to the brain following an obstruction of a cerebral artery b. involves intracranial surgery to join a superficial extracranial artery to an intracranial artery c. involves removing an atherosclerotic plaque in the carotid artery to prevent an impending stroke d. is sued to open a stenosis in a carotid artery with a balloon and stent to restore cerebral circulation

C: Involves removing an atherosclerotic plaque in the carotid artery to prevent an impending stroke- An endarterectomy is a removal of an atherosclerotic plaque, and plaque in the carotid artery may impair circulation enough to cause a stroke. A carotid endarterectomy is performed to prevent a cerebrovascular accident (CVA), as are most other surgical procedures. An extacranial-intracranial bypass involves cranial surgery to bypass a sclerotic intacranial artery. Percutaneous transluminal angioplasty uses a balloon to compress stenotic areas in the carotid and vertebrobasilar arteries and often includes inserting a stent to hold the artery open.

A patient with right hemisphere stroke has a nursing diagnosis of unilateral neglect related to sensory perceptual deficits. During the patient's rehabilitation, it is important for the nurse to a. avoid positioning the patient on the affected side b. place all objects for care on the patient's unaffected side c. teach the patient to care consciously for the affected side d. protect the affected side from injury with pillows and supports

C: Teach the patient to care consciously for the affected side- unilateral neglect, or neglect syndrome, occurs when the patient with a stroke is unaware of the affected side of the body, which puts the patient at risk for injury. During the acute phase, the affected side is cared for by the nurse with positioning and support, during rehabilitation the patient is taught to care consciously for and attend to the affected side of the body to protect it from injury. Patients may be positioned on the affected side for up to 30 minutes.

The neurologic functions that are affected by a stroke are primarily related to a. the amount of tissue area involved b. the rapidity of onset of symptoms c. the brain area perfused by the affected artery d. the presence or absence of collateral circulation

C: The brain area perfused by the affected artery- clinical manifestation of altered neurologic function differ, depending primarily on the specific cerebral artery involved and the area of the brain that is perfused by the artery. The degree of impairment depends on rapidity of onset, the size of the lesion, and the presence of collateral circulation.

An appropriate food for a patient with a stroke who has mild dysphagia is a. fruit juices b. pureed meat c. scrambled eggs d. fortified milkshakes

C: scrambled eggs- soft foods that provide enough texture, flavor, and bulk to stimulate swallowing should be used for the patient with dysphasia. Thin liquids are difficult to swallow, and patients may not be able to control them in the mouth. Pureed foods are often too bland and to smooth, and milk products should be avoided because they tend to increase the viscosity of mucus and increase salivation.

How is Cerebral Perfusion Pressure calculated?

CPP = MAP - ICP

You would expect the following diagnostic tests to be done for increased ICP (4):

CT scan MRI Cerebral angio EEG

This measurement figure represents the pressure gradient driving cerebral blood flow (CBF) -- and therefore driving oxygen and metabolite delivery

Cerebral Perfusion Pressure (CPP)

Cluster Respirations

Clusters of breaths with irregular spaced pauses

this type of brain injury is characterized by visible bruising due to trauma/blood, possible surface hemorrhage, longer duration of unconsciousness (than concussion), contracoup phenomenon

Contusion

The nurse has identified the nursing diagnosis of disturbed thought processes related to effects of dementia for a patient with late-stage Alzheimer's disease (AD). An appropriate intervention for this problem is to a. maintain a consistent daily routine for the patient's care. b. encourage the patient to discuss events from the past. c. reorient the patient to the date and time every few hours. d. provide the patient with current newspapers and magazines.

Correct Answer: A Rationale: Providing a consistent routine will decrease anxiety and confusion for the patient. In late-stage AD, the patient will not remember events from the past. Reorientation to time and place will not be helpful to the patient with late-stage AD. The patient with late-stage AD will not be able to read. Cognitive Level: Application Text Reference: p. 1571 Nursing Process: Planning NCLEX: Physiological Integrity

Risperidone (Risperdal) is prescribed for an outpatient with moderate Alzheimer's disease (AD). Which information obtained by the nurse at the next clinic appointment indicates that the medication is effective? a. The patient has less agitation. b. The patient is dressed appropriately. c. The patient is able to swallow a pill. d. The patient's speech is clearer.

Correct Answer: A Rationale: Risperidone is an antipsychotic used to treat the agitation, aggression, and behavioral problems associated with AD. The other improvements might occur with cholinesterase inhibitors. Cognitive Level: Application Text Reference: p. 1568 Nursing Process: Evaluation NCLEX: Physiological Integrity

A patient who has been taking bromocriptine (Parlodel) and benztropine (Cogentin) for Parkinson's disease is experiencing a worsening of symptoms. The nurse will anticipate that patient may benefit from a. complete drug withdrawal for a few weeks. b. use of levodopa (L-dopa)-carbidopa (Sinemet). c. withdrawal of anticholinergic therapy. d. increasing the dose of bromocriptine.

Correct Answer: B Rationale: After the dopamine receptor agonists begin to fail to relieve symptoms, the addition of L-dopa with carbidopa can be added to the regimen. Complete drug withdrawal will result in worsening of symptoms. Anticholinergic therapy should be continued to help maintain the balance between the actions of dopamine and acetylcholine. Increasing the dose of bromocriptine will increase the risk for toxic effects. Cognitive Level: Comprehension Text Reference: p. 1551 Nursing Process: Planning NCLEX: Physiological Integrity

The nurse identifies the nursing diagnosis of impaired physical mobility related to bradykinesia for a patient with Parkinson's disease. To assist the patient to ambulate safely, the nurse should a. allow the patient to ambulate only with assistance. b. instruct the patient to rock from side to side to initiate leg movement. c. have the patient take small steps in a straight line directly in front of the feet. d. teach the patient to keep the feet in contact with the floor and slide them forward.

Correct Answer: B Rationale: Rocking the body from side to side stimulates balance and improves mobility. The patient should initially be ambulated with assistance but might not require continual assistance with ambulation. The patient should maintain a wide base of support to help with balance. The patient should lift the feet and avoid a shuffling gait. Cognitive Level: Application Text Reference: p. 1554 Nursing Process: Implementation NCLEX: Physiological Integrity

A family member of a patient with possible Alzheimer's disease asks the nurse the purpose of the Mini-Mental State Examination (MMSE). Which response by the nurse is appropriate? a. The MMSE helps in establishing the diagnosis of Alzheimer's disease (AD). b. The MMSE is useful in determining the degree of mental impairment. c. The MMSE determines the choice of the most appropriate treatment. d. The MMSE aids in differentiating acute delirium from chronic dementia.

Correct Answer: B Rationale: The MMSE establishes the degree of mental impairment at the time it is given. It does not establish a diagnosis of AD but when given repeatedly over time may help to determine the progression of AD. The choice of treatment is made on the basis of multiple data, not just the MMSE. The MMSE may be abnormal with either delirium or dementia and is not useful in determining which condition the patient has. Cognitive Level: Application Text Reference: p. 1563 Nursing Process: Implementation NCLEX: Physiological Integrity

When teaching the spouse of a patient who is being evaluated for Alzheimer's disease (AD) about the disorder, the nurse explains that a. the most important risk factor for AD is a family history of the disorder. b. a diagnosis of AD can be made only when other causes of dementia have been ruled out. c. new drugs have been shown to reverse AD dramatically in some patients. d. the presence of brain atrophy detected by MRI confirms the diagnosis of AD in patients with dementia.

Correct Answer: B Rationale: The diagnosis of AD is one of exclusion. Age is the most important risk factor for development of AD. Drugs can slow the deterioration but do not dramatically reverse the effects of AD. Brain atrophy is a common finding in AD, but it can occur in other diseases as well. Cognitive Level: Comprehension Text Reference: p. 1568 Nursing Process: Implementation NCLEX: Physiological Integrity

A home-health patient with Alzheimer's disease (AD) and mild dementia has a new prescription for donepezil (Aricept). Which nursing action will be most effective in ensuring compliance with the medication? a. Setting the medications up weekly in a medication box b. Calling the patient daily with a reminder to take the medication c. Having the patient's spouse administer the medication d. Posting reminders to take the medications in the patient's house

Correct Answer: C Rationale: Because the patient with mild dementia will have difficulty with learning new skills and forgetfulness, the most appropriate nursing action is to have someone else administer the Aricept. The other nursing actions will not be as effective in ensuring that the patient takes the medications. Cognitive Level: Application Text Reference: pp. 1563, 1567 Nursing Process: Implementation NCLEX: Physiological Integrity

When assessing a patient with Alzheimer's disease (AD) who is being admitted to a long-term care facility, the nurse learns that the patient has had several episodes of wandering away from home. Which nursing action will the nurse include in the plan of care? a. Ask the patient why the wandering episodes have occurred. b. Reorient the patient to the new living situation several times daily. c. Place the patient in a room close to the nurses' station. d. Have the family bring in familiar items from the patient's home.

Correct Answer: C Rationale: Patients at risk for problems with safety require close supervision. Placing the patient near the nurse's station will allow nursing staff to observe the patient more closely. Use of "why" questions is frustrating for the patient with AD, who are unable to understand clearly or verbalize the reason for wandering behaviors. Because of the patient's short-term memory loss, reorientation will not help to prevent wandering behavior. Because the patient had wandering behavior at home, familiar objects will not prevent wandering. Cognitive Level: Application Text Reference: p. 1573 Nursing Process: Planning NCLEX: Safe and Effective Care Environment

A patient is seen in the health clinic with symptoms of a stooped posture, shuffling gait, and pill rolling-type tremor. The nurse will anticipate teaching the patient about a. preparation for an MRI. b. purpose of EEG testing. c. antiparkinsonian drugs. d. oral corticosteroids.

Correct Answer: C Rationale: The diagnosis of Parkinson's is made when two of the three characteristic signs of tremor, rigidity, and bradykinesia are present. The confirmation of the diagnosis is made on the basis of improvement when antiparkinsonian drugs are administered. This patient has symptoms of tremor and bradykinesia; the next anticipated step will be treatment with medications. MRI and EEG are not useful in diagnosing Parkinson's disease, and corticosteroid therapy is not used to treat it. Cognitive Level: Application Text Reference: p. 1550 Nursing Process: Planning NCLEX: Physiological Integrity

21. A patient with Parkinson's disease has decreased tongue mobility and an inability to move the facial muscles. The nurse recognizes that these impairments commonly contribute to the nursing diagnosis of a. disuse syndrome related to loss of muscle control. b. self-care deficit related to bradykinesia and rigidity. c. impaired verbal communication related to difficulty articulating. d. impaired oral mucous membranes related to inability to swallow.

Correct Answer: C Rationale: The inability to use the tongue and facial muscles decreases the patient's ability to socialize or communicate needs. Disuse syndrome is not an appropriate nursing diagnosis because the patient is continuing to use the muscles as much as possible. There is no indication in the stem that the patient has a self-care deficit, bradykinesia, or rigidity. The oral mucous membranes will continue to be moist and should not be impaired by the patient's difficulty swallowing. Cognitive Level: Application Text Reference: p. 1554 Nursing Process: Diagnosis NCLEX: Physiological Integrity

A patient with Alzheimer's disease (AD) is hospitalized with a urinary tract infection. The spouse tells the nurse, "I am just exhausted from the constant care and worry. We don't have any children and we can't afford a nursing home. I don't know what to do." The most appropriate nursing diagnosis for the spouse is a. anxiety related to limited financial resources. b. ineffective health maintenance related to stress. c. caregiver role strain related to limited resources for caregiving. d. social isolation related to unrelieved caregiving responsibilities.

Correct Answer: C Rationale: The spouse's statements are most consistent with caregiver role strain. The other diagnoses each address one aspect of the spouse's problem, but caregiver-role strain related to limited resources for caregiving addresses all the information the nurse has about this situation. Cognitive Level: Application Text Reference: pp. 1574-1575 Nursing Process: Diagnosis NCLEX: Psychosocial Integrity

A 62-year-old patient is brought to the clinic by a family member who is concerned about the patient's increasing sleep disturbances and inability to solve common problems. To obtain information about the patient's current mental status, which question should the nurse ask the patient? a. "Where were you were born?" b. "Do have any feelings of sadness?" c. "What day of the week is it today?" d. "How positive is your self-image?"

Correct Answer: C Rationale: This question tests the patient's orientation to time, which is decreased in early Alzheimer's disease (AD) or dementia. Asking the patient about birthplace tests for remote memory, which is intact in the early stages. Questions about the patient's emotions and self-image are helpful in assessing emotional status, but they are not as helpful in assessing mental state. Cognitive Level: Application Text Reference: pp. 1564, 1567 Nursing Process: Assessment NCLEX: Physiological Integrity

A patient has a new prescription for levodopa (L-dopa) to control symptoms of Parkinson's disease. Which assessment data obtained by the nurse may indicate a need for a decrease in the dose? a. The patient has a chronic dry cough. b. The patient has 4 loose stools in a day. c. The patient develops a deep vein thrombosis. d. The patient's blood pressure is 90/46 mm Hg.

Correct Answer: D Rationale: Hypotension is an adverse effect of L-dopa, and the nurse should check with the health care provider before giving the medication. Diarrhea, cough, and deep vein thrombosis are not associated with L-dopa use. Cognitive Level: Application Text Reference: p. 1552 Nursing Process: Evaluation NCLEX: Physiological Integrity

A 72-year-old patient hospitalized with pneumonia is disoriented and confused 2 days after admission. Which assessment information obtained by the nurse about the patient indicates that the patient is experiencing delirium rather than dementia? a. The patient is disoriented to place and time but oriented to person. b. The patient has a history of increasing confusion over several years. c. The patient's speech is fragmented and incoherent. d. The patient was oriented and alert when admitted.

Correct Answer: D Rationale: The onset of delirium occurs acutely. The degree of disorientation does not differentiate between delirium and dementia. Increasing confusion for several years is consistent with dementia. Fragmented and incoherent speech may occur with either delirium or dementia. Cognitive Level: Application Text Reference: p. 1562 Nursing Process: Assessment NCLEX: Physiological Integrity

A patient with Parkinson's disease has decreased tongue mobility and an inability to move the facial muscles. Which nursing diagnosis is of highest priority?

Imbalanced nutrition: less than body requirements rational: The data about the patient indicate that poor nutrition will be a concern because of decreased swallowing. The other diagnoses also may be appropriate for a patient with Parkinson's disease, but the data do not indicate they are current problems for this patient.

Sinemet side effects

Impaired Voluntary Movements

Surgical interventions for brain tumors include:

biopsy craniotomy

Parietal Lobe

Interpretation of taste, pain, touch, temp, and pressure Spatial Perception

Brudzinski Sign

Involuntary flexion of the hip and knee when the neck is passively flexed- indicates meningeal irritation

Apneustic Respirations

Irregular with pauses at the end of inspiration and expiration

A 54-year-old patient is about to have a bone scan. In teaching the patient about this procedure, the nurse should include what information? A. Two additional follow-up scans will be required. B. There will be only mild pain associated with the procedure. C. The procedure takes approximately 15 to 30 minutes to complete. D. The patient will be asked to drink increased fluids after the procedure.

D. The patient will be asked to drink increased fluids after the procedure. Patients are asked to drink increased fluids after a bone scan to aid in excretion of the radioisotope, if not contraindicated by another condition. No follow-up scans and no pain are associated with bone scans that take 1 hour of lying supine.

Increase in any of the contents of the skull (3) frequently results in:

Ischemia of brain tissue

Diagnostic studies for brain tumor include

EEG CT scan Brain scan

Which nursing diagnosis is of highest priority for a patient with Parkinson's disease who is unable to move the facial muscles? a. Activity intolerance b. Self-care deficit: toileting c. Ineffective self-health management d. Imbalanced nutrition: less than body requirements

D. The data about the patient indicate that poor nutrition will be a concern because of decreased swallowing. The other diagnoses may also be appropriate for a patient with Parkinson's disease, but the data do not indicate that they are current problems for this patient.

Bell's Palsy Medications

Corticosteroids, Zovirax, Valtrex, Famvir

The nurse who notes that a 59-year-old female patient has lost 1 inch in height over the past 2 years will plan to teach the patient about a. discography studies. b. myelographic testing. c. magnetic resonance imaging (MRI). d. dual-energy x-ray absorptiometry (DXA).

D. The decreased height and the patient's age suggest that the patient may have osteoporosis and that bone density testing is needed. Discography, MRI, and myelography are typically done for patients with current symptoms caused by musculoskeletal dysfunction and are not the initial diagnostic tests for osteoporosis.

Limbic System

Emotional and visceral patterns for survival Learning and memory

A patient with ICP monitoring has pressure of 12 mm Hg. The nurse understand that this pressure reflects a. a severe decrease in cerebral perfusion pressure b. an alteration in the production of CSF c. the loss of autoregulatory control of ICP d. a normal balance between brain tissue, blood, and CSF

D. A normal balance between brain tissue, blood, and CSF- normal is 10- 15 mm Hg

Successful achievement of patient outcomes for the patient with cranial surgery would be best indicated by the a. ability to return home in 6 days b. ability to meet all self-care needs c. acceptance of residual neurologic deficits d. absence of signs and symptoms of increased ICP

D. Absence of signs and symptoms of increased ICP- The primary goal after cranial surgery is prevention of increased ICP, and interventions to prevent ICP and infection postoperatively are nursing priorities. The residual deficits, rehabilitation potential, and ultimate function of the patient depend on the reason for surgery, the postoperative course, and the patient's general state of health

The nurse suspects the presence of an arterial epidural hematoma in the patient who experiences a. failure to regain consciousness following a head injury b. a rapid deterioration of neurologic function within 24 to 48 hours following a head injury c. nonspecific, nonlocalizing progression of alteration in LOC occurring over weeks or months d. unconsciousness at the time of a head injury with a brief period of consciousness followed by a decrease in LOC

D. An arterial epidural hematoma is the most acute neurologic emergency, and the typical symptoms include unconsciousness at the scene, with a brief lucid interval followed by a decrease in LOC. An acute subdural hematoma manifests signs within 48 hours of an injury; a chronic subdural hematoma develops over weeks or months

A patient is admitted to the hospital with possible bacterial meningitis. During the initial assessment, the nurse questions the patient about a recent history of a. mosquito or tick bites b. chickenpox or measles c. cold sores or fever blisters d. an upper respiratory infection

D. An upper respiratory infection- Meningitis is often a result of an upper respiratory infection or middle ear infection, where organisms gain entry to the CNS. Epidemic encephalitis is transmitted by ticks and mosquitoes, and nonepidemic encephalitis may occur as a complication of measles, chickenpox, or mumps. Encephalitis caused by the herpes simplex virus carries a high fatality rate

Which medication taken by a patient with restless legs syndrome should the nurse discuss with the patient? a. Multivitamin (Stresstabs) b. Acetaminophen (Tylenol) c. Ibuprofen (Motrin, Advil) d. Diphenhydramine (Benadryl)

D. Antihistamines can aggravate restless legs syndrome. The other medications will not contribute to restless legs syndrome.

Kernigs Sign

Loss of the ability of a supine client to straighten the leg completely when it is fully flexed at the knee and hip

Drugs or disease that impair the function of the extrapyramidal system may cause loss of? A. sensations of pain and temperature B. regulation of the automonic nervous system C. integration of somatic and special sensory inputs D. Automatic movements asociated with skeletal muscle activity

Correct answer: D Rationale: A group of descending motor tracts carries impulses from the extrapyramidal system, which includes all motor systems (except the pyramidal system) concerned with voluntary movement. It includes descending pathways originating in the brainstem, basal ganglia, and cerebellum. The motor output exits the spinal cord by way of the ventral roots of the spinal nerves.

In a patient with a disease that affects the myelin sheath of nerves, such as multiple sclerosis the glial cells affected are the? A. microglia b. Astrocytes. C. ependymal cells D. Oligodendrocytes

Correct answer: D Rationale: Glial cell types include oligodendrocytes, astrocytes, ependymal cells, and microglia, and each has specific functions. Oligodendrocytes are specialized cells that produce the myelin sheath of nerve fibers in the central nervous system (CNS), and they are located primarily in the white matter of the CNS.

Hemianopsia

Loss of vision in one half of the visual field of one or both eyes

Data regarding mobility, strength, coordination and activity tolerance are important for the nurse to obtain because? A. Many neurologic diseases affect one or more of these areas B. Patients are less able to identify other neurologic impairments C. These are the first functions to be affected by neurologic diseases D. Aspects of movement are the most important function of the nervous system

Correct answer: a Rationale: Many neurologic disorders affect the patient's mobility, strength, and coordination. These problems can alter the patient's usual activity and exercise patterns.

A patient's eyes jerk while the patient looks to the left. You will record this finding as? A. Nystagmus B. CN VI palsy C. Oculocephalia D. Ophthalmic dyskenesia

Correct answer: a Rationale: Nystagmus is defined as fine, rapid jerking movements of the eyes.

10. The nurse is caring for a patient with peripheral neuropathy who is going to have EMG studies tomorrow morning. The nurse should? A. Ensure the patient has an empty bladder B. Instruct the patient that there is no risk of electric shock C. Ensure the patient has no metallic jewelry or metal fragments D. Instruct the patient that she or he may experience pain during the study

Correct answer: b Rationale: Electromyography (EMG) is used to assess electrical activity associated with nerves and skeletal muscles. Activity is recorded by insertion of needle electrodes to detect muscle and peripheral nerve disease. The nurse should inform the patient that pain and discomfort are associated with insertion of needles. There is no risk of electric shock with this procedure.

During neurologic testing, the patient is able to perceive pain elicited by pinprick based on this finding the nurse may omit testing for? A. Position sense B. Patellar reflexes C. Temperature perception D. Heel-to- shin movements

Correct answer: c Rationale: If pain sensation is intact, assessment of temperature sensation may be omitted because both sensations are transmitted by the same ascending pathways

An obstruction of the anterior cerebral arteries will affect functions of? A. Visual imaging B. balane and coordination C. Judgment, insight, and reasoning D. Visual and auditory integration for language comprehension

Correct answer: c Rationale: The anterior cerebral artery feeds the medial and anterior portions of the frontal lobes. The anterior portion of the frontal lobe controls higher order processes such as judgment and reasoning.

Assessment of muscle strength of older adults cannot be compared with that of younger adults because? A. Stroke is more common in older adults B. Nutritional status is better in younger adults C. Most young people exercise more than older people D. Aging leads to a decrease in muscle bulk and strength

Correct answer: d Rationale: Changes associated with aging include decreases in muscle strength and agility in relation to decreased muscle bulk.

Paralysis of lateral gaze indicates a lesion of cranial nerve A. II B. III C. IV D. VI

Correct answer: d Rationale: Cranial nerves III (oculomotor), IV (trochlear), and VI (abducens) are responsible for eye movement. The lateral rectus eye muscle is innervated by cranial nerve VI, and it is the primary muscle that is responsible for lateral eye movement.

A result of stimulation of the parasympathetic nervous system is (select all that apply) A. constriction of the bronchi B. Dilation of skin blood vessels C. increased secreation of insulin D. increased blood glucose levels E. relaxation of the urinary sphincters

Correct answers: a, b, c, e Rationale: Stimulation of the parasympathetic nervous system results in constriction of the bronchi, dilation of blood vessels to the skin, increased secretion of insulin, and relaxation of the urinary sphincter. Stimulation of the sympathetic nervous system results in increased blood glucose levels.

The nurse is completing a neurovascular assessment on the patient with a tibial fracture and a cast. The feet are pulseless, pale, and cool. The patient says they are numb. What should the nurse suspect is occurring? A Paresthesia B Pitting edema C Poor venous return D Compartment syndrome

D Compartment syndrome The nurse should suspect compartment syndrome with one or more of the following six Ps: paresthesia, pallor, pulselessness, pain distal to the injury and unrelieved with opioids, pressure increases in the compartment, and paralysis. Although paresthesia and poor venous return are evident, these are just some of the manifestations of compartment syndrome.

This morning a 21-year-old male patient had a long leg cast applied and wants to get up and try out his crutches before dinner. The nurse will not allow this. What is the best rationale that the nurse should give the patient for this decision? A The cast is not dry yet, and it may be damaged while using crutches. B The nurse does not have anyone available to accompany the patient. C Rest, ice, compression, and elevation are in process to decrease pain. D Excess edema and other problems are prevented when the leg is elevated for 24 hours

D Excess edema and other problems are prevented when the leg is elevated for 24 hours For the first 24 hours after a lower extremity cast is applied, the leg will be elevated on pillows above the heart level to avoid excessive edema and compartment syndrome. The cast will also be drying during this 24-hour period. RICE is used for soft tissue injuries, not with long leg casts.

The nurse is caring for a 75-year-old woman who underwent left total knee arthroplasty and has a new order to be "up in chair today before noon." What action should the nurse take to protect the knee joint while carrying out the order? A Administer a dose of prescribed analgesic before completing the order. B Ask the physical therapist for a walker to limit weight bearing while getting out of bed. C Keep the continuous passive motion machine in place while lifting the patient from bed to chair. D Put on a knee immobilizer before moving the patient out of bed and keep the surgical leg elevated while sitting.

D Put on a knee immobilizer before moving the patient out of bed and keep the surgical leg elevated while sitting. The nurse should apply a knee immobilizer for stability before assisting the patient to get out of bed. This is a standard measure to protect the knee during movement following surgery. Although an analgesic should be given before the patient gets up in the chair for the first time, it will not protect the knee joint. Full weight bearing is begun before discharge, so a walker will not be used if the patient did not need one before the surgery. The CPM machine is not kept in place while the patient is getting up to the chair.

The nurse is completing discharge teaching with an 80-year-old male patient who underwent right total hip arthroplasty. The nurse identifies a need for further instruction if the patient states the need to A avoid crossing his legs. B use a toilet elevator on toilet seat. C notify future caregivers about the prosthesis. D maintain hip in adduction and internal rotation.

D maintain hip in adduction and internal rotation. The patient should not force hip into adduction or force hip into internal rotation as these movements could displace the hip replacement. Avoiding crossing the legs, using a toilet elevator on a toilet seat, and notifying future caregivers about the prosthesis indicate understanding of discharge teaching.

When assessing motor function of a patient admitted with a stroke, the nurse notes mild weakness of the arm demonstrated by downward drifting of the arm. How should the nurse most accurately document this finding? A. Athetosis B. Hypotonia C. Hemiparesis D. Pronator drift

D. Downward drifting of the arm or pronation of the palm is identified as a pronator drift. Athetosis is a slow, writhing, involuntary movement of the extremities. Hypotonia is flaccid muscle tone, and hemiparesis is weakness of one side of the body.

A patient is having a transsphenoidal hypophysectomy. The nurse should provide preoperative patient teaching about what potential deficit as a result of the surgery? A. Increased heart rate B. Loss of coordination C. Impaired swallowing D. Altered sense of smell

D. Using a transsphenoidal approach to remove the pituitary gland includes a risk of damage to the olfactory cranial nerve because the cell bodies of the olfactory nerve are located in the nasal epithelium. With damage to this nerve, the sense of smell would be altered. Increased heart rate, loss of coordination, and impaired swallowing will not be potential deficits from this surgery.

A 76-year-old patient is being treated with carbidopa/levodopa (Sinemet) for Parkinson's disease. Which information is most important for the nurse to report to the health care provider? a. Shuffling gait b. Tremor at rest c. Cogwheel rigidity of limbs d. Uncontrolled head movement

D. Dyskinesia is an adverse effect of the Sinemet, indicating a need for a change in medication or decrease in dose. The other findings are typical with Parkinson's disease.

A 62-year-old patient who has Parkinson's disease is taking bromocriptine (Parlodel). Which information obtained by the nurse may indicate a need for a decrease in the dose? a. The patient has a chronic dry cough. b. The patient has four loose stools in a day. c. The patient develops a deep vein thrombosis. d. The patient's blood pressure is 92/52 mm Hg.

D. Hypotension is an adverse effect of bromocriptine, and the nurse should check with the health care provider before giving the medication. Diarrhea, cough, and deep vein thrombosis are not associated with bromocriptine use.

Which information about a 72-year-old patient who has a new prescription for phenytoin (Dilantin) indicates that the nurse should consult with the health care provider before administration of the medication? a. Patient has generalized tonic-clonic seizures. b. Patient experiences an aura before seizures. c. Patient's most recent blood pressure is 156/92 mm Hg. d. Patient has minor elevations in the liver function tests.

D. Many older patients (especially with compromised liver function) may not be able to metabolize phenytoin. The health care provider may need to choose another antiseizure medication. Phenytoin is an appropriate medication for patients with tonic-clonic seizures, with or without an aura. Hypertension is not a contraindication for phenytoin therapy.

Which nursing diagnosis is of highest priority for a patient with Parkinson's disease who is unable to move the facial muscles? a. Activity intolerance b. Self-care deficit: toileting c. Ineffective self-health management d. Imbalanced nutrition: less than body requirements

D. The data about the patient indicate that poor nutrition will be a concern because of decreased swallowing. The other diagnoses may also be appropriate for a patient with Parkinson's disease, but the data do not indicate that they are current problems for this patient.

Which prescribed intervention will the nurse implement first for a patient in the emergency department who is experiencing continuous tonic-clonic seizures? a. Give phenytoin (Dilantin) 100 mg IV. b. Monitor level of consciousness (LOC). c. Obtain computed tomography (CT) scan. d. Administer lorazepam (Ativan) 4 mg IV.

D. To prevent ongoing seizures, the nurse should administer rapidly acting antiseizure medications such as the benzodiazepines. A CT scan is appropriate, but prevention of any seizure activity during the CT scan is necessary. Phenytoin will also be administered, but it is not rapidly acting. Patients who are experiencing tonic-clonic seizures are nonresponsive, although the nurse should assess LOC after the seizure.

Metabolic and nutritional needs of the patient with increased ICP are best met with a. enteral feedings that are low in sodium b. the simple glucose available in D5W IV solutions c. a fluid restriction that promotes a moderate dehydration d. balanced, essential nutrition in a form that the patient can tolerate

D. Balanced, essential nutrition in a form that the patient can tolerate= A patient with increased ICP is in a hypermetabolic and hypercatabolic state and needs adequate glucose to maintain fuel for the brain and other nutrients to meet metabolic needs. Malnutrition promotes cerebral edema, and if a patient cannot take oral nutrition, other means of providing nutrition should be used, such as tube feedings or parenteral nutrition. Glucose alone is not adequate to meet nutritional requirements, and 5% dextrose solutions may increase cerebral edema by lowering serum osmolarity. Patients should remain in a normovolemic fluid state with close monitoring of clinical factors such as urine output, fluid intake, serum and urine osmolality, serum electrolytes, and insensible losses.

When assessing the body function of a patient with increased ICP, the nurse should initially assess a. corneal reflex testing b. extremity strength testing c. pupillary reaction to light d. circulatory and respiratory status

D. Circulatory and respiratory status- Of the body functions that should be assessed in an unconscious patient, cardiopulmonary status is the most vital function and gives priorities to the ABCs (airway, breathing, and circulation)

Which information about a 60-year-old patient with MS indicates that the nurse should consult with the health care provider before giving the prescribed dose of dalfampridine (Ampyra)? a. The patient has relapsing-remitting MS. b. The patient walks a mile a day for exercise. c. The patient complains of pain with neck flexion. d. The patient has an increased serum creatinine level.

D. Dalfampridine should not be given to patients with impaired renal function. The other information will not impact whether the dalfampridine should be administered.

A 76-year-old patient is being treated with carbidopa/levodopa (Sinemet) for Parkinson's disease. Which information is most important for the nurse to report to the health care provider? a. Shuffling gait b. Tremor at rest c. Cogwheel rigidity of limbs d. Uncontrolled head movement

D. Dyskinesia is an adverse effect of the Sinemet, indicating a need for a change in medication or decrease in dose. The other findings are typical with Parkinson's disease.

The nurse is reinforcing health teaching about osteoporosis with a 72-year-old patient admitted to the hospital. In reviewing this disorder, what should the nurse explain to the patient? A. With a family history of osteoporosis, there is no way to prevent or slow bone resorption. B. Continuous, low-dose corticosteroid treatment is effective in stopping the course of osteoporosis. C. Estrogen therapy must be maintained to prevent rapid progression of the osteoporosis. D. Even with a family history of osteoporosis, the calcium loss from bones can be slowed by increased calcium intake and exercise.

D. Even with a family history of osteoporosis, the calcium loss from bones can be slowed by increased calcium intake and exercise. The rate of progression of osteoporosis can be slowed if the patient takes calcium supplements and/or foods high in calcium and engages in regular weight-bearing exercise. Corticosteroids interfere with bone metabolism. Estrogen therapy is no longer used to prevent osteoporosis because of the associated increased risk of heart disease and breast and uterine cancer.

example of diuretic treatment and what to monitor for:

Manitol (helps with kidney failure, increased intraocular pressure, increased cranial pressure) -headache, edema, CHF, fluid and electrolyte balance, etc

What causes headache and photophobia when there is increased ICP?

compression of other intracranial structures such as arteries and veins, as well as the cranial nerves (no pain receptors in actual brain tissue)

A 62-year-old patient who has Parkinson's disease is taking bromocriptine (Parlodel). Which information obtained by the nurse may indicate a need for a decrease in the dose? a. The patient has a chronic dry cough. b. The patient has four loose stools in a day. c. The patient develops a deep vein thrombosis. d. The patient's blood pressure is 92/52 mm Hg.

D. Hypotension is an adverse effect of bromocriptine, and the nurse should check with the health care provider before giving the medication. Diarrhea, cough, and deep vein thrombosis are not associated with bromocriptine use.

Which information about a 72-year-old patient who has a new prescription for phenytoin (Dilantin) indicates that the nurse should consult with the health care provider before administration of the medication? a. Patient has generalized tonic-clonic seizures. b. Patient experiences an aura before seizures. c. Patient's most recent blood pressure is 156/92 mm Hg. d. Patient has minor elevations in the liver function tests.

D. Many older patients (especially with compromised liver function) may not be able to metabolize phenytoin. The health care provider may need to choose another antiseizure medication. Phenytoin is an appropriate medication for patients with tonic-clonic seizures, with or without an aura. Hypertension is not a contraindication for phenytoin therapy.

A 67-year-old patient hospitalized with osteomyelitis has an order for bed rest with bathroom privileges with the affected foot elevated on two pillows. The nurse would place highest priority on which intervention? A. Ambulate the patient to the bathroom every 2 hours. B. Ask the patient about preferred activities to relieve boredom. C. Allow the patient to dangle legs at the bedside every 2 to 4 hours. D. Perform frequent position changes and range-of-motion exercises.

D. Perform frequent position changes and range-of-motion exercises. The patient is at risk for atelectasis of the lungs and for contractures because of prescribed bed rest. For this reason, the nurse should place the priority on changing the patient's position frequently to promote lung expansion and performing range-of-motion (ROM) exercises to prevent contractures. Assisting the patient to the bathroom will keep the patient safe as the patient is in pain, but it may not be needed every 2 hours. Providing activities to relieve boredom will assist the patient to cope with the bed rest, and dangling the legs every 2 to 4 hours may be too painful. The priority is position changes and ROM exercises.

Which prescribed intervention will the nurse implement first for a patient in the emergency department who is experiencing continuous tonic-clonic seizures? a. Give phenytoin (Dilantin) 100 mg IV. b. Monitor level of consciousness (LOC). c. Obtain computed tomography (CT) scan. d. Administer lorazepam (Ativan) 4 mg IV.

D. To prevent ongoing seizures, the nurse should administer rapidly acting antiseizure medications such as the benzodiazepines. A CT scan is appropriate, but prevention of any seizure activity during the CT scan is necessary. Phenytoin will also be administered, but it is not rapidly acting. Patients who are experiencing tonic-clonic seizures are nonresponsive, although the nurse should assess LOC after the seizure.

Skull radiographs and a computed tomography (CT) scan provide evidence of a depressed parietal fracture with a subdural hematoma in a patient admitted to the emergency department following an automobile accident. In planning care for the patient, the nurse anticipates that a. the patient will receive life-support measures until the condition stabilizes b. immediate burr holes will be made to rapidly decompress the intracranial activity c. the patient will be treated conservatively with close monitoring for changes in neurologic condition d. the patient will be taken to surgery for a craniotomy for evacuation of blood and decompression of the cranium

D. When there is a depressed fracture and fractures with loose fragments, a craniotomy is indicated to elevate the depressed bone and remove free fragments. A craniotomy is also indicated in cases of acute subdural and epidural hematomas to remove the blood and control the bleeding. Burr holes may be used in an extreme emergency for rapid decompression, but with a depressed fracture, surgery would be the treatment of choice

A nursing intervention is indicated for the patient with hemiplegia is a. the use of a footboard to prevent plantar flexion b. immobilization of the affected arm against the chest with a sling c. positioning the patient in bed with each joint lower than the joint proximal to it d. having the patient perform passive ROM of the affected limb with the unaffected limb

D: Having the patient perform passive ROM of the affected limb with the unaffected limb- active ROM should be initiated on the unaffected side as soon as possible, and passive ROM of the affected side should be started on the first day. Having the patient actively exercise the unaffected side provides the patient with active and passive ROM as needed. Use of footboards is controversial because they stimulate plantar flexion. The unaffected arm should be supported, but immobilization may precipitate a painful shoulder-hand syndrome. The patient should be positioned with each joint higher than the joint proximal to it to prevent dependent edema.

The nurse can assist the patient and the family in coping with the long term effects of a stroke by a. informing family members that the patient will need assistance with almost all ADLs b. explaining that the patient's prestroke behavior will return as improvement progresses c. encouraging the patient and family members to seek assistance from family therapy or stroke support groups d. helping the patient and family understand the significance of residual stroke damage to promote problem solving and planning

D: Helping the patient and family understand the significance of residual stroke damage to promote problem solving and planning- the patient and family need accurate and complete information about the effects of the stroke to problem solve and make plans for chronic care of the patient. It is uncommon for patients with major strokes to return completely to pre stroke function, behaviors, and role, and both the patient and family will mourn these losses. The patient's specific needs for care must be identified, and rehabilitation efforts should be continued at home. Family therapy and support groups may be helpful for some patients and families.

The priority intervention in the emergency department for the patient with a stroke is a. intravenous fluid replacement b. administration of osmotic diuretics to reduce cerebral edema c. initiation of hypothermia to decrease the oxygen needs of the brain d. maintenance of respiratory function with a patent airway and oxygen administration

D: Maintenance of respiratory function with a patent airway and oxygen administration- the first priority in acute management of the patient with a stroke is preservation of life. Because the patient with a stroke may be unconscious or have a reduced gag reflex, it is most important to maintain a patent airway for the patient and provide oxygen if respiratory effort is impaired. IV fluid replacement, treatment with osmotic diuretics, and perhaps hypothermia may be used for further treatment.

A patient's wife asks the nurse why her husband did not receive the clot busting medication (tPA) she has been reading about. Her husband is diagnosed with a hemorrhagic stroke. What should the nurse respond? a. He didn't arrive within the time frame for that therapy b. Not every is eligible for this drug. Has he had surgery lately? c. You should discuss the treatment of your husband with your doctor d. The medication you are talking about dissolves clots and could cause more bleeding in your husband's head

D: The medication you are talking about dissolves clots and could cause more bleeding in your husband's head- tPA dissolves clots and increases the risk for bleeding. It is not used with hemorrhagic strokes. If the patient had a thrombotic/embolic stroke the time frame would be important as well as a history of surgery. The nurse should answer the question as accurately as possible and then encourage the individual to talk with the primary care physician if he or she has further questions.

A patient comes to the emergency department immediately after experiencing numbness of the face and an inability to speak, but while the patient awaits examination, the symptoms disappear and the patient request discharge. The nurse stresses that it is important for the patient to be evaluated primarily because a. the patient has probably experienced an asymptomatic lacunar stroke b. the symptoms are likely to return and progress to worsening neurologic deficit in the next 24 hours c. neurologic deficits that are transient occur most often as a result of small hemorrhages that clot off d. the patient has probably experienced a transient ischemic attack (TIA), which is a sign of progressive cerebral vascular disease

D: The patient has probably experienced a transient ischemic attack (TIA), which is a sign of progressive cerebral vascular disease- A TIA is a temporary focal loss of neurologic function caused by ischemia of an area of the brain, usually lasting only about 3 hours. TIAs may be due to microemboli from heart disease or carotid or cerebral thrombi and are a warning of progressive disease. Evaluation is necessary to determine the cause of the neurologic deficit and provide prophylactic treatment if possible.

Tensilon Test

Diagnosis of Myasthenia Gravis - gets better + if gets worse -

Dilantin and Birth Control

Dilantin decreases effectiveness of BC

Babinski Reflex

Dorsiflexion of the big toe with extension by stroking the lateral aspect of the sole of the foot

GCS Eye- opening

Eye- opening 4 Spontaneous = 4 In response to sound = 3 In response to pain = 2 No response = 1

Neuro EMERGENCY . that indicates transtentorial herniation of the brain:

FIXED, UNILATERALLY DILATED PUPIL

Cranial nerve VII

Facial; movement of facial muscles, secretions of lacrimal and salivary glands, taste in anterior two-thirds of tongue

Spinal Shock

Flaccid paralysis Loss of reflex activity below the level of injury Bradycardia Paralytic ileus Hypotension

Subdural hematoma

Forms slowly and results from a venous bleed Occur under the dura ad a result of tear in the veins across the subdural space

if rhinorrhea, you should test for:

GLUCOSE!

A patient with multiple sclerosis (MS) is to begin treatment with glatiramer acetate (Copaxone). Which information will the nurse include in patient teaching?

How to draw up and administer injections of the medication rational: Copaxone is administered by self-injection. Oral contraceptives are an appropriate choice for birth control. There is no need to avoid driving or drink large fluid volumes when taking glatiramer.

Cranial nerve XII

Hypoglossal; tongue movement

Any patient who becomes acutely unconscious, regardless of the cause, should be suspected of having....

INCREASED ICP

A patient who has numbness and weakness of both feet is hospitalized with Guillain-Barré syndrome. The nurse will anticipate the need to teach the patient about _____________

IV infusion of immunoglobulin (Sandoglobulin). rational: Because the Guillain-Barré syndrome is in the earliest stages (as evidenced by the symptoms), use of high-dose immunoglobulin is appropriate to reduce the extent and length of symptoms. Mechanical ventilation and tube feedings may be used later in the progression of the syndrome but are not needed now. Corticosteroid use is not helpful in reducing the duration or symptoms of the syndrome.

LOC, retrograde amnesia, memory loss, and headache are COMMON signs/symptoms of which type of brain injury?

concussion

Potentially life-threatening situation that results from an increase in any or all of the three components in the skull

Increased ICP

Quadriplegia

Injury between C1 and C8 All 4 extremities

Paraplegia

Injury between T1 and L4 Only lower extremities

Skull Tongs

Inserted into outer aspect of the clients skull and traction is applied

Which action will the nurse take when evaluating a patient who is taking phenytoin (Dilantin) for adverse effects of the medication?

Inspect the oral mucosa. rational: Phenytoin can cause gingival hyperplasia, but does not affect bowel tones, lung sounds, or pupil reaction to light.

An unconscious patient has a nursing diagnosis of ineffective cerebral tissue perfusion related to cerebral tissue swelling. Which nursing intervention will be included in the plan of care?

Keep the head of the bed elevated to 30 degrees. rational: The patient with increased intracranial pressure (ICP) should be maintained in the head-up position to help reduce ICP. Flexion of the hips and knees increases abdominal pressure, which increases ICP. Because the stimulation associated with nursing interventions increases ICP, clustering interventions will progressively elevate ICP. Coughing increases intrathoracic pressure and ICP.

Method of direct ICP monitoring & brain oxygenation measurement in which a probe is placed in healthy white brain matter (measures brain oxygenation and temperature):

LICOX

GCS Motor

Motor 6 Obeys simple response = 6 Localize painful stimuli= 5 Normal flexion (withdrawl) = 4 Decorticate = 3 Decerebrate = 2 No motor response to pain = 1

Intracerebral hemorrhage

Multiple hemorrhages occur around a contused area

What type of analgesics should be avoided for ICP treatment?

NSAIDS

Following a thymectomy, a patient with myasthenia gravis receives the usual dose of pyridostigmine (Mestinon). An hour later, the patient complains of nausea and severe abdominal cramps. Which action should the nurse take first?

Notify the patient's health care provider. rational: The patient's history and symptoms indicate a possible cholinergic crisis. The health care provider should be notified immediately, and it is likely that atropine will be prescribed. The other actions will be appropriate if the patient is not experiencing a cholinergic crisis.

When the nurse is assessing a patient with myasthenia gravis, which action will be most important to take?

Observe respiratory effort. rational: Because respiratory insufficiency may be life threatening, it will be most important to monitor respiratory function. The other data also will be assessed but are not as critical.

Cranial nerve III

Oculomotor; four eye movement muscles, smooth muscle in eyeball (motor)-connection with midbrain

Cranial nerve I

Olfactory; smell (sensory) - connection with cerebrum

Cranial nerve II

Optic; vision (sensory)-connection with thalamus

Mannitol (Osmitrol)

Osmotic diuretic Decrease ICP by drawing fluid from the interstitial space

which neurons can regenerate?

PNS neurons

Left-brain damage

Paralysis on right side impaired speech/aphasias Impaired discrimination Slow performance Aware of deficits, depression, anxiety Impaired comprehension related to language, math

Which of these nursing actions for a patient with Guillain-Barré syndrome is most appropriate for the nurse to delegate to an experienced nursing assistant?

Passive range of motion to extremities q8hr rational: Assisting a patient with movement is included in nursing assistant education and scope of practice. Administration of tube feedings, administration of ordered medications, and assessment are skills requiring more education and scope of practice, and the RN should perform these skills.

What is a positive Romberg's test?

Pt falls when they close their eyes standing up -- indicates increased ICP

Neurogenic Hyperventilation Respirations

Regular, rapid, and deep sustained respirations

Cheyne-Stokes respirations

Rhytmis, with periods of apnea

symptoms of brain contusions are similar to those of

SHOCK (motionless, shallow respirations, cool/pale skin, incontinence, decreased BP and temp)

Which action will the nurse in the outpatient clinic include in the plan of care for a patient with mild cognitive impairment (MCI)?

Schedule the patient for more frequent appointments. rational: Ongoing monitoring is recommended for patients with MCI. MCI does not interfere with activities of daily living, acetylcholinesterase drugs are not used for MCI, and an assisted living facility is not indicated for MCI.

Parkinson's Medications

Sinemet, Parlodel, Permax, Requip, Mirapex, Artane, Cogentin, Benadryl, Entacapone

Nimdipine (Nimotop)

Subarachnoid hemorrhage therapy agent Avoid Grapefruit

Autonomic Dysreflexia

Sudden onset, severe throbbing headache Severe Hypertension Flushing above the level of the injury

A patient with Parkinson's disease has a nursing diagnosis of impaired physical mobility related to bradykinesia. Which action will the nurse include in the plan of care?

Suggest that the patient rock from side to side to initiate leg movement. rational: Rocking the body from side to side stimulates balance and improves mobility. The patient will be encouraged to continue exercising because this will maintain functional abilities. Maintaining a wide base of support will help with balance. The patient should lift the feet and avoid a shuffling gait.

A patient with multiple sclerosis (MS) has urinary retention caused by a flaccid bladder. Which action will the nurse plan to take?

Teach the patient how to use the Credé method. rational: The Credé method can be used to improve bladder emptying. Decreasing fluid intake will not improve bladder emptying and may increase risk for urinary tract infection (UTI) and dehydration. The use of incontinence briefs and frequent toileting will not improve bladder emptying.

Which nursing action will the home health nurse include in the plan of care for a patient with paraplegia in order to prevent autonomic dysreflexia?

Teach the purpose of a prescribed bowel program. rational: Fecal impaction is a common stimulus for autonomic dysreflexia. The other actions may be included in the plan of care but will not reduce the risk for autonomic dysreflexia.

Trigeminal Neuralgia Medications

Tegretol, Trileptal, local anesthetics, biofeedback

Apraxia

The inability to carry out learned sequential movements on demand

Agnosia

The inability to recognize an object by sight, touch, or hearing

brain injury characterized by a mild blow & the shearing of brain cells

concussion

Which information about a patient with MS indicates that the nurse should consult with the health care provider before giving the prescribed dose of fampridine (Ampyra)?

The patient has an increased creatinine level. rational: Fampridine should not be given to patients with impaired renal function. The other information will not impact on whether the fampridine should be administered.

When caring for a patient who has Guillain-Barré syndrome, which assessment data obtained by the nurse will require the most immediate action?

The patient has continuous drooling of saliva. rational: Drooling indicates decreased ability to swallow, which places the patient at risk for aspiration and requires rapid nursing and collaborative actions such as suctioning and possible endotracheal intubation. The foot pain should be treated with appropriate analgesics, and the BP requires ongoing monitoring, but these actions are not as urgently needed as maintenance of respiratory function. Absence of the reflexes should be documented, but this is a common finding in Guillain-Barré syndrome.

A patient has a new prescription for bromocriptine (Parlodel) to control symptoms of Parkinson's disease. Which information obtained by the nurse may indicate a need for a decrease in the dose?

The patient's blood pressure is 90/46 mm Hg. rational: Hypotension is an adverse effect of bromocriptine, and the nurse should check with the health care provider before giving the medication. Diarrhea, cough, and deep vein thrombosis are not associated with bromocriptine use.

difficulty awakening, dysphasia, confusion, severe headache, vomiting, and one-sided weakness are EMERGENCY s/s's of which type of brain injury?

concussion

Ataxic Respirations

Totally Iregular in rhythm and depth

Cranial nerve V

Trigeminal: connection with pons (sensory) Opthalmic: sensation of forehead, eye , superior nasal cavity Maxillary: sensation from inferior nasal cavity, upper teeth, mucosa of superior mouth Mandibular: sensation from surface of jaw. lower teeth, mucosa of lower mouth, anterior tongue

Cranial nerve IV

Trochlear;one eye movement muscle, superior oblique muscle (motor)-connection with medbrain

Which information about a patient who is being treated with carbidopa/levodopa (Sinemet) for Parkinson's disease is most important for the nurse to report to the health care provider?

Uncontrolled head movement rational: Dyskinesia is an adverse effect of the Sinemet, indicating a need for a change in medication or decrease in dose. The other findings are typical with Parkinson's disease.

Cranial X

Vagus; swallowing, parasympathetic nervous system activities such as heart rate, respiratory rate, digestion, sensation in pharynx and larynx

Sumatriptan (Imitrex)

Vascular headache suppressant Acute treatment of cluster headaches- migraine

GCS Verbal

Verbal 5 Oriented = 5 Confused = 4 Inappropriate response = 3 Sounds = 2 No verbal = 1

Cranial nerve VIII

Vestibulocochlear; sense of hearing and equilibrium

Occipital Lobe

Visual Area

What does Doll's Eye reflex look like?

When client moves head side to side, their eyes do not track. Fixed, midline gaze

rapid diagnosis of suspected viral meningitis is performed with:

Xpert EV test (sample of CSF is evaluated for enterovirus)

Identify two ways the following three-volume components of intracranial pressure (ICP) can be changed to adapt to small increases in intracranial pressure. a. Cerebrospinal fluid (CSF) b. Brain Tissue c. Blood tissue

a. Increased absorption, decreased production, displacement into spinal canal b. herniation, lesion, edema, collapse of veins and dural sinuses, increased venous outflow and decreased blood flow c. distention of dura, slight compression of tissue

main treatment for skull fractures:

conservative treatment ; surgery if depressed

15. Priority Decision: The nurse finds a patient in bed having a generalized tonic-clonic seizure. During the seizure activity, what actions should the nurse take (select all that apply)? a. Loosen restrictive clothing. b. Turn the patient to the side. c. Protect the patient's head from injury. d. Place a padded tongue blade between the patient's teeth. e. Restrain the patient's extremities to prevent soft tissue and bone injury.

a, b, c. The focus is on maintaining a patent airway and preventing patient injury. The nurse should not place objects in the patient's mouth or restrain the patient.

In which patient would it be the most important for the nurse to assess the glossopharyngeal and vagus nerves? a. A 50-yr-old woman with lethargy from a drug overdose b. A 40-yr-old man with a complete lumbar spinal cord injury c. A 60-yr-old man with severe pain from trigeminal neuralgia d. A 30-yr-old woman with a high fever and bacterial meningitis

a. A 50-yr-old woman with lethargy from a drug overdose The glossopharyngeal and vagus nerves innervate the pharynx and are tested by the gag reflex. It is important to assess the gag reflex in patients who have a decreased level of consciousness, brainstem lesion, or disease involving the throat musculature. If the reflex is weak or absent, the patient is in danger of aspirating food or secretions.

The nurse is completing a health assessment for a newly admitted patient. Which assessment should the nurse perform to determine the cognitive function of the patient? a. Ask the patient a question such as, "Who were the last three presidents?" b. Determine the level of consciousness, body posture, and facial expressions. c. Observe for signs of agitation, anger, or depression during the health check. d. Request that the patient mimic rapid alternating movements with both hands.

a. Ask the patient a question such as, "Who were the last three presidents?" Cognition is one component of the mental status examination to determine cerebral functioning. Cognition is assessed by determining orientation, memory, general knowledge, insight, judgment, problem solving, and calculation. A question often used to determine cognition for adults living in the United States is, "Who were the last three presidents?" General appearance and behavior are additional components and include level of consciousness, body posture, and facial expressions. Mood and affect are assessed by observing for agitation, anger, or depression. Cerebellar function is determined by assessing balance and coordination, and may include testing rapid alternating movements of the upper and lower extremities.

The nurse is caring for a patient after a lumbar puncture. Which is a priority action by the nurse? a. Assess for drainage or bleeding from the puncture site. b. Monitor for bladder dysfunction and bowel incontinence. c. Maintain bed rest until lower extremities move normally. d. Check for loss of muscle strength in the upper extremities.

a. Assess for drainage or bleeding from the puncture site. After a lumbar puncture, the nurse should monitor the puncture site for drainage or bleeding. Other assessments include headache intensity, meningeal irritation (nuchal rigidity), signs and symptoms of local trauma (e.g., hematoma, pain), neurologic signs, and vital signs. A lumbar puncture does not affect bowel or bladder function or upper extremity muscle strength. Bed rest until lower extremity movement returns is indicated for the patient after spinal anesthesia.

When assessing a patient with a traumatic brain injury, the nurse notes uncoordinated movement of the extremities. How should the nurse document this assessment? a. Ataxia b. Apraxia c. Anisocoria d. Anosognosia

a. Ataxia Ataxia is a lack of coordination of movement, possibly caused by lesions of sensory or motor pathways, cerebellum disorders, or certain medications. Apraxia is the inability to perform learned movements despite having the desire and physical ability to perform them related to a cerebral cortex lesion. Anisocoria is inequality of pupil size from an optic nerve injury. Anosognosia is the inability to recognize a bodily defect or disease related to lesions in the right parietal cortex.

The patient with type 1 diabetes mellitus is having a seizure. Which medication should the nurse anticipate will be administered first? a. IV dextrose solution b. IV diazepam (Valium) c. IV phenytoin (Dilantin) d. Oral carbamazepine (Tegretol)

a. IV dextrose solution This patient's seizure could be caused by low blood glucose, so IV dextrose solution would be given first to stop the seizure. IV diazepam, IV phenytoin, and oral carbamazepine would be used first to treat seizures from other causes such as head trauma, drugs, and infections. These drugs will be tried if the IV dextrose is ineffective.

The nurse is caring for a patient with a neurologic disease that affects the pyramidal tract. What clinical manifestation does the nurse assess in this patient? a. Impaired muscle movement b. Decreased deep tendon reflexes c. Decreased level of consciousness d. Impaired sensation of touch, pain, and temperature

a. Impaired muscle movement Among the most important descending tracts are the corticobulbar and corticospinal tracts, collectively termed the pyramidal tract. These tracts carry volitional (voluntary) impulses from the cortex to the cranial and peripheral nerves. Dysfunction of the pyramidal tract is likely to manifest as impaired movement because of hypertonicity. Diseases affecting the pyramidal tract do not result in changes in level of consciousness, impaired reflexes, or decreased sensation.

A 48-yr-old man was just diagnosed with Huntington's disease. His 20-yr-old son is upset about his father's diagnosis. What is the nurse's best response? a. Provide emotional and psychologic support. b. Encourage him to get diagnostic genetic testing. c. Explain that cognitive deterioration will be treated with counseling. d. Instruct that chorea and psychiatric disorders can be treated with haloperidol (Haldol).

a. Provide emotional and psychologic support. The patient's son will first need emotional and psychologic support. He should be taught about diagnostic genetic testing but should decide for himself with a genetic counselor if and when he wants this done. The treatment plan for his father will be determined based on his father's needs.

Which care measure is a priority for a patient with multiple sclerosis (MS)? a. Vigilant infection control and adherence to standard precautions b. Careful monitoring of neurologic assessment and frequent reorientation c. Maintenance of a calorie count and hourly assessment of intake and output d. Assessment of blood pressure and monitoring for signs of orthostatic hypotension

a. Vigilant infection control and adherence to standard precautions Infection control is a priority in the care of patients with MS because infection is the most common cause of an exacerbation of the disease. Decreases in cognitive function are less likely, and MS does not typically result in malnutrition, hypotension, or fluid volume excess or deficit.

A patient with osteomyelitis is treated with surgical debridement with implantation of antibiotic beads. When the patient asks why the beads are used, the nurse answers (select all that apply) a."The beads are used to directly deliver antibiotics to the site of the infection." b."There are no effective oral or IV antibiotics to treat most cases of bone infection." c."This is the safest method of delivering long-term antibiotic therapy for a bone infection." d."The beads are an adjunct to debridement and oral and IV antibiotics for deep infections." e."The ischemia and bone death that occur with osteomyelitis are impenetrable to IV antibiotics."

a."The beads are used to directly deliver antibiotics to the site of the infection." d."The beads are an adjunct to debridement and oral and IV antibiotics for deep infections." Treatment of chronic osteomyelitis includes surgical removal of the poorly vascularized tissue and dead bone and the extended use of IV and oral antibiotics. Antibiotic-impregnated polymethylmethacrylate bead chains may be implanted during surgery to aid in combating the infection.

the following are characteristic of which type of hematoma? -bleeding between dura & arachnoid -assoc w/ contusion or laceration -rapid headache, <LOC, coma -ipsilateral pupil dilation -good prognosis with surgery

acute subdural

nutritional-metabolic

ale to feed yourself adequate nutrition chewing, swallowing difficulties

Patients being given corticosteroids should also be given ____ or _____ why?

antacids/histamine receptor blockers (cimetidine, ranitidine) OR PPIs (omeprazole, pantoprazole) *prevent GI ulcers and bleeding

A patient is seen in the health clinic with symptoms of a stooped posture, shuffling gait, and pill rolling-type tremor. The nurse will anticipate teaching the patient about _______________

antiparkinsonian drugs. rational: The diagnosis of Parkinson's is made when two of the three characteristic signs of tremor, rigidity, and bradykinesia are present. The confirmation of the diagnosis is made on the basis of improvement when antiparkinsonian drugs are administered. This patient has symptoms of tremor and bradykinesia; the next anticipated step will be treatment with medications. MRI and EEG are not useful in diagnosing Parkinson's disease, and corticosteroid therapy is not used to treat it.

elimination

any incontinence hesitancy, urgency or retention postpone urination or defication medication to manage neurologic elimination problems

activity-exercise

any problems with usual activities weakness or lack of coordination independent with personal hygiene ADL's

pronator drift

arms out straight, close eyes-if 1 arm drops down this indicates weakness in upper extremity

finger to nose test

assesses for balance

walking heel to toe

assesses muscle tone

motor function assessment

assesses strength, tone, coordination and symmetry of major muscle groups

A patient with myasthenia gravis is admitted to the hospital with respiratory insufficiency and severe weakness. When is a diagnosis of cholinergic crisis made? a. The patient's respiration is impaired because of muscle weakness. b. Administration of edrophonium (Tensilon) increases muscle weakness. c. Administration of edrophonium (Tensilon) results in improved muscle contractility. d. EMG reveals decreased response to repeated stimulation of muscles.

b. The reduction of the acetylcholine (ACh) effect in myasthenia gravis (MG) is treated with anticholinesterase drugs, which prolong the action of ACh at the neuromuscular synapse, but too much of these drugs will cause a cholinergic crisis with symptoms very similar to those of MG. To determine whether the patient's manifestations are due to a deficiency of ACh or to too much anticholinesterase drug, the anticholinesterase drug edrophonium chloride (Tensilon) is administered. If the patient is in cholinergic crisis, the patient's symptoms will worsen; if the patient is in a myasthenic crisis, the patient will improve.

The nurse provides information to the caregiver of a 68-yr-old man with epilepsy who has tonic-clonic seizures. Which statement by the caregiver indicates a need for further teaching? a. "It is normal for a person to be sleepy after a seizure." b. "I should call 911 if breathing stops during the seizure." c. "The jerking movements may last for 30 to 40 seconds." d. "Objects should not be placed in the mouth during a seizure."

b. "I should call 911 if breathing stops during the seizure." Caregivers do not need to call an ambulance or send a person to the hospital after a single seizure unless the seizure is prolonged, another seizure immediately follows, or extensive injury has occurred. Altered breathing is a clinical manifestation of a tonic-clonic seizure. Contact emergency medical services (or call 911) if breathing stops for more than 30 seconds. No objects (e.g., oral airway, padded tongue blade) should be placed in the mouth. Lethargy is common in the postictal phase of a seizure. Jerking of the extremities occurs during the clonic phase of a tonic-clonic seizure. The clonic phase may last 30 to 40 seconds.

The nurse teaches a 38-yr-old woman who has migraine headaches about sumatriptan (Imitrex). Which statement by the patient requires clarification by the nurse? a. "The injection might feel like a bee sting." b. "This medicine will prevent a migraine headache." c. "I can take another dose if the first does not work." d. "This drug for migraine headaches could cause birth defects."

b. "This medicine will prevent a migraine headache." Sumatriptan is given to abort an ongoing migraine headache and is not used to prevent migraine headaches. When given as a subcutaneous injection, this drug may cause transient pain and redness at the injection site. This drug may be repeated after a specified time period if the first dose is not effective. This drug should be avoided during pregnancy and is classified as a Food and Drug Administration Pregnancy Risk Category C drug.

The nurse is performing a neurologic assessment for a patient. When assessing the accessory nerve, what action should the nurse take? a. Assess the gag reflex by stroking the posterior pharynx. b. Ask the patient to shrug the shoulders against resistance. c. Ask the patient to push the tongue to either side against resistance. d. Have the patient say "ah" while visualizing elevation of soft palate.

b. Ask the patient to shrug the shoulders against resistance. The spinal accessory nerve is tested by asking the patient to shrug the shoulders against resistance and to turn the head to either side against resistance while observing the sternocleidomastoid muscles and the trapezius muscles. Assessing the gag reflex and saying "ah" are used to assess the glossopharyngeal and vagus nerves. Asking the patient to push the tongue to either side against resistance is used to assess the hypoglossal nerve.

The nurse observes a 74-yr-old man with Parkinson's disease rocking side to side while sitting in the chair. Which action by the nurse is most appropriate? a. Provide the patient with diversional activities. b. Document the activity in the patient's health record. c. Take the patient's blood pressure sitting and standing. d. Ask if the patient is feeling either anxious or depressed.

b. Document the activity in the patient's health record. Patients with Parkinson's disease are instructed to rock from side to side to stimulate balance mechanisms and decrease akinesia.

A female patient complains of a throbbing headache. The nurse learns the patient has experienced photophobia and headaches previously. Which diagnosis does the nurse suspect? a. Cluster headache b. Migraine headache c. Polycythemia vera d. Hemorrhagic stroke

b. Migraine headache Although a headache may occur with any of these options, a migraine headache is the only one that has a throbbing headache with an aura (the photophobia). Headache from polycythemia vera is from erythrocytosis. The cluster headache pain is sharp and stabbing. The headache associated with a hemorrhagic stroke has a sudden onset and is not recurrent.

A patient has been receiving scheduled doses of phenytoin (Dilantin) and begins to experience diplopia. Which additional findings would the nurse expect? a. An aura or focal seizure b. Nystagmus or confusion c. Abdominal pain or cramping d. Irregular pulse or palpitations

b. Nystagmus or confusion Diplopia is a sign of phenytoin toxicity. The nurse should assess for other signs of toxicity, which include neurologic changes, such as nystagmus, ataxia, confusion, dizziness, or slurred speech. An aura, focal seizure, abdominal pain or cramping, irregular pulse, or palpitations are not associated with phenytoin toxicity.

A 50-yr-old male patient has been diagnosed with amyotrophic lateral sclerosis (ALS). What strategy will prevent a common cause of death for patients with ALS? a. Reduce fat intake. b. Reduce the risk of aspiration. c. Decrease injury related to falls. d. Decrease pain secondary to muscle weakness.

b. Reduce the risk of aspiration. Reducing the risk of aspiration can help prevent respiratory infections that are a common cause of death from deteriorating muscle function. Reducing fat intake may reduce cardiovascular disease, but this is not a common cause of death for patients with ALS. Decreasing injury related to falls and decreasing pain secondary to muscle weakness are important nursing interventions for patients with ALS but are unrelated to causes of death for these patients.

The nurse is caring for an older adult patient. Which normal nervous system changes of aging put this patient at higher risk of falls? Select all that apply. a. Memory deficit b. Sensory deficit c. Motor function deficit d. Cranial and spinal nerves e. Reticular activation system f. Central nervous system changes

b. Sensory deficit c. Motor function deficit f. Central nervous system changes Normal changes of aging in the nervous system decrease the sensory function that leads to poor ability to maintain balance and a widened gait. The motor function deficit decreases muscle strength and agility. The central nervous system changes in the brain lead to a diminished kinesthetic sense or position sense. These changes all contribute to an increased risk of falls for the older adult. Memory deficits, normal changes of cranial and spinal nerves, and the reticular activation system do not increase the risk for falls.

In caring for a patient after a spinal fusion, the nurse would immediately report to the physician which patient symptom? a.The patient experiences a single episode of emesis. b.The patient is unable to move the lower extremities. c. The patient is nauseated and has not voided in 4 hours. d. The patient complains of pain at the bone graft donor site.

b.The patient is unable to move the lower extremities. After spinal fusion surgery, the nurse should frequently monitor peripheral neurologic signs. Movement of the arms and legs and assessment of sensation should be unchanged in comparison with the preoperative status. These assessments are usually repeated every 2 to 4 hours during the first 48 hours after surgery, and findings are compared with those of the preoperative assessment. Paresthesias, such as numbness and tingling sensation, may not be relieved immediately after surgery. The nurse should document any new muscle weakness or paresthesias and report this to the surgeon immediately.

Most dangerous form of meningitis

bacterial

Components of cranium to consider- (3)

brain blood CSF

anoxia/hypoxia refers to

brain cell death from decreased or nonexistent O2 delivery

A 65-year-old woman was just diagnosed with Parkinson's disease. The priority nursing intervention is a. searching the Internet for educational videos. b. evaluating the home for environmental safety. c. promoting physical exercise and a well-balanced diet. d. designing an exercise program to strengthen and stretch specific muscles.

c Rationale: Promotion of physical exercise and a well-balanced diet are major concerns of nursing care for patients with Parkinson's disease.

In providing care for patients with chronic, progressive neurologic disease, what is the major goal of treatment that the nurse works toward? a. Meet the patient's personal care needs. b. Return the patient to normal neurologic function. c. Maximize neurologic functioning for as long as possible. d. Prevent the development of additional chronic diseases.

c. Many chronic neurologic diseases involve progressive deterioration in physical or mental capabilities and have no cure, with devastating results for patients and families. Health care providers can only attempt to alleviate physical symptoms, prevent complications, and assist patients in maximizing function and self-care abilities for as long as possible.

During care of a patient in myasthenic crisis, maintenance of what is the nurse's first priority for the patient? a. Mobility b. Nutrition c. Respiratory function d. Verbal communication

c. The patient in myasthenic crisis has severe weakness and fatigability of all skeletal muscles, affecting the patient's ability to breathe, swallow, talk, and move. However, the priority of nursing care is monitoring and maintaining adequate ventilation.

The nurse provides dietary instructions to the in-home caregiver of a 45-yr-old man with Huntington's disease. The nurse is most concerned if the caregiver makes which statement? a. "Depression is common and may cause a decrease in appetite." b. "If swallowing becomes difficult, a feeding tube may be needed." c. "Calories should be restricted to prevent unnecessary weight gain." d. "Muscles in the face are affected, and chewing may become impossible."

c. "Calories should be restricted to prevent unnecessary weight gain." Patients with Huntington's disease may require 4000 to 5000 calories per day to maintain body weight. Weight loss occurs in patients with Huntington's disease because of choreic movements, difficulty swallowing, depression, and mental deterioration.

The nurse is caring for a group of patients on a medical unit. After receiving report, which patient should the nurse see first? a. A 42-yr-old patient with multiple sclerosis who was admitted with sepsis b. A 72-yr-old patient with Parkinson's disease who has aspiration pneumonia c. A 38-yr-old patient with myasthenia gravis who declined prescribed medications d. A 45-yr-old patient with amyotrophic lateral sclerosis who refuses enteral feedings

c. A 38-yr-old patient with myasthenia gravis who declined prescribed medications Patients with myasthenia gravis who discontinue pyridostigmine (Mestinon) will experience myasthenic crisis. Myasthenia crisis results in severe muscle weakness and can lead to a respiratory arrest.

Which nursing diagnosis is a priority in the care of a patient with myasthenia gravis (MG)? a. Acute confusion b. Bowel incontinence c. Activity intolerance d. Disturbed sleep pattern

c. Activity intolerance The primary feature of MG is fluctuating weakness of skeletal muscle. Bowel incontinence and confusion are unlikely signs of MG. Although sleep disturbance is likely, activity intolerance is of primary concern.

How should the nurse most accurately assess the position sense of a patient with a recent traumatic brain injury? a. Ask the patient to close his or her eyes and slowly bring the tips of the index fingers together. b. Ask the patient to close his or her eyes and identify the presence of a common object on the forearm. c. Ask the patient to stand with the feet together and eyes closed and observe for balance maintenance. d. Place the two points of a calibrated compass on the tips of the fingers and toes and ask the patient to discriminate the points.

c. Ask the patient to stand with the feet together and eyes closed and observe for balance maintenance. The Romberg test is an assessment of position sense in which the patient stands with the feet together and then closes his or her eyes while attempting to maintain balance. The other cited tests of neurologic function do not directly assess position sense.

A patient's sudden onset of hemiplegia has necessitated a CT scan of her head. Which action should be the nurse's priority before this diagnostic study? a. Assess the patient's immunization history. b. Screen the patient for any metal parts or a pacemaker. c. Assess the patient for allergies to shellfish, iodine, or dyes. d. Assess the patient's need for tranquilizers or antiseizure medications.

c. Assess the patient for allergies to shellfish, iodine, or dyes. Allergies to shellfish, iodine, or dyes contraindicate the use of contrast media in CT. The patient's immunization history is not a central consideration, and the presence of metal in the body does not preclude the use of CT as a diagnostic tool. The need to assess for allergies supersedes the need for tranquilizers or antiseizure medications in the majority of patients.

When establishing a diagnosis of multiple sclerosis (MS), which diagnostic tests will the nurse expect? Select all that apply. a. EEG b. ECG c. CT scan d. Carotid duplex scan e. Evoked response testing f. Cerebrospinal fluid analysis

c. CT scan e. Evoked response testing f. Cerebrospinal fluid analysis No definitive diagnostic test exists for MS. Along with history and physical examination, CT scan, evoked response testing, cerebrospinal fluid analysis, and MRI are used to establish a diagnosis of MS. EEG, ECG, and carotid duplex scan are not used to diagnose MS.

The nurse is admitting a patient with a diagnosis of frontal lobe dementia. What functional difficulties should the nurse expect in this patient? a. Lack of reflexes b. Endocrine problems c. Higher cognitive function abnormalities d. Respiratory, vasomotor, and cardiac dysfunction

c. Higher cognitive function abnormalities Because the frontal lobe is responsible for higher cognitive function, this patient may have difficulty with memory retention, voluntary eye movements, voluntary motor movement, and expressive speech. The lack of reflexes would occur if the patient had problems with the reflex arcs in the spinal cord. Endocrine problems would be evident if the hypothalamus or pituitary gland were affected. Respiratory, vasomotor, and cardiac dysfunction would occur if there were a problem in the medulla.

The nurse is caring for a group of healthy older adults at a community day center. Which neurologic finding associated with aging would the nurse expect to note in older adults? a. Quicker reaction time b. Improved sense of taste c. Orthostatic hypotension d. Hyperactive deep tendon reflexes

c. Orthostatic hypotension Older adults are more likely to experience orthostatic hypotension related to altered coordination of neuromuscular activity. Other neurologic changes in older adults include atrophy of taste buds with decreased sense of taste, below-average reflex score (and diminished deep tendon reflexes), and slowed reaction times.

A male patient with a diagnosis of Parkinson's disease (PD) is admitted to a long-term care facility. Which action should the health care team take to promote adequate nutrition for this patient? a. Provide multivitamins with each meal. b. Provide a diet that is low in complex carbohydrates and high in protein. c. Provide small, frequent meals throughout the day that are easy to chew and swallow. d. Provide the patient with a minced or pureed diet that is high in potassium and low in sodium.

c. Provide small, frequent meals throughout the day that are easy to chew and swallow. Nutritional support is a priority in the care of individuals with PD. Patients may benefit from smaller, more frequent meals that are easy to chew and swallow. Multivitamins are not necessary at each meal. Vitamin and protein intake must be monitored to prevent interactions with medications. Introducing a minced or pureed diet is likely premature, and a low carbohydrate diet is not indicated.

The nurse is preparing the patient for an electromyogram (EMG). What should the nurse include in teaching the patient before the test? a. The patient will be tilted on a table during the test. b. It is noninvasive, and there is no risk of electric shock. c. The pain that occurs is from the insertion of the needles. d. The passive sensor does not make contact with the patient.

c. The pain that occurs is from the insertion of the needles. With an EMG, pain may occur when needles are inserted to record the electrical activity of nerve and skeletal muscle. The patient is not tilted on a table during a myelogram. The electroencephalogram is noninvasive without a danger of electric shock. The magnetoencephalogram is done with a passive sensor that does not make contact with the patient.

Which characteristic will the nurse associate with a focal seizure? a. The patient lost consciousness during the seizure. b. The seizure involved both sides of the patient's brain. c. The seizure involved lip smacking and repetitive movements. d. The patient fell to the ground and became stiff for 20 seconds.

c. The seizure involved lip smacking and repetitive movements. Complex focal seizure is characterized commonly by lip smacking and automatisms (repetitive movements that may not be appropriate). Loss of consciousness, bilateral brain involvement, and a tonic phase are associated with generalized seizure activity.

30. To reduce the risk for falls in the patient with Parkinson's disease, what should the nurse teach the patient to do? a. Use an elevated toilet seat. b. Use a walker or cane for support. c. Consciously lift the toes when stepping. d. Rock side to side to initiate leg movements.

c. The shuffling gait of PD causes the patient to be off balance and at risk for falling. Teaching the patient to use a wide stance with the feet apart, to lift the toes when walking, and to look ahead helps to promote a more balanced gait. Use of an elevated toilet seat and rocking from side to side will enable a patient to initiate movement. Canes and walkers are difficult for patients with PD to maneuver and may make the patient more prone to injury.

Which type of seizure occurs in children, is also known as a petit mal seizure, and consists of a staring spell that lasts for a few seconds? a. Atonic b. Simple focal c. Typical absence d. Atypical absence

c. The typical absence seizure is also known as petit mal and the child has staring spells that last for a few seconds. Atonic seizures occur when the patient falls from loss of muscle tone accompanied by brief unconsciousness. Simple focal seizures have focal motor, sensory, or autonomic symptoms related to the area of the brain involved without loss of consciousness. Staring spells in atypical absence seizures last longer than those in typical absence seizures and are accompanied by peculiar behavior during the seizure or confusion after the seizure.

While obtaining subjective assessment data related to the musculoskeletal system, it is particularly important to ask a patient about other medical problems such as a.hypertension. b.thyroid problems. c.diabetes mellitus. d.chronic bronchitis.

c.diabetes mellitus The nurse should question the patient about past medical problems because certain illnesses are known to affect the musculoskeletal system directly or indirectly. These diseases include tuberculosis, poliomyelitis, diabetes mellitus, parathyroid problems, hemophilia, rickets, soft tissue infection, and neuromuscular disabilities.

The bone cells that function in the resorption of bone tissue are called a.osteoids b.osteocytes c.osteoclasts d.osteoblasts

c.osteoclasts Osteoclasts participate in bone remodeling by assisting in the breakdown of bone tissue.

Neuron function

carry impulses to and from the cell body

side effects of bacT meningitis include

cerebral edema and increased ICP

cognitive-perceptual

changes in memory dizziness, heat cold sensitivity numbness or tingling chronic pain difficulty with written or verbal communication changes in vision or hearing

role-relationship

changes in relationships with family

the following are characteristic of which type of hematoma? -blood encased in membrane -s/s develop over weeks or months following injury -gradual onset of headache and behavioral changes -ipsilateral pupil dilation -frequently attributed to senility (and ETOH -- increased fall risk)

chronic subdural

brain stem

compose of midbrain, pons, medulla

The nurse is reinforcing teaching with a newly diagnosed patient with amyotrophic lateral sclerosis. Which statement would be appropriate to include in the teaching? a. "ALS results from an excess chemical in the brain, and the symptoms can be controlled with medication." b. "Even though the symptoms you are experiencing are severe, most people recover with treatment." c. "You need to consider advance directives now, since you will lose cognitive function as the disease progresses." d. "This is a progressing disease that eventually results in permanent paralysis, though you will not lose any cognitive function."

d Rationale: The disease results in destruction of the motor neurons in the brainstem and spinal cord, causing gradual paralysis. Cognitive function is maintained. Because there is no cure for amyotrophic lateral sclerosis (ALS), collaborative care is palliative and based on symptom relief. Death usually occurs within 3-6 years after diagnosis.

infectious disease of the CNS characterized by changes in both white and gray matter of spinal cord and brain, leading to brain inflammation

encephalitis

the following are characteristic of which type of hematoma? -bleeding beneath skull & dura -unconsciousness followed by wakefullness & lucid thinking for hours and THEN -rapid LOC -ipsilateral pupil dilation -good prognosis if rapid assessment & sent to OR to stop bleeding

epidural

A patient is having a transsphenoidal hypophysectomy. The nurse should provide preoperative patient teaching about what potential deficit as a result of the surgery? a. Increased heart rate b. Loss of coordination c. Impaired swallowing d. Altered sense of smell

d. Altered sense of smell Using a transsphenoidal approach to remove the pituitary gland includes a risk of damage to the olfactory cranial nerve because the cell bodies of the olfactory nerve are located in the nasal epithelium. With damage to this nerve, the sense of smell would be altered. Increased heart rate, loss of coordination, and impaired swallowing will not be potential deficits from this surgery.

When assessing motor function of a patient admitted with a stroke, the nurse notes mild weakness of the arm. The patient also is unable to hold the arm level. How should the nurse most accurately document this finding? a. Athetosis b. Hypotonia c. Hemiparesis d. Pronator drift

d. Pronator drift Downward drifting of the arm or pronation of the palm is identified as pronator drift. Athetosis is a slow, writhing, involuntary movement of the extremities. Hypotonia is flaccid muscle tone, and hemiparesis is weakness of one side of the body.

Broca's area

expressive speech; coordinates the complex muscular activity that makes speech possible

A patient with heart failure and type 1 diabetes mellitus is scheduled for a positron emission tomogram (PET) of the brain. Which medication prescribed by the health care provider should the nurse expect to administer before the diagnostic study? a. Furosemide 20 mg (IV) b. Alprazolam 0.5 mg (PO) c. Ciprofloxacin 500 mg (PO) d. Regular insulin 6 units (SQ)

d. Regular insulin 6 units (SQ) Patients with type 1 diabetes mellitus must have insulin administered the day of the PET if glucose metabolism is the focus of the PET. Diuretics should not be administered before the PET unless a urinary catheter is inserted. The patient must remain still during the procedure (1 to 2 hours). Sedatives and tranquilizers (e.g., alprazolam) should not be administered before a PET of the brain because the patient may need to perform mental activities, and these medications may affect glucose metabolism. Prophylactic antibiotics are not necessary. Patients are NPO before a PET of the brain and should not receive oral medications (alprazolam and ciprofloxacin).

22. The nurse explains to a patient newly diagnosed with MS that the diagnosis is made primarily by a. spinal x-ray findings. b. T-cell analysis of the blood. c. analysis of cerebrospinal fluid. d. history and clinical manifestations.

d. There is no specific diagnostic test for MS. A diagnosis is made primarily by history and clinical manifestations. Certain diagnostic tests may be used to help establish a diagnosis of MS. Positive findings on MRI include evidence of at least two inflammatory demyelinating lesions in at least two different locations within the central nervous system (CNS). Cerebrospinal fluid (CSF) may have increased immunoglobulin G and the presence of oligoclonal banding. Evoked potential responses are often delayed in persons with MS.

posturing indicative of problems within midbrain or pons

decerebrate

posturing indicative of cervical spinal tract or cerebral hemisphere:

decorticate

sleep-rest

describe sleep pattern any trouble sleeping, and solutions for

sexuality-reproductive

dissatisfied with sexual function sexual dysfunction that may cause relationship problems any need for professional counseling

Babinskis sign

dorsiflexion of the great toe while fanning of other toes (abnormal in anyone over 2 years old)

Why do changes in LOC happen with increased ICP

due to impaired CBF, which affects the cells of the cerebral cortex

meninges

dura mater (outer), arachnoid (middle), pia mater (innermost-blood vessel supply that brings oxygen from the spinal cord)

guideline for avoiding hypOtension in treatment of ICP: Why?

keep MAP >120 *if pt becomes hypotensive, ischemia of brain tissue will occur (limited perfusion)

a basilar skull fracture is...

linear

spinal cord

link between PNS and CNS

diagnostics for meningitis

lumbar puncture bacT cultures CBC XRay CT scan

extensive __________________ destruction may occur from encephalitis

nerve cell

primary functional unit of the nervous system

neurons

main treatment for concussion and contusion:

observation and management of ICP

A 24-year-old patient is hospitalized with the onset of Guillain-Barré syndrome. During this phase of the patient's illness, the most essential assessment for the nurse to carry out is ______________

observing respiratory rate and effort. rational: The most serious complication of Guillain-Barré syndrome is respiratory failure, and the nurse should monitor respiratory function continuously. The other assessments also will be included in nursing care, but they are not as important as respiratory assessment.

What is Cushing Triad?

one or more of any of the following 1) increasing SBP 2) bradycardia w/ full bounding pulse 3) irreg. resp pattern represents LATE sign of increased ICP

When teaching a patient with myasthenia gravis (MG) about management of the disease, the nurse advises the patient to ______________

perform physically demanding activities in the morning. rational: Muscles are generally strongest in the morning, and activities involving muscle activity should be scheduled then. Plasmapheresis is not routinely scheduled but is used for myasthenia crisis or for situations in which corticosteroid therapy should be discontinued. There is no decrease in sensation with MG, and muscle atrophy does not occur because muscles are used during part of the day.

The brain requires a constant flow of ____________ and __________, using a total of 20% of the body's ____________ and 25% of the body's ____________

oxygen, glucose oxygen, glucose

This ocular finding, seen on retinal exam, is a non-specific sign that is associated with long-standing ICP:

papilledema (optic disk swelling)

Right-brain damage

paralysis of left side spatial-perceptual deficits Impulsive Impaired judgement Impaired time

persisten headache, lethargy, personality & behavior changes, shortened attention span, decreased short-term memory, and changes in intellectual ability are all signs of :

postconcussion syndrome

CSF

presence of concentric rings

what is the next step after performing Xpert EV test for viral meningitis diagnosis?

prophylactic administration of broad spectrum antibiotics in case it is bacterial because test takes a while to show results

blood brain barrier

protects brain from harmful agents (some drugs) while allowing nutrients and gases to enter

neuroglial cell function

provide nourishment, support and protection

signs of skull fracture may include:

raccoon eyes and battle's sign (bruising behind ears)

a diffuxe axonal injury is marked by the:

shearing of cells, rapid acceleration-deceleration motion during trauma (similar to contracoup phenomenon but not caused by blunt force trauma. think like a car accident/whiplash type shit)

Babinski sign causes

some disease, alcoholism, drugs, after seizures

value-belief

specific cultural beliefs and attitudes

Some benefits of corticosteroid use for ICP include--

stabilizing cell membrane inhibiting prostaglandins (preventing formation of pro-inflammatory mediators) improve neuro function by improving CBF and restoring autoregulation

why barbituates for ICP therapy?

they decrease cerebral metabolism, causing a decrease in ICP and reduction in cerebral edema and production of a more uniform blood supply to the brain.

complications of meningitis may include:

thrombosis decreased CBF seizures cranial nerve damage bradycardia brain damage death

epidemic encephalitis may be transmitted by

ticks and mosquitos

assessing sensory function

touch, pain, vibration, position

What type of craniectomy includes >keep nasal packing in place >check for CSF leakage on nasal dressing & frequent swallowing post-op may indicate leaking CSF >elevate HOB

transsphenoidal

coping-stress tolerance

usual coping patterns are coping patterns adequate unmet needs by current support system

when an ICP pt is placed on sedation, what must also be done?

ventilator

how does bacterial meningitis causative agent reach the brain and spread?

via bloodstream or ear or sinus infection --causes CSF production to increase, then purulent secretions spread to other areas of brain in the CSF

if culture is negative, the meningitis is ______________ in nature

viral

the cause of encephalitis is ____________ in nature

viral

This "unexpected" sign of increased ICP is due to pressure changes in the cranium

vomiting -- usually NOT preceded by nausea

health perception-health management

what are you usual daily activities alcohol, tobacco, recreational drug use safety practices in car, motorcycle, bicycle HTN, controlled or not hospitalization for neurologic problems

if left untreated, the mortality rate for bacterial meningitis is ...

~100%


Ensembles d'études connexes

Texas Law of Contracts - Chp. 2 Basics of Real Estate Law

View Set

EMT: Chapter 23 [gynecologic emergencies]

View Set

Tennessee Laws and Departmental Rules Common to all Lines

View Set

Environmental Science: review questions 1

View Set

MGT 3020 Chapter 6 Perceptions and Individual decision making

View Set